Head and Neck (Gray's)

¡Supera tus tareas y exámenes ahora con Quizwiz!

C. Usually, deficits of the cranium involve the squamous part of the occipital bone and, in some cases, the posterior aspect of the foramen magnum. If the herniation or protruding brain includes part of the ventricular system (most likely the posterior horn of the lateral ventricles), then it is referred to as meningohydroencephalocele. The deficit in the squamous part of the occipital bone usually occurs at the posterior fontanelle of the skull.

A 1-day-old infant presents with meningohydroencephalocele. Which of the following bones is most commonly affected? A. Squamous part of temporal bone B. Petrous part of temporal bone C. Squamous part of occipital bone D. Sphenoid bone E. Ethmoid bone

D. The greater palatine nerve is responsible for the sensory innervation of the hard palate, or the hard part of the roof of the mouth. The lesser palatine nerve supplies the soft palate and palatine tonsil but is not involved in supply to the hard palate. The posterior superior alveolar nerve supplies multiple structures, including posterior portions of the gums, cheeks, and the upper posterior teeth. However, it is not involved in nerve supply to the hard palate. The inferior alveolar nerve has several branches, including the mental nerve, incisive branch, nerve to the mylohyoid, and inferior dental branch. These nerves do not supply the roof of the mouth and thus are not involved. The lingual nerve supplies taste and general sensation to the anterior two thirds of the tongue.

A 25-year-old medical student discovers that his alarm has not worked and he is running late. Desperate to get to his biochemistry lecture in time, yet unbearably hungry, he quickly throws some bread in the toaster as he gets ready. Despite the toast burning a little, he eats it quickly and rushes out the door. The burnt parts of the toast scratch the roof of his mouth, leaving him with a stinging sensation there. What nerve is collecting this sensation from the hard palate? A. Posterior superior alveolar nerve B. Inferior alveolar nerve C. Lingual nerve D. Greater palatine nerve E. Lesser palatine nerve

B. The lateral pterygoid muscle is a muscle of mastication innervated by the lateral pterygoid nerve of the mandibular division of the trigeminal nerve. The lateral pterygoid acts to protrude the mandible and open the jaw. The anterior portion of temporalis is a muscle of mastication innervated by the deep temporal nerves of the mandibular division of the trigeminal nerve that elevates the mandible when contracted. The medial pterygoid muscle is a muscle of mastication innervated by the mandibular division of the trigeminal nerve. This muscle closes the jaw and works with the contralateral medial pterygoid in side-to-side (grinding) jaw movements. The masseter muscle is a muscle of mastication innervated by the mandibular division of the trigeminal nerve that specifically assists in chewing. The platysma is a thin muscle of facial expression that lies within the superficial fascia of the neck and lower face. It is innervated by the cervical branch of the facial nerve. The platysma produces a slight wrinkling of the surface of the skin of the neck in an oblique direction, depresses the lower jaw, and draws down the lower lip and angle of the mouth.

A 27-year-old man is admitted to the hospital after a middleweight boxing match. During physical examination the strength and symmetry of strength in opening the jaws are tested. Which of the following muscles is the most important in jaw protrusion and depressing the mandible? A. Anterior portion of temporalis B. Lateral pterygoid C. Medial pterygoid D. Masseter E. Platysma

C. If the orbital rim is involved in the fracture, the patient may demonstrate a palpable bony "stepoff" and complain of pain with palpation of the rim. Presentations of anterior cranial fossa fractures often include CSF rhinorrhea and bruising around the eyes.

A 30-year-old man was brought to the emergency department after being involved in a head-on collision with another car. He did not wear a seat belt and was found unconscious in the driver's seat, from which he had to be extricated. Examination revealed that he was poorly responsive with a low blood pressure, tachycardia, and several lacerations about his face and body. There was an obvious deformity of the face but "stepoff" defects were palpated over the root of the nose and the glabella. There was also clear fluid draining from the nasal cavity. Once the patient was stabilized, a CT of the skull was performed and revealed a fracture in the middle region of the anterior cranial fossa. Which of the following complaints will the patient most likely complain of once he regains consciousness? A. Blurred vision B. Diplopia C. Anosmia D. Blindness E. Dry left eye

C. The patient suffers from either a damaged nerve or a trapped muscle; the inferior oblique muscle is defective. Complete or partial ptosis results from an inability to lift the upper eyelid and will not result from damage to the lower orbit. Inability to depress the abducted eyeball will result from damage to the superior oblique muscle. The nerve to the superior oblique and the muscle itself run superomedially and will therefore not be damaged in this patient. An inability to produce tears will result from parasympathetic damage and will only result from damage to the lacrimal nerve or the fibers traveling from the pterygopalatine ganglion.

A 30-year-old man was brought to the emergency department with complaints of severe pain to the right eye and double vision. While playing a game of cricket, he was struck in the face just below the eye. On examination, there was periorbital edema and bruising of the skin around the eye and tenderness over the inferior orbital margin. A defect of the bone along this margin was palpated. A CT scan of the brain and face revealed a displaced fracture of the inferior margin and floor of the orbit along the suture line between the zygomatic and maxillary bones. Which of the following best explains this patient's symptoms? A. Partial ptosis of the upper eyelid B. Complete ptosis of the upper eyelid C. Inability to elevate the abducted eyeball D. Inability to depress the abducted eyeball E. Inability to produce tears

C. The recurrent laryngeal nerve, a branch of the vagus nerve (CN X), supplies motor innervation to all the intrinsic muscles of the larynx excluding the cricothyroid muscle and also supplies sensory fibers to the larynx below the vocal cord. Being two in number (right and left), the left recurrent laryngeal nerve upon emerging from the vagus nerve in the neck hooks (recurs) under the ligamentum arteriosum at the arch of aorta and then travels through the tracheoesophageal groove to the larynx. The right nerve hooks under the right subclavian artery. In the course of their recurrent path into the neck, these nerves are posterior to the middle part of the thyroid gland. Therefore, the recurrent laryngeal nerve may be inadvertently damaged during thyroidectomy. The internal laryngeal nerve, a branch of the superior laryngeal nerve, supplies sensory fibers to the laryngeal mucosa above the vocal cord. Injury to this nerve causes anesthesia in the larynx above the vocal cord. The external laryngeal nerve, also a branch of the superior laryngeal nerve, supplies the cricothyroid muscle. This muscle serves to tense the vocal cord. Paralysis of the external laryngeal nerve results in a monotonous voice because of inability to tense the vocal cord. Paralysis of the superior laryngeal nerve will also result in anesthesia above the vocal cord and loss of the protective reflex of the larynx (GAS Fig. 8-176).

A 30-year-old woman underwent a total thyroidectomy for a cancerous thyroid nodule. Three weeks after the procedure she complains of unresolved hoarseness. Which nerve was most likely traumatized during the operation? A. Internal laryngeal B. External laryngeal C. Recurrent laryngeal D. Superior laryngeal E. Glossopharyngeal

C. The greater petrosal nerve, a parasympathetic branch of the facial nerve, provides innervation to the lacrimal gland in the orbit. The chorda tympani provides innervation to the submandibular and sublingual glands and also taste to the anterior two thirds of the tongue. The deep petrosal nerve carries sympathetic innervation to the blood vessels and mucous glands of the head and neck. The lesser petrosal nerve provides parasympathetic innervation to the parotid gland. The nasociliary nerve provides sensory innervation to the ethmoidal sinuses and the cornea as well as innervation to the skin of the eyelids and superior nose regions.

A 55-year-old woman is diagnosed with a tumor at the base of the skull, resulting in a decrease in tear production. Which of the following nerves is most likely injured? A. Chorda tympani B. Deep petrosal C. Greater petrosal D. Lesser petrosal E. Nasociliary

D. The superior thyroid artery, a branch of the external carotid artery, and the inferior thyroid artery, a branch of the thyrocervical trunk, provide the blood supply to the thyroid and parathyroid glands. The superior thyroid artery, superior thyroid vein, and external branch of the superior laryngeal nerve course together in a neurovascular triad that originates superior to the thyroid gland and lateral to the thyroid cartilage. Because the external branch of the superior laryngeal nerve courses close to the superior thyroid artery, it is at risk of injury during thyroidectomy.

A 60-year-old man is undergoing a thyroidectomy procedure. The superior thyroid artery is ligated and special attention needs to be paid to the adjacent nerve. Which of the following muscles is innervated by this nerve? A. Thyroarytenoid B. Lateral cricoarytenoid C. Posterior cricoarytenoid D. Cricothyroid E. Aryepiglotticus

B. Herpes zoster can affect spinal and cranial nerves. It usually follows reactivation of a dormant varicella infection, which resides in a sensory ganglion. Illness, stress or immunosuppression can reactivate the dormant virus, which may result in both motor and sensory dysfunction. In this case a dominant herpes infection in the geniculate ganglion of the facial nerve has been reactivated, resulting in dysfunction of the facial nerve branch that supplies the stapedius muscle (nerve to the stapedius). The stapedius muscle inserts in the neck of the stapes bone in the middle ear and functions to dampen the vibrations of the stapes by way of putting tension on the neck of the stapes. This results in loudness of sound in the ear (hyperacusis) (GAS Fig. 8-120).

A 72-year-old woman presents at the clinic with a herpes zoster oticus. Upon questioning, she complains that she must hold the phone away from her ear because it sounds too loud. Which cranial nerve is affected by this infection to result in the hyperacusis? A. Hypoglossal B. Facial C. Spinal accessory D. Vagus E. Glossopharyngeal

A. The central retinal artery is the major blood supply to the intima of the eye. It is a branch of the ophthalmic artery and travels with the optic nerve close to the eyeball to get to the intima or retina of the eye. The ciliary arteries arise from the ophthalmic artery and supply the sclera, choroid, conjunctiva, ciliary processes, and rectus muscles. The central retinal artery only rarely arises from the posterior ciliary arteries.

A physician performs an ophthalmoscopic examination on a 45-year-old man and can clearly observe the macula lutea and optic disc. He also sees several branching arteries emanating from the optic disc. These arteries are most likely derived from which of the following? A. Central retinal artery B. Long posterior ciliary artery C. Short posterior ciliary artery D. Ophthalmic artery E. Anterior ciliary artery

C. The jugulodigastric node, also known as the tonsillar lymph node, receives drainage from the palatine tonsils, tongue, and pharynx. It is often enlarged during tonsillitis. The submandibular lymph nodes drain the back of the tongue, gums, upper lip, parts of the lower lip, and sides of the face. They drain into the deep cervical group of nodes. The parotid nodes are located superficially and deep to the parotid gland and drain aspects of the cheek, external acoustic meatus, the lateral aspects of the eyelids and posterior orbit. The submental nodes drain the tip of the tongue bilaterally, the lower lip, and floor of the mouth. Finally, the retropharyngeal lymph nodes drain the nasopharynx, nasal cavities, and auditory tubes.

Accompanied by his parents, an 11-year-old boy visits the outpatient clinic with a history of recurrent infections of his palatine tonsils (tonsillitis). Which of the following lymph nodes is most likely to first become visibly enlarged during tonsillitis? A. Submandibular B. Parotid C. Jugulodigastric D. Submental E. Preauricular

C. The superior paravertebral ganglion (the superior cervical ganglion of each sympathetic trunk) lies at the base of the cranium. It lies deep to the sheath of the internal carotid artery and internal jugular vein (carotid sheath), and anterior to the longus capitis muscle. It contains neurons that supply sympathetic innervation to a number of target organs within the head and also contributes to the cervical plexus. The postsynaptic sympathetic fibers stimulate contraction of the blood vessels (vasomotor) and erector muscles associated with hairs (pilomotor), and cause sweating (sudomotion). Postsynaptic sympathetic fibers in the cranium also innervate dilator muscle of the iris (dilator pupillae); therefore compression causes complete pupillary constriction. It also innervates the muscle that maintains elevation of the upper lid (holding the eye open); therefore compression causes partial ptosis (drooping of the upper lid). Paralysis of the CN III causes complete ptosis. (GAS Fig. 8-190).

After undergoing a thyroidectomy for a cancerous nodule, a 29-year-old woman develops a postoperative hematoma in her neck. Physical examination reveals partial ptosis of her eyelid and pupillary constriction on the side of the hematoma. The hematoma is most likely compressing which ganglion to result in the symptoms? A. Submandibular B. Trigeminal C. Superior cervical D. Geniculate E. Ciliary

C. Superior salivatory nucleus (or superior salivary nucleus) of the facial nerve is a visceromotor cranial nerve nucleus located in the pontine tegmentum. Preganglionic parasympathetic fibers arise from the nucleus and pass laterally to exit the pons with motor VII axons. Then some of the preganglionic fibers pass to the pterygopalatine ganglion via the greater petrosal (via the pterygoid canal, also known as the Vidian canal) and synapse in the pterygopalatine ganglion. Postganglionic efferent fibers travel to innervate the lacrimal gland and the mucosal glands of the nose, palate, and pharynx, whereas the other preganglionic parasympathetic fibers are also distributed partly via the chorda tympani and lingual nerves to the submandibular ganglion, and postsynaptic fibers to the submandibular gland and sublingual gland. Therefore other symptoms in this patient will be reduced salivary secretion from submandibular and sublingual glands.

An MRI performed on a 71-year-old man who presents at the clinic confirms the presence of a brain tumor. Additionally, he complains that the right side of his nasal cavity is dry and irritated, and he frequently uses drops in his right eye, as it is always dry. The tumor probably involves which cranial nerve nucleus? A. Inferior salivatory nucleus B. Dorsal vagal nucleus C. Superior salivatory nucleus D. Edinger-Westphal nucleus E. Nucleus ambiguus

A. An injury to the oculomotor nerve (CN III) would cause the eye to point downward and laterally due to the unopposed contractions of the muscles innervated by the trochlear (CN IV) and abducens (CN VI) nerves (superior oblique and lateral rectus muscles, respectively). The oculomotor nerve also provides innervation to the levator palpebrae superioris; thus, any injury will cause complete ptosis or inability to raise the eyelid. The constriction of the pupil is provided by parasympathetic nerves via the oculomotor nerve. The optic nerve is responsible only for the sensory aspect of light via the retina in the eye. The facial nerve innervates the facial muscles, including the orbicularis oculi, which closes the eylid for the blink reflex. The ciliary ganglion could be damaged in this patient, but the loss of parasympathetic supply will not adequately explain the ptosis of the eyelid. The superior cervical ganglion provides sympathetic innervation to the head and neck including the smooth muscle in the upper lid (superior tarsal muscle of Müller), but no loss of sympathetics is evident in this patient.

An unconscious 48-year-old woman is admitted to the hospital. CT scan reveals a tumor in her brain. When she regains consciousness, her right eye is directed laterally and downward, with complete ptosis of her upper eyelid, and her pupil is dilated. Which of the following structures was most likely affected by the tumor to result in these symptoms? A. Oculomotor nerve B. Optic nerve C. Facial nerve D. Ciliary ganglion E. Superior cervical ganglion

B. The tentorial/uncal herniation described in this case is most likely to occur as a result of a temporal lobe tumor. The uncus is part of the temporal lobe, and when enlarged, it will be compressed against the foramen magnum. This results in the symptoms manifested by damage to the nearby oculomotor nerve. The uncus is not a part of the other named lobes.

An unconscious 54-year-old woman is admitted to the hospital. A CT scan reveals a tumor in her brain, producing a transtentorial herniation. When she regains consciousness, her right eye is directed laterally and downward, with complete ptosis of her upper eyelid and pupillary dilation. Which of the following lobes of the brain is affected by the tumor? A. Parietal B. Temporal C. Occipital D. Frontal E. Parietal and temporal

B. The great auricular nerve is derived from the ventral rami of the second and third cervical nerves and supplies the skin over the angle of the mandible up to the level of the TMJ.

Cardiac pain is referred in some cases to the mandible and the region of the temporomandibular joint. Cutaneous sensation over the angle of the mandible is normally supplied by which of the following nerves? A. Cervical branch of facial B. Great auricular nerve C. Mandibular branch of trigeminal nerve D. Mandibular branch of facial nerve E. Transverse cervical nerve

C. For proper movements of the eye to occur, all the cranial nerves that innervate the extraocular eye muscles are required (oculomotor, trochlear, and abducens nerves). The inferior division of the oculomotor innervates the inferior rectus, the medial rectus, and the inferior oblique. Lateral movement of the eye is initiated by the lateral rectus (abducens nerve), assisted thereafter by the superior oblique (trochlear nerve). The inferior rectus (inferior division of the oculomotor nerve) balances the upward deviation exerted by the superior rectus (superior division of the oculomotor nerve). The medial rectus (superior division of the oculomotor nerve) must relax to facilitate the lateral excursion. Answers A, B, and D all have branches of the trigeminal nerve, which have no role in motor movement of the eye. Finally, for answer E, the superior division innervates the superior rectus and the levator palpebrae; therefore, C is the best answer.

During a routine ophthalmologic exam, the globe, the retina, and the cornea of each eye are tested. Which of the following nerves must be functioning properly if the patient is to be able to turn the eye laterally (abduction) without difficulty and without upward or downward deviation? A. Superior division of oculomotor, ophthalmic nerve, abducens nerve B. Trochlear nerve, abducens nerve, nasociliary nerve C. Inferior division of oculomotor, trochlear, abducens D. Oculomotor and ophthalmic nerves E. Superior division of oculomotor, trochlear, and abducens nerves

E. The anterior fontanelle is located at the junction of the sagittal and coronal sutures and closes at around 18 months of age. The posterior fontanelle is located at the junction of the sagittal suture and lambdoid suture, and it closes at around 2 to 3 months. The mastoid fontanelle is located at the junction of the squamous suture and the lambdoid suture, and it closes at the end of the first year. The sphenoidal fontanelle is located at the junction of the squamous suture and the coronal suture and closes at around 2 to 3 months. There is a lambdoid suture but not a lambdoid fontanelle.

Early closure of the fontanelles of the infant skull can result in compression of the brain, restricting brain growth. Which of the following fontanelles is located at the junction of the sagittal and coronal sutures and at what age does this fontanelle typically close? A. Posterior fontanelle, which closes at about 2 years B. Mastoid fontanelle, which closes at about 16 months C. Lambdoid fontanelle, which closes at 8 months to 1 year D. Sphenoidal fontanelle, which closes at 3 years E. Anterior fontanelle, which closes at 18 months

A. The oculomotor nerve (CN III) supplies the majority of the muscles controlling eye movements. Thus, damage to this nerve results in displaced outward and downward eye as a result of unopposed lateral rectus (innervated by the abducens nerve, CN VI) and superior oblique (innervated by the trochlear nerve, CN IV). The ciliary muscle within the ciliary body changes the shape of the lens. In the absence of nerve stimulation, the diameter of the relaxed muscular ring is larger, making the lens flatter for far vision. Parasympathetic stimulation via the oculomotor nerve (CN III) causes sphincter-like contraction of the ciliary muscle and the lens get thicker for near vision. These fibers also innervate the sphincter pupillae. Thus interruption of these fibers causes dilation of the pupil because of the unopposed action of the sympathetically innervated dilator pupillae muscle. Because the oculomotor nerve supplies the levator palpebrae superioris, a lesion there causes paralysis of the muscle; the patient cannot raise the superior eyelid (cannot open the eye). Ptosis is related more to paralysis of the superior tarsal muscle.

Found unconscious by her husband, a 55-year-old woman is brought to the hospital with a constellation of symptoms that suggests a ruptured aneurysm. Physical examination of the right eye reveals ptosis, dilated pupil, and the eye deviated inferolaterally. Angiography most likely reveals a hematoma compressing which structure? A. Oculomotor nerve B. Optic nerve C. Facial nerve D. Ciliary ganglion E. Superior cervical ganglion

B. The herpetic infection of the forehead likely spread to supratrochlear veins, which join to form a single trunk that runs down near the middle line of the forehead parallel with the vein of the opposite side. The two veins are joined, at the root of the nose, by a transverse branch, called the nasal arch. At the medial angle of the orbit, it joins the supraorbital vein to form the angular vein and drains into the superior ophthalmic vein, which communicates with the cavernous sinus. The infection can also make its way to the inferior ophthalmic vein via facial and deep facial GAS Fig. 8-176 veins to spread to the cavernous sinus. The pterygoid venous plexus has communication with the anterior facial vein and communicates with the cavernous sinus but is not the danger area of the face since it does not drain directly into the pterygoid plexus. The superior petrosal sinus is located in the superior petrosal sulcus of the petrous part of the temporal bone. It drains the cavernous sinus and travels posterolaterally to drain into the transverse sinus. The basilar plexus is made up of multiple venous channels between the layers of the dura mater above the clivus, where it interconnects the two inferior petrosal sinuses. The parietal emissary vein connect the superior sagittal sinus with tributaries of the superficial temporal vein

Herpetic lesions of the forehead require expeditious treatment because the infection can spread to the cavernous sinus, leading to intracranial complications. Which venous structure is the most likely route of transmission of this infectious process to the cavernous sinus? A. Pterygoid venous plexus B. Ophthalmic vein C. Superior petrosal sinus D. Basilar venous plexus E. Parietal emissary vein

C. The superior orbital fissure is an opening in the skull found between the lesser and greater wings of the sphenoid bone and communicates with the orbit and maxillary sinus. Among other structures, it transmits the oculomotor nerve (CN III), branches of the ophthalmic division of trigeminal nerve (CN V1), abducens nerve (CN VI), and ophthalmic vein. Tumors around the superior orbital fissure can compress these structures. Abduction of the eye is done by the oculomotor nerve, and the ophthalmic nerve supplies sensation to the cornea and serves as the afferent limb of the corneal reflex. These functions were lost because of compression of these nerves. The optic canal is found in the anterior cranial fossa and transmits the optic nerve and ophthalmic artery. Similarly, the inferior orbital fissure is located inside of the orbit, transmitting the zygomatic branch of the maxillary nerve and branches of the pterygopalatine ganglion. The foramen rotundum and ovale are located in the middle cranial fossa

Physical examination of an 88-year-old man reveals inability to abduct his right eye and an absent corneal reflex on the same side. These symptoms might indicate a growth in which part of the skull? A. Inferior orbital fissure B. Optic canal C. Superior orbital fissure D. Foramen rotundum E. Foramen ovale

B. The arachnoid villi are extensions of the arachnoid mater into the superior sagittal sinus. The villi allow for proper drainage of the CSF into the venous bloodstream from the subarachnoid space in which the CSF circulates. These villi are a crucial element in maintaining proper intracranial pressure and circulation of the CSF.

The arachnoid villi allow cerebrospinal fluid to pass between which two of the following spaces? A. Choroid plexus and subdural space B. Subarachnoid space and superior sagittal sinus C. Subdural space and cavernous sinus D. Superior sagittal sinus and jugular vein E. Epidural and subdural space

A. Extradural (epidural) hemorrhage is arterial in origin. Blood from torn branches of a middle meningeal artery (which passes through foramen spinosum) collects between the external periosteal layer of the dura and the calvaria. The extravasated blood strips the dura from the cranium. Usually this follows a hard blow to the head; an extradural (epidural) hematoma then forms. Typically, a brief concussion (loss of consciousness) occurs, followed by a lucid interval of some hours. Later, drowsiness and coma (profound unconsciousness) occur, and if the pressure is not relieved the patient will die (GAS Fig. 8-46). GAS

The goalkeeper of a soccer team accidentally hit his head against the goal post while trying to reach for the ball. He was confused for several minutes and later resumed playing. Four hours later, he was found unconscious and was immediately rushed to the emergency department. A CT scan of his head shows a hemorrhage. The vessel that was ruptured to produce this hemorrhage enters the skull through which of the following openings? A. Foramen spinosum B. Foramen ovale C. Jugular foramen D. Hypoglossal canal E. Foramen lacerum

B. The papillary light reflex is mediated by an afferent limb carried by the optic nerve (CN II) and the efferent limb carried by the oculomotor nerve (CN III). The pupil constricts when the light is shone because the optic nerve was able to pick up the light sensation while the oculomotor nerve mediates the contraction of the sphincter pupillae muscle of the iris that results in pupillary constriction. This muscle is specifically innervated by the parasympathetic component of the oculomotor nerve, which has its preganglionic cell bodies at the Edinger-Westphal nucleus and postganglionic fibers at the ciliary ganglion. The pupillary light reflex is not mediated by the combination of the other nerves.

Upon neurologic evaluation of a comatose head trauma patient, the physician performs an examination to evaluate the cranial nerves. A light is directed into the right eye and the pupil constricts. This confirms which two nerves are intact? A. Optic and facial B. Optic and oculomotor C. Maxillary and facial D. Ophthalmic and oculomotor E. Ophthalmic and facial

A. In holoprosencephaly, loss of midline structures results in malformations of the brain and face. There is a single telencephalic vesicle, fused eyes, and a single nasal chamber. Also, there is often hypoplasia of the olfactory bulbs, olfactory tracts, and corpus callosum. Children with Smith-Lemli-Opitz syndrome have craniofacial and limb defects and 5% have holoprosencephaly. Schizencephaly is rare and is characterized by large clefts in the cerebral hemispheres, which in some cases cause a loss in brain tissue. Exencephaly is caused by failure of the cephalic part of the neural tube to close; therefore, the skull does not close, leaving the brain exposed. Meningohydroencephalocele is a deficit of the cranium involving the squamous part of the occipital bone and, in some cases, the posterior aspect of the foramen magnum. It can include the meninges if the herniation or protruding brain includes part of the ventricular system.

A 1-day-old male infant presents with a telencephalic vesicle; the eyes are fused, and a single nasal chamber is present in the midline. In addition an MRI examination reveals that the olfactory bulbs and tracts and the corpus callosum are hypoplastic. Which of the following is the most likely diagnosis? A. Holoprosencephaly B. Smith-Lemli-Opitz syndrome C. Schizencephaly D. Exencephaly E. Meningoencephalocele

C. The area indicated by the arrow is the mastoid air cells of the mastoid process. The infection likely spread via mastoid emissary veins, which passed through the mastoid foramen of the temporal bone to the sigmoid sinus. The sigmoid sinus begins beneath the temporal bone and travels to the jugular foramen, at which point it joins the inferior petrosal sinus to form the jugular vein. The cavernous sinus does not have any direct relations to the mastoid process or mastoid air cells. The pterygoid venous plexus is located in the infratemporal fossa and communicates with the anterior facial vein and the cavernous sinus, by branches through the sphenoidal emissary foramen (of Vesalius), foramen ovale, and foramen lacerum. It has no relation to the mastoid process or mastoid emissary veins. The straight sinus is located within the dura mater, where the falx cerebri meets the midline of tentorium cerebelli. The superior petrosal sinus is located in the petrosal sulcus on the petrous part of the temporal bone. It receives blood from the cavernous sinus and passes backward and laterally to drain into the transverse sinus.

A 10-year-old child appearing acutely ill is seen in the emergency department because of severe ear pain, headache, and fever for the past five days. Physical examination reveals a red and bulging tympanic membrane with a purulent effusion, and tenderness over the area indicated by the arrow in the coronal CT (Fig. 7-28). Assuming that this area is now infected, which of the following venous channels is most at risk for thrombosis as a direct result of proximity to the infected/inflamed bone? A. Cavernous sinus B. Pterygoid venous plexus C. Sigmoid sinus D. Straight sinus E. Superior petrosal sinus

B. The gag reflex is composed of both an afferent and an efferent limb. These reflexes are mediated by the glossopharyngeal and vagus nerves, respectively. Together, the glossopharyngeal and vagus nerves are responsible for the contraction of the muscles of the pharynx involved in the gag reflex. In this case the glossopharyngeal nerve was injured when the tonsils were excised, resulting in the loss of the sensory side of the reflex. The mandibular and maxillary nerves are part of the trigeminal nerve and are thus largely associated with the sensory supply of the face, sinuses, and oral cavity. The hypoglossal nerve innervates most of the muscles of the tongue. The facial nerve is involved with taste of the anterior two thirds of the tongue; however, it does not mediate the gag reflex.

A 10-year-old girl is admitted to the hospital with tonsillitis. A tonsillectomy is performed and the tonsils are removed. On physical examination 1 week later the patient has absence of the gag reflex on the left when the posterior part of the tongue is depressed. The sensory portion of which of the following nerves was most likely injured? A. Facial B. Glossopharyngeal C. Mandibular D. Maxillary E. Hypoglossal

C. The frontal sinuses are located in the frontal bone above the orbital margin. The maxillary sinus in the cheek region is within the maxillary bone. The ethmoid sinus is located between the nose and the eye. The sphenoidal sinus is within the sphenoid bone and cannot be palpated externally.

A 12-year-old boy complains of symptoms suggestive of sinusitis. During the physical examination the physician taps the area slightly superior to the midportion of the patient's eyebrows and this maneuver elicits pain. Which anatomic area is the physician most likely examining? A. Maxillary sinus B. Temporal bone C. Frontal sinus D. Sphenoid sinus E. Ethmoid sinus

A. Papilledema is optic disc swelling ("edema of the papilla") that is caused by increased intracranial pressure caused increased CSF pressure. If a lumbar puncture is performed in a patient with elevated CSF pressure and fluid is withdrawn from the lumbar cistern, the brain can become displaced caudally and the brainstem is pushed against the tentorial notch. This is a potentially fatal complication known as brain herniation. Separation of the pars optica retinae anterior to the ora serrata, or retinal detachment, may result in vision loss or blindness. A hemorrhage from medial retinal branches may result in damage to the fovea centralis and can result in macular degeneration. Opacity of the lens (cataracts) will cause gradual yellowing and may reduce the perception of blue colors. Cataracts typically progress slowly to cause vision loss and are potentially blinding if untreated. Compression of the optic disc, resulting from increased intrabulbar pressure, will lead to an excessive accumulation of serous fluid in the tissue space.

A 12-year-old boy is admitted to the emergency department with signs of meningitis. To determine the specific type of meningitis (i.e., viral or bacterial), it is necessary to aspirate CSF with a lumbar puncture for laboratory examination. However, before performing a lumbar puncture, it must be established that the CSF pressure is not elevated. What condition in the eye would indicate that CSF pressure is too elevated for a lumbar puncture to be performed? A. Papilledema B. Separation of the pars optica retinae anterior to the ora serrata C. The foveal centralis exhibits hemorrhage from medial retinal branches D. Obvious opacity of the lens E. Pitting or compression of the optic disc

B. The facial nerve innervates the stapedius muscle, which is responsible for limiting movement of the stapes, thereby reducing the intensity of the sound entering the inner ear. The hypoglossal nerve innervates tongue muscles; the spinal accessory nerve supplies the trapezius and sternocleidomastoid muscles; the vagus nerve does not provide any innervation for sound in the ear; and the glossopharyngeal nerve only supplies sensation to the posterior third of the tongue, pharynx (gag reflex), middle ear cavity, and tympanic membrane, and muscle innervation to the stylopharyngeus muscle.

A 34-year-old man complains of hyperacusis (sensitivity to loud sounds). Injury to which of the following cranial nerves is most likely responsible? A. Hypoglossal B. Facial C. Accessory D. Vagus E. Glossopharyngeal

C. Parasympathetic innervation of the parotid gland is provided by axons carried by the glossopharyngeal nerve that emerge from the tympanic plexus of the middle ear as the lesser petrosal nerve. These preganglionic parasympathetic fibers terminate at synapses in the otic ganglion, which supplies the secretory parasympathetic innervation to the parotid gland. Glandular secretions of the nasal cavity, soft palate, and lacrimal gland all receive parasympathetic innervation from the fibers of the greater petrosal nerve and would remain intact following a tympanic plexus lesion. Axons for secretory innervation to the sublingual and submandibular glands are carried by the facial nerve, then course through the chorda tympani, before synapsing in the submandibular ganglion, with postganglionic fibers eventually reaching the glands via the lingual nerve.

A 12-year-old girl is admitted to the emergency department with a middle ear infection. Physical examination reveals a long history of chronic middle ear infections that have produced a lesion in the tympanic plexus in the middle ear cavity. Since the preganglionic parasympathetic fibers that pass through the plexus have been lost, which of the following conditions will be detectable during physical examination? A. Diminished mucus in the nasal cavity B. Diminished mucus on the soft palate C. Diminished saliva production by the parotid gland D. Diminished saliva production by the submandibular and sublingual glands E. Diminished tear production by the lacrimal gland

D. The thyroid gland develops as an outpouching from the floor of the primitive pharynx. It is temporarily connected to the tongue at the foramen cecum by the thyroglossal duct, which then degenerates. It descends in the neck passing anterior to the hyoid bone. Incomplete degeneration of the thyroglossal duct can lead to a cystic mass in the path of the descent of the thyroid gland.

A 12-year-old girl is brought to the physician by her parents because of a sore neck for 4 days. Her temperature is 37.7° C (99.9° F). Physical examination shows a tender swelling anterior to, and just above, the thyroid notch of the neck. The physician explains that there is a cyst in the tract along which the thyroid gland descended. All of the tract tissue must be removed between which of the following two structures to treat this patient's condition? A. Left lobe of thyroid gland and tonsillar fossa B. Right lobe of thyroid gland and epiglottis C. Right lobe of thyroid gland and hyoid bone D. Thyroid isthmus and foramen cecum E. Thyroid isthmus and piriform recess

B. A branch of the facial artery would be of primary concern because its branches supply the oropharynx and it is the primary source of arterial supply to the palatine tonsil. The location of the lingual artery is inferior to the oropharynx and it would be less likely to be injured in the event of a surgical procedure. The superior laryngeal artery is also located lower and would not be subject to injury by surgery in the area of the oropharynx. The ascending pharyngeal artery arises in the carotid triangle from the external carotid artery and gives rise to pharyngeal, palatine, inferior tympanic, and meningeal branches. This vessel is located inferiorly to the site of surgery. Terminal branches of the descending palatine artery could be encountered at the upper pole of the palatine tonsil, but the main stem of the vessel would not be endangered in the surgical treatment here.

A 14-year-old girl has been suffering from peritonsillar abscess (quinsy) on the right side of her oropharynx. During surgical removal of the pathologic tissue, or during incision and drainage of the area, which of the following arteries will be at greatest risk? A. Lingual B. A branch of facial C. Superior laryngeal artery D. Ascending pharyngeal artery E. Descending palatine artery

A. During embryologic development of the pituitary gland, an outgrowth from the roof of the pharynx (Rathke's pouch) grows cephalad and forms the anterior lobe (pars distalis) of the pituitary gland. Since this gland normally occupies the sella turcica, it is most likely a tumor derived from the Rathke's pouch that is extending up into the sella turcica and the space just above it, the suprasellar space.

A 15-year-old boy is admitted to the emergency department with severe headache and hydrocephalus. Radiographic examination reveals a craniopharyngioma occupying the sella turcica, primarily involving the suprasellar space. Which of the following is the most likely cause of this tumor? A. Persistence of a small portion of Rathke's pouch B. Abnormal development of pars tuberalis C. Abnormal development of the foramina of Monro D. Abnormal development of the alar plates that form the lateral wall of diencephalon E. Abnormal development of diencephalon

B. The area of damage is in the external acoustic meatus, which is supplied by the facial, glossopharyngeal, and vagus nerves. The auriculotemporal and great auricular nerves supply the TMJ and external ear, respectively. The lesser occipital nerves supply the skin on the posterior aspect of the skull. The chorda tympani is responsible for taste to the anterior two thirds of the tongue and sensation to the middle ear. The lesser petrosal nerve and tympanic plexus carry autonomic innervation to and through the middle ear and are not associated with the external ear canal.

A 15-year-old boy was brought to the emergency department with complaints of right ear pain. While attempting to clean his itchy ear with a swab, his little brother bumped into his elbow, causing the stick to penetrate deeply into the ear. On examination with the otoscope, the tympanic membrane was pearly white and there was no cone of light visible. There was clotted blood in the external auditory meatus. Rinne's test was positive (bone conduction was better than air conduction). Which of the following best describes the nerves responsible for the perception of pain from the injured area? A. Auriculotemporal and great auricular nerves B. Facial, glossopharyngeal, and vagus nerves C. Lesser occipital and great auricular nerve D. Chorda tympani and glossopharyngeal nerve E. Tympanic plexus and lesser petrosal nerve

D. Cavernous sinus thrombosis can often result from squeezing pimples or other infectious processes located around the danger area of the face, which includes the area of the face directly surrounding the nose. This physical pressure has the potential to move infectious agents from the pimple into the ophthalmic vein, which then carries it to the cavernous sinus. The pterygoid venous plexus and ophthalmic vein both communicate with the cavernous sinus and therefore offer a route of travel for the spread of infection, but the path provided by the superior ophthalmic vein is a more direct route. Additionally, the superior ophthalmic vein receives blood supply from the supraorbital, supratrochlear, and angular veins that supply the area around the nose and lower forehead. (Venous blood in the head can flow in either direction because these veins do not possess valves.) The emissary veins communicate between the venous sinuses and the veins of the scalp and would therefore not be involved in the spread of infection between the nose and cavernous sinus. The middle meningeal artery courses between the dura and periosteum, whereas the carotid artery, specifically the ICA, traverses through the cavernous sinus and provides origin to the ophthalmic artery. As with the middle meningeal artery, the carotid artery would not offer a route of communication between the area of infection and the cavernous sinus

A 16-year-old boy is admitted to the hospital with fever, a confused mental state, and drowsiness. During physical examination it is noted that the boy suffers from severe acne. Radiologic examination reveals cavernous sinus thrombosis. Which of the following routes of entry to the cavernous sinus would most likely be responsible for the infection and thrombosis? A. Carotid artery B. Mastoid emissary vein C. Middle meningeal artery D. Ophthalmic vein E. Parietal emissary vein

C. The CT scan shows an orbital (blow-out) fracture. The inferior rectus muscle originates from the inferior part of the common tendinous ring and inserts on the inferior anterior part of the eyeball. With an orbital fracture, an injury to the floor of the orbit results in entrapment of the inferior rectus muscle by a fragment of bone. This tethers the inferior rectus muscle to bone resulting in loss of function of the muscle. Patients experience diplopia when attempting upward gaze.

A 16-year-old boy presents to the emergency department with pain and blurred vision following a blow to his left eye with a baseball bat. On examination, when asked to gaze upwards his eyes move as seen in the photograph (Fig. 7-27B). Imaging studies reveal a fracture, as indicated by the arrow in the image (Fig. 7-27A). Which of the following muscles is most likely affected? A. Levator palpebrae superioris B. Inferior oblique C. Inferior rectus D. Medial rectus E. Superior rectus

A. With all of the ventricles enlarged and no obvious single site of complete ventricular obstruction, the problem must be a condition of communicating hydrocephalus, with inadequate drainage through the arachnoid granulations into the superior sagittal sinus. There is no evidence of obstruction of CSF flow somewhere in the ventricular system. The other choices listed are all examples of noncommunicating hydrocephalus that result from obstruction, not just overproduction or filtration problems

A 2-month-old female infant is hospitalized with hydrocephalus. MRI reveals a ventricular system that is entirely dilated. Which of the following conditions will most likely lead to this type of clinical picture? A. Lack of absorption through arachnoid granulations into venous system B. Occlusion of cerebral aqueduct (of Sylvius) C. Blockage of the left foramina of Luschka D. Congenital absence of the cisterna magna E. Closure of the interventricular foramina of Monro

A. The inferior rectus and inferior oblique muscles are entrapped in the fissure between the parts of the fractured orbital floor. Normally, the superior rectus and the inferior oblique are responsible for an upward movement of the eyeball. In this case, however, the broken orbital plate of the maxilla has snared or entrapped the inferior rectus and inferior oblique muscles, causing them to act as anchors on the eyeball, preventing upward movement of the eye. The muscles are not necessarily damaged and there is no apparent nerve injury in this patient. Freeing the muscles from the bone will allow free movement of the eye again, barring any other injury. Damage to the medial and inferior recti would result in a laterally and superiorly deviated eye. The inferior oblique rotates the eye upward and laterally. Damage to this muscle would therefore cause the pupil to be directed somewhat downward. Damage to the medial rectus would result in lateral deviation of the eyeball. The inferior rectus is responsible for downward movement of the eye, and damage to this muscle would result in a superiorly deviated eyeball or an inability to gaze upward symmetrically with both eyes.

A 16-year-old female volleyball player is admitted to the hospital after being hit in the eye with a ball spiked at the net. Radiographic examination reveals a blow-out fracture of the inferior wall of the orbit. Physical examination also reveals that the pupil of her eye cannot be turned upward. Which of the following muscle(s) is (are) most likely injured? A. Inferior rectus and inferior oblique B. Medial and inferior recti C. Inferior oblique D. Medial rectus, inferior rectus, and inferior oblique E. Inferior rectus

C. The ophthalmic branch of the trigeminal nerve (CN V1) supplies sensory innervation to the eyeball, leading to pain when damaged. Pain in the hard palate and lower eyelid and anesthesia of the upper lip would be carried by the maxillary branch of the trigeminal nerve (CN V2). Paraesthesia over the buccal portion of the face would be mediated by the maxillary division of the trigeminal nerve.

A 17-year-old girl is admitted to the hospital with signs of cavernous sinus thrombosis, as revealed by radiographic and physical examinations. Thrombophlebitis in the "danger area" of the face can spread to the cavernous sinus and involve the ophthalmic branch of the trigeminal nerve. Which of the following symptoms will most likely be present during physical examination? A. Pain in the hard palate B. Anesthesia of the upper lip C. Pain from the eyeball D. Pain over the lower eyelid E. Tingling sensation over the buccal region of the face

D. The glossopharyngeal nerve mediates general somatic sensation from the pharynx, the auditory tube, and from the middle ear. Painful sensations from the pharynx, including the auditory tube, can be referred to the ear by this nerve, as in this case of tonsillectomy. The auriculotemporal nerve supplies skin of the auricle and tympanic membrane and scalp. This nerve would not be involved directly or indirectly in the operation. The lesser petrosal nerve contains preganglionic parasympathetic fibers that run in the glossopharyngeal and tympanic nerves before synapsing in the otic ganglion. The vagus nerve mediates general somatic afferent supply to the auricle and external acoustic meatus; stimulation of the meatus can trigger a gag reflex or coughing reflex. The chorda tympani mediates taste for the anterior two thirds of the tongue.

A 17-year-old girl is admitted to the hospital with tonsillitis. A tonsillectomy is performed and the patient complains postoperatively of ear pain. Which of the following nerves was most likely injured during the surgical procedure? A. Auriculotemporal B. Lesser petrosal C. Vagus D. Glossopharyngeal E. Chorda tympani

A. Infection in the "danger area of the face" can lead to cavernous sinus thrombosis because infection spreads from the nasal venous tributary to the angular vein, then to the superior ophthalmic vein, which passes into the cavernous sinus. None of the other routes listed would be correct for drainage from the danger area of the face.

A 17-year-old girl visits a dermatologist because of severe facial acne. During physical examination a rather obvious and painful lesion on the side of her nose was found. The patient was given antibiotics and warned not to press or pick at the large, inflamed swelling. If she were to squeeze, prick, or incise such a lesion in the area between the eye and the upper lip, or between the eye and the side of the nose, the infection could spread to the cavernous sinus. Which of the following pathways of spread of infection would be most typical? A. Nasal venous tributary to angular vein, to superior ophthalmic vein, then to cavernous sinus B. Retromandibular vein to supraorbital vein, then to inferior ophthalmic vein, then to cavernous sinus C. Dorsal nasal vein to superior petrosal vein, then inferior ophthalmic vein to cavernous sinus D. Facial vein to maxillary vein, then middle meningeal vein to cavernous sinus E. Transverse facial vein to superficial temporal vein to emissary vein to cavernous sinus

A. At the point where the facial nerve exits the stylomastoid foramen it is most susceptible to shearing forces. In the absence of a skull fracture whereby the facial nerve can be damaged within the facial canal, the nerve is most commonly injured as it exits the stylomastoid foramen. In infants, in whom the mastoid process has not yet developed, the facial nerve lies unprotected, just beneath the skin.

A 2-month-old male infant is admitted to the hospital after falling from his stroller. During physical examination the infant shows signs of facial nerve injury. What is the most common place for facial nerve injury in an infant? A. At the stylomastoid foramen B. Posterior to the parotid gland C. Anterior to the parotid gland D. Proximal to the stylomastoid foramen E. Mandibular involvement of zygomatic and buccal branches

B. Scalp lacerations are the most common type of head injury requiring surgical care. These wounds bleed profusely because the arteries entering the periphery of the scalp bleed from both ends owing to abundant anastomoses. The arteries do not retract when lacerated because they are held open by the dense connective tissue in the second layer two of the scalp. The scalp is composed of five layers, the first three of which are connected intimately and move as a unit. Each letter in the word SCALP serves as a mnemonic for one of its five layers, skin, connective tissue, aponeurosis, loose areolar tissue, and pericranium. The loose connective tissue layer (fourth layer) of the scalp is the danger area of the scalp because pus or blood spreads easily in it. Infection in this layer can also pass into the cranial cavity through small emissary veins, which pass through parietal foramina in the calvaria and reach intracranial structures such as the meninges.

A 20-year-old construction worker presented to the emergency department with an occipital scalp laceration. The wound was closed. Three days later he presents at the clinic with a tender erythematous infected wound that has spread anteriorly on the scalp. Between which layers of the scalp has the infection spread? A. The periosteum and bone B. The aponeurosis and the periosteum C. The dense connective tissue and the aponeurosis D. The dense connective tissue and the skin E. The dermis and the epidermis

A. The butterfly-shaped middle cranial fossa has a central part composed of the sella turcica on the body of the sphenoid and large, depressed lateral parts on each side. The temporal bones are situated at the sides and base of the skull and consist of the squama temporalis, mastoid portion, petrous portion, tympanic part, zygomatic process, and styloid process. The occipital bone is situated at the back and lower part of the skull and is pierced by a large oval opening, the foramen magnum. The ethmoid and frontal bones are found in the anterior cranial fossa.

A 20-year-old man fell off a motorcycle and sustained a head injury. A CT scan of his head revealed a fracture of the sella turcica. Which of the following bones was most likely damaged in this patient? A. Sphenoid B. Temporal C. Occipital D. Ethmoid E. Frontal

C. Four nerves participate in providing cutaneous supply to the neck. The nerves are the supraclavicular, great auricular, transverse cervical, and the lesser occipital. The area over the angle of the jaw is innervated by the great auricular nerve. It ascends from spinal segments from C2 and C3 and innervates the skin over the angle of the jaw and posteroinferior to the auricle of the ear. The transverse cervical also originates from the C2-3 spinal segments but passes anteriorly to innervate the anterior and lateral aspects of the neck. The lesser occipital nerve innervates skin in the area of the back of the neck and posterior occiput. The supraclavicular nerves originate from C3-4 and innervate the more inferior aspects of the neck, the upper deltoid region, and skin inferior to the clavicles.

A 20-year-old man is admitted to the emergency department with a stab wound in the superior region of his neck. A radiographic examination reveals that the wound has not affected any major structures. Physical examination reveals that the patient has lost sensation from the skin over the angle of the jaw. Which of the following nerves is most likely injured? A. Supraclavicular B. Transverse cervical C. Great auricular D. Greater occipital E. Lesser occipital

E. The nerves that control lacrimal secretion pass through the superior orbital fissure, which is found in the orbit between the lesser and greater wings of the sphenoid bone. Taste from the anterior two thirds of the tongue and secretions from the submandibular and sublingual glands were likely affected because their nerve signals are carried by the chorda tympani nerve, which joins the facial nerve in the facial canal located in the petrous part of the temporal bone. Movements of the right side of the face are also likely compromised or deficient because these muscles are innervated by branches of the facial nerve, which was likely impinged in the facial canal as a result of the fracture to the temporal bone.

A 20-year-old man presented to the emergency department with complaints of fever, swelling over the right side of the temple of his head, and headache for 5 days. He had a history of trauma 1 week before presentation, when he was struck from his motorcycle by a car and suffered a minimally displaced open fracture of right temporal bone. The wound was irrigated and debrided and after 24 hours of neurologic observation in hospital, he was discharged. On examination, he was in painful distress. There was a tender swelling over the right temporal bone where sutured skin appeared flushed (erythematous). Neurologic examination produced findings consistent with neuritis (irritation) of the nerve passing through the distal part of the facial canal. Which of the following functions is likely to be spared by this lesion? A. Taste from the anterior two thirds of the tongue B. Movements of the right side of the face C. Secretions from the submandibular gland D. Secretions from the sublingual gland E. Secretions from the lacrimal gland

B. Coursing through the pterygoid canal are the artery, vein, and nerve of the pterygoid canal (Vidian canal). The Vidian nerve (nerve of the pterygoid canal) contains presynaptic parasympathetic fibers from the facial nerve via the greater petrosal nerve, which eventually go on to synapse in the pterygopalatine ganglion and postsynaptic sympathetic fibers from the deep petrosal nerve, which do not synapse in pterygopalatine ganglion

A 20-year-old man was brought to the emergency department after the motorcycle he was driving was struck by a car. He was unable to recall all events of the incident and complained of pain to right side of the head and face, lower back, right elbow, and right knee. On examination, he was alert and oriented. There were multiple abrasions to his upper and lower limbs but no neurologic deficits were evident. There was a tender 6 × 6 cm swelling over the right temporal bone and a laceration to the superior aspect of the helix of the right ear. A CT scan of the head and brain was done, revealing a minimally displaced fracture of the floor of the middle cranial fossa involving the pterygoid canal. Which of the following describes all fiber types that are most likely affected in this injury? A. Presynaptic parasympathetic B. Postsynaptic sympathetic; presynaptic parasympathetic C. Postsynaptic sympathetic; postsynaptic parasympathetic D. Special sensory; postsynaptic parasympathetic E. General sensory; postsynaptic parasympathetic

C. The muscles of facial expression are supplied by the facial nerve, which emerges from the stylomastoid foramen and passes through the parotid gland. The nerve gives off five major branches within the parotid gland from superior to inferior: temporal, zygomatic, buccal, marginal mandibular, and cervical. A fun mnemonic for these five branches is To Zanzibar By Motor Car. (This not only rhymes, but it's a geographical joke. You can't get to Zanzibar by motor car!). The lateral pterygoid and temporalis muscles are muscles of mastication and are supplied by the mandibular division of the trigeminal nerve (CN V3). The common carotid artery provides branches to supply the neck and the face while the vertebral artery supplies the spinal cord and the posterior part of the brain.

A 20-year-old man was in a bar fight and sustained a deep laceration to his face after being slashed with a broken glass bottle. Physical examination revealed a deep, 10-cm laceration running obliquely across his right cheek. He is now unable to close his right eye and cannot smile on the right side. Which additional structure was most likely damaged in this patient? A. Vertebral artery B. Common carotid artery C. Parotid gland D. Lateral pterygoid muscle E. Temporalis muscle

E. Treacher Collins syndrome (craniofacial dyostosis) is an autosomal disorder characterized by malar hypoplasia, underdeveloped mandible, downward slanting palpebral fissures, defects of the lower eyelids and external ears, and abnormalities of the middle and internal ears. This is a first arch syndrome due to failure of neural crest cells to migrate into the first arch during the fourth week of development.

A 20-year-old woman delivered an infant boy at term after an uneventful pregnancy. On examination of the infant, the intern noted that the corners of the eyes were downward slanting and the pinnae and the mandible were underdeveloped. The mother of the infant stated that he had difficulty with breastfeeding and feeding by bottle because he was unable to create the suction needed to feed, and the milk often refluxed through his nose. She also stated that the father of the child had similar facial abnormalities. Which of the following is the most likely cause of the deformities seen in this infant? A. Failed fusion of lateral nasal prominences and maxillary processes B. Lack of fusion of medial nasal prominences C. Failed migration of neural crest cells to the first pharyngeal arch D. Failed migration of neural crest cells to the third and fourth pharyngeal pouches E. Failed migration of ectoderm between maxillary and lateral nasal prominences

C. Hydrocephalus is an abnormal accumulation of cerebrospinal fluid in the ventricles of the brain. This can be due to abnormal flow (often blockage in the cerebral aqueduct of Sylvius), impaired reabsorption, or rarely, excessive production of CSF. Communicating hydrocephalus is caused by impaired reabsorption of CSF due to abnormal functioning of the arachnoid granulations, which are responsible for drainage of the CSF into the venous system. In this condition, all the ventricles of the brain become dilated. Incidentally, although the fact that this baby was delivered by cesarean section is in no way responsible for the hydrocephalus, the term "cesarean section" is interesting. Some people think the term is applied because Julius Caesar was delivered through an incision after his mother died during labor. However, that story is probably apocryphal. A more likely explanation is that when Julius Caesar was emperor of the Roman Empire he issued a series of rules called the Cesarean codes. One of these rules required somebody attending to a woman in labor (probably equivalent to a midwife) to attempt delivery of a viable infant immediately if the mother died during childbirth.

A 20-year-old woman delivered an infant boy at term by cesarean section (C-section). During the third trimester an abdominal ultrasound revealed enlarged ventricles of the fetal brain. On examination, the circumference of the infant's head was 40 cm (normal range, 34 to 36 cm). He showed decreased movement of the limbs and seemed drowsy. A CT scan of the brain with contrast medium showed dilatation of all the ventricles. Which of the following is likely to be the cause of the infant's condition A. Narrowing of the median aperture B. Narrowing of the lateral aperture C. Abnormality of arachnoid granulations D. Lack of choroid plexus in the lateral ventricles E. Increased blood flow in the cerebral arteries

D. Oblique facial clefts are rare and often bilateral facial anomalies that extend from the upper lip to the medial margin of the orbit. In this condition, the nasolacrimal ducts are open grooves draining the tears from the lacrimal gland that lubricate the conjunctiva. This results from failure of the fusion of the mesenchyme of the maxillary process with the lateral nasal process. Failure of fusion of the medial nasal prominences results in a median cleft lip and palate, while failure of fusion of the intermaxillary segment with the maxillary processes results in a unilateral cleft lip

A 20-year-old woman delivered an infant girl at term after an uneventful pregnancy. She admitted to the use of alcohol and tobacco throughout the pregnancy. Upon examination of the infant, the intern discovered that there was a deformity of the face in which a groove connected the medial canthus of the right eye to the right corner of the upper lip. There was a clear, watery discharge from the groove. A defect in which of the following developmental processes most likely resulted in the deformity seen in this infant? A. Fusion of maxillary processes to each other B. Fusion of medial nasal prominences to each other C. Migration of ectoderm between maxillary and lateral nasal prominences D. Fusion of the maxillary and lateral nasal processes E. Fusion of the intermaxillary segment with the maxillary processes

C. The infant's condition is known as coloboma and results from failure of the choroid fissure to fuse. Failure of the neuroectoderm to induce the surface ectoderm to differentiate results in failure of the eye to form. Failed obliteration of the intraretinal space results in retinal separation between the pigmented and neural layers. If the optic cup does not overlap the developing lens, the iris will fail to form entirely. If the lens vesicle does not separate from the surface ectoderm, the eye cavities will not form.

A 20-year-old woman delivered an infant girl at term after an uneventful pregnancy. Upon examination of the infant, the intern noticed that the iris of the left eye had a defect inferolaterally, so the pupil appeared to have an extended opening. The mother of the infant was reassured that this would not likely cause any significant visual impairment, as the defect was very small. Which of the following explains the defect noticed in the infant's eye? A. Failed induction of surface ectoderm by the neuroectoderm B. Failed obliteration of the intraretinal space C. Failed closure of the choroid fissure D. Optic cup does not overlap the developing lens E. Lens vesicle remains connected to surface ectoderm

D. The fracture to the zygomatic bone, due to proximity, likely distorted the orbit resulting in damage to the structures running within the superior orbital fissure, where the nerves that transmit sensations from the lacrimal gland pass. It is divided into three parts, arranged from lateral to medial: the lateral part transmits the lacrimal frontal and trochlear nerves and the meningeal branch of the lacrimal artery. The lacrimal nerve, being the structure closest to the fracture, will likely be the first to be injured.

A 20-year-old woman presented to the emergency department after being thrown from her horse during an equestrian competition. She denied loss of consciousness but complained of pain on the left side of her face, hand, and elbow, as well as double vision. On examination, there was a tender swelling over the superolateral aspect of the left eye, and a defect in the bone was palpable. There was significant periorbital edema and bruising of the inferior skin of the eye. Pupillary light reflexes were normal, and funduscopy showed no abnormalities of the retina. A CT scan of the head and face revealed a comminuted, depressed fracture of the frontal process of the left zygomatic bone involving the orbit. Which of the following describes the most likely consequence of her injury? A. Weak abduction of the eye B. Diplopia C. Blurred vision D. Decreased secretions from the left lacrimal gland E. Partial ptosis

D. A torn cerebral vein often results in a relatively slow-bleeding subdural hematoma. Such a hematoma can be involved in gradual compression of the brain, resulting in confusion, dizziness, clumsiness, and memory loss. There would be no sign of blood in the CSF because the bleeding is into the subdural space, not the subarachnoid space. This would fit the description of symptoms in this case. Middle meningeal artery rupture results in an epidural hematoma, which is much more acute and often includes a brief period of unconsciousness followed by a lucid interval and can proceed to death if the bleeding is left untreated. Fracture of the pterion also can result in an epidural hematoma because the middle meningeal artery is the adjacent vasculature mentioned. Rupture of the anterior communicating artery would result in a subarachnoid hematoma, and there would be blood in the CSF upon lumbar puncture. In a cavernous sinus thrombosis there would be cranial nerve involvement due to compression of those nerves that run through or near the cavernous sinus, including the oculomotor, trochlear, trigeminal (maxillary and mandibular divisions), and abducens nerves.

A 21-year-old male baseball player is brought to the emergency department after having severe dizziness. During physical examination the patient demonstrates lack of equilibrium and memory impairment. A 3-cm wound is noted in his scalp from an injury suffered in a game several weeks earlier. A lumbar puncture does not reveal blood in the CSF. Which of the following is the most likely diagnosis? A. The middle meningeal artery was torn, resulting in epidural hematoma B. There is a fracture in the pterion with injury to the adjacent vasculature C. The injury resulted in the bursting of a preexisting aneurysm of the anterior communicating artery of the cerebral circle D. A cerebral vein is torn E. The cavernous sinus has a thrombus

C. Ophthalmic nerve (ophthalmic division of the trigeminal nerve, CN V1) carries sensory branches from the eye, conjunctiva, and orbital contents, including the lacrimal gland. It also receives sensory branch from the nasal cavity, frontal sinus, ethmoidal cells, falx cerebri, dura in the anterior cranial fossa and superior part of the tentorium cerebelli, upper lid, dorsum of the nose, and anterior part of the scalp. Branches of ophthalmic nerve are lacrimal, supraorbital, supratrochlear, infratrochlear, and external nasal nerves. Maxillary nerve (CN V2) and mandibular nerve (CN V3) carry sensory information from other regions of the face.

A 21-year-old man has suspected intracranial inflammation from an infection. A CT scan showed an abscess compressing the ophthalmic branch of the trigeminal nerve. Which of the following would be a plausible complaint of this individual? A. Pain in the hard palate B. Anesthesia of the upper lip C. Painful eyeball D. Pain over the lower eyelid E. Tingling sensation over the buccal region of the face

E. The loose connective tissue layer (layer four) of the scalp is the danger area of the scalp because purulence or blood spreads easily in it. Infection in this layer can also pass into the cranial cavity through small emissary veins, which pass through foramina in the calvaria, and reach intracranial structures such as the meninges. The diploic veins are found in the skull and drain the diploic space. This is found in the bones of the vault of the skull and is the marrow-containing area of cancellous bone between the inner and outer layers of compact bone

A 21-year-old man shaved his head with a razor blade and sustained a deep cut to the scalp. An infection occurred at the site of the cut. The man did not seek medical attention and the infection eventually subsided. Three weeks later he woke up with a severe headache and nausea. He was brought to the emergency department and found to have an infection of the superior sagittal sinus. Which of the following veins was most likely responsible for this patient's current infection? A. Intercavernous B. Inferior sagittal C. Diploic D. Basilar E. Emissary

D. Kiesselbach's plexus (also called Little's area) is an anastomosis of four arteries on the anterior nasal septum. The four arteries are the anterior ethmoidal artery, sphenopalatine artery, superior labial artery, and greater palatine artery. The two largest contributors, however, are the septal branches of the sphenopalatine (from the maxillary artery) and superior labial arteries (branches of the facial artery, which in turn is a branch of the external carotid artery).

A 21-year-old man was brought to the emergency department because of severe epistaxis (nosebleed) from the nasal septum. This area, known as Kiesselbach's (or Little's) area, involves mostly anastomoses between which of the following arteries? A. Ascending palatine and ascending pharyngeal B. Posterior superior alveolar and accessory meningeal C. Lateral branches of posterior ethmoidal and middle meningeal D. Septal branches of the sphenopalatine and superior labial E. Descending palatine and tonsillar branches of the pharyngeal

B. The optic and oculomotor nerves are responsible for the sensory and motor portions, respectively, of the pupillary light reflex. The optic nerve would include the sensory portion, but the facial nerve only closes the eyelid and does not affect the pupil. The maxillary division of the trigeminal nerve only provides sensory innervation to the skin surrounding the maxillary bone. The ophthalmic division of the trigeminal nerve provides sensory innervation to the cornea for the corneal reflex, but not the light reflex.

A 33-year-old woman is unconscious when she is admitted to the hospital after she fell, hitting her head. The physician in the emergency department performs a pupillary light reflex test. The integrity of which of the following nerves is being checked? A. Optic and facial B. Optic and oculomotor C. Maxillary and facial D. Ophthalmic and oculomotor E. Ophthalmic and facial

D. The short ciliary nerves and ciliary ganglion is the correct answer option. It is clear from the vignette that the problem lies with accommodation, which involves pupil accommodation, lens accommodation, and convergence. Eye movements were not compromised and as such parasympathetic deficit is the likely cause of his condition. Preganglionic axons from the Edinger-Westphal nucleus are carried by the oculomotor nerve, which synapse at the ciliary ganglion. These parasympathetics then travel through the ciliary ganglion located in the posterior orbit and from here postganglionic axons, carried by the short ciliary nerves, go on to innervate the sphincter pupillae muscle, which constricts the pupil and the ciliary muscle. Contraction of the ciliary muscle relieves tension on the zonular fibers, allowing the lens to be more convex. Absence of action of the ciliary muscles and sphincter pupillae results in loss of the ability to focus on near objects. The trochlear nerve and abducens nerve is an incorrect option. These nerves innervate the superior oblique and lateral rectus muscles, respectively. Since eye movements were unaffected, this combination can be eliminated as an answer option immediately

A 22-year-old baseball player is hit in the eye by a baseball. On examination he was able to move his eye normally and see distant objects but was unable to focus on close objects. Injury to which of the following nerve structures would most likely be the cause of his condition? A. Trochlear nerve and abducens nerve B. Oculomotor nerve and abducens nerve C. Superior cervical ganglion and long ciliary nerves D. Short ciliary nerves and ciliary ganglion E. Infratrochlear nerve and ciliary ganglion

A. Anesthetics are injected into the submuscular layer of delicate (areolar) connective tissue, the layer that contains nerves of the eyelid. This space is continuous with the "danger zone" of the scalp. A blow to the forehead can result in a hematoma known as a "black eye," with the passage of blood into the submuscular space. Infections can, likewise, pass within this space. One can insert a needle through the upper eyelid, near the orbital margin, and then direct it deeply toward the orbital apex. The anesthetic can there infiltrate the branches of the ophthalmic division of the trigeminal nerve, including its nasociliary branch, resulting in anesthesia of the area.

A 22-year-old man is admitted to the emergency department after he was injured in a street fight (Fig. 7-6). Radiographic examination reveals that he has suffered a forehead fracture, resulting in black and swollen eyes. Because the patient is in severe pain, an anesthetic solution is injected into his orbit. Which of the following nerves is most likely to be anesthetized? A. Ophthalmic B. Infraorbital C. Anterior ethmoidal D. Frontal E. Optic

E. The rima glottidis is the opening between the vocal folds and the arytenoid cartilages on the right and left sides. The piriform recess is the recess lateral to the laryngeal opening of the laryngopharynx. The vestibule is the region between the epiglottis and rima glottidis. The ventricle is the area between the true and false vocal cords. The vallecula is a bilateral recess anterior to the epiglottis just posterior to the tongue in the laryngopharynx.

A 22-year-old man is admitted to the emergency department and intubated. An endotracheal tube is passed through an opening between the vocal folds. What is the name of this opening? A. Piriform recess B. Vestibule C. Ventricle D. Vallecula E. Rima glottidis

A. The lingual nerve is joined by the chorda tympani about 2 cm below the base of the skull, deep to the lower border of the lateral pterygoid muscle. It supplies the anterior two thirds of the tongue with common sensation and taste, the latter mediated by fibers in the chorda tympani. The secretomotor fibers of the chorda tympani are given off to the submandibular ganglion, which is suspended from the lingual nerve. They relay in the ganglion for the submandibular gland, and some postganglionic fibers rejoin the lingual nerve for transport to the salivary glands in the floor of the mouth. The lingual nerve also supplies all the mucous membrane of the floor of the mouth and the lingual gingiva (gums).

A 26-year-old man suffers a traumatic injury to the midface region. Physical examination reveals intact taste and salivation but no general sensation from his anterior tongue. Which nerve was probably injured? A. Lingual proximal to its junction with the chorda tympani B. Chorda tympani C. Inferior alveolar D. Lingual distal to its junction with the chorda tympani E. Glossopharyngeal

A. The ophthalmic artery is a branch of the internal carotid artery and provides origin to the ocular and orbital vessels, including the central artery of the retina, which supplies the retina. The central artery of the retina is an end artery that has no anastomoses with other arterial sources; therefore, occlusion of this artery will result in loss of vision. The nasociliary nerve is a branch of the ophthalmic division of the trigeminal nerve. It is the general sensory nerve for the eye and is the afferent limb of the corneal blink reflex; it has no direct effect on vision. The anterior ethmoidal nerve is a branch of the nasociliary nerve and supplies the anterior ethmoid air cells, the nasal septum, and the lateral walls of the nasal cavity; it also supplies the skin on the bridge of the nose. The trochlear nerve is the fourth of the 12 cranial nerves and innervates the superior oblique muscle, one of the six extraocular muscles. The extraocular muscles function in the movement of the eyeball and not the perception of light. The optic nerve is the second of the 12 cranial nerves and is responsible for vision. A lesion of the optic nerve would lead to blindness; however, based on the location of the patient's infection, the optic nerve was not affected by the loss of arterial supply

A 22-year-old man is admitted to the emergency department with a sinus infection. Radiographic examination reveals posterior ethmoidal cell infection. During history taking the patient complains of progressive loss of vision. Which of the following structures is most likely affected? A. Ophthalmic artery B. Nasociliary nerve C. Anterior ethmoidal nerve D. Trochlear nerve E. Abducens nerve

B. A fracture of the lamina papyracea of the ethmoid bone is likely to entrap the medial rectus muscle, causing an inability to gaze laterally. A fracture of the orbital plate of the frontal bone could perhaps entrap the superior oblique or superior rectus muscle, but this would be very unusual. A fracture of the orbital plate of the maxilla can entrap the inferior rectus or inferior oblique muscles, limiting upward gaze. A fracture of the cribriform plate could damage olfactory nerves and result in leakage of CSF through the nose (CSF rhinorrhea), with associated meningeal infection. A fracture of the greater wing of the sphenoid is not likely to entrap any extraocular muscles.

A 22-year-old man is admitted to the hospital after he was hit in the right eye with a frozen fish, thrown playfully by a friend while they were passing through the seafood section of the market. During physical examination considerable swelling and discoloration of the eyelids was observed. In addition, the patient could not turn his pupil laterally from forward gaze, indicating probable muscle entrapment. Which of the following bones was most likely fractured? A. Orbital plate of the frontal bone B. Lamina papyracea of the ethmoid bone C. Orbital plate of the maxilla D. Cribriform plate of the ethmoid bone E. Greater wing of the sphenoid bone

B. The sensory supply of the mucous membrane of the oral part (anterior two thirds), but not the region of the vallate papillae, is by the lingual nerve. General sensation is via the mandibular division of the trigeminal nerve (with cell bodies in the trigeminal ganglion) and taste via chorda tympani, from the facial nerve, which have their cell bodies in the geniculate ganglion. The submandibular and pterygopalatine ganglia contain parasympathetic fibers from chorda tympani and greater petrosal nerves, respectively.

A 22-year-old man presents with severe facial trauma after a car crash. A CT scan of the head reveals multiple fractures of the facial bones. Following surgery to correct his facial fractures, he asserts that he cannot taste anything on the tip of his tongue. Which ganglion contains the neuron cell bodies of taste fibers from this part of the tongue? A. Trigeminal B. Geniculate C. Inferior glossopharyngeal D. Submandibular ganglion E. Pterygopalatine

C. An incision at the level of the third and fourth tracheal cartilages usually results in the fewest complications during a tracheostomy (or tracheotomy). The isthmus of the thyroid gland (a richly vascular structure) is usually at the level of the second tracheal cartilage and this incision is just inferior to that. However, other vascular structures such as a thyroidea ima artery or tributaries of the external jugular veins make a tracheostomy a surgical procedure to be performed with care.

A 22-year-old woman is admitted to the emergency department unconscious after falling over the handlebars of her bicycle. An emergency tracheotomy is performed to insert a tracheotomy tube. What is the most common tracheal cartilage level at which a tracheotomy incision is performed? A. First to second B. Second to third C. Third to fourth D. Fourth to fifth E. Fifth to sixth

C. The afferent/sensory limb of the corneal (blink) reflex is carried by the nasociliary nerve. It is a branch of the ophthalmic division of the trigeminal nerve. The frontal and lacrimal nerves provide cutaneous supply to parts of the orbit and face, but they do not innervate the cornea. The facial nerve is the efferent limb of the corneal reflex and mediates the closing of both eyes in response to irritation of the cornea. The oculomotor nerve mediates the reopening of the eyes by contraction of the levator palpebrae superioris. The optic nerve also innervates the eye for the sense of vision and is the afferent limb of the pupillary light reflex.

A 22-year-old woman is admitted to the hospital with an injury to her eye. The corneal reflex is tested and found to be present. Which of the following nerves is responsible for the afferent limb of this reflex? A. Frontal B. Lacrimal C. Nasociliary D. Oculomotor E. Optic

E. The nasolacrimal duct is the only duct that normally drains into the inferior meatus of the nose and therefore would be affected by a focal inflammation in this region.

A 22-year-old woman visits the outpatient clinic with a sinus infection of 2 weeks duration. Physical examination reveals that the patient has focal inflammation, with mucosal edema in the inferior nasal meatus. Drainage from which of the following structures is most likely to be obstructed by this inflammation and edema? A. Anterior ethmoidal air cells B. Frontonasal duct C. Maxillary sinus D. Middle ethmoidal air cells E. Nasolacrimal duct

B. The superior cerebellar artery is the penultimate branch of the basilar artery. It originates from the basilar artery and runs laterally to supply the superior and medial parts of the cerebellum. In its course, it travels very close to the trochlear nerve. This nerve innervates the superior oblique muscle, which is responsible for depression of the eyeball when it is in the adducted state. This allows for proper vision when reading a newspaper or walking down stairs

A 34-year-old man complains of diplopia that becomes worse when reading a newspaper or walking down the stairs. Physical examination in the neurosurgery outpatient clinic reveals weakness of downward movement of the left eye. A CT scan reveals a large aneurysm in one of the arteries of the posterior cerebral circulation compressing the nerve resulting in the symptoms. In which artery in the diagram (Fig. 7-23) is the aneurysm? A. A B. B C. C D. D E. E

D. The superior thoracic aperture is also referred to as the thoracic inlet because noncirculating substances (air and food) may only enter the thorax and not exit. Obstruction actually occurs outside the aperture in the root of the neck, and the manifestations of the syndromes involve the upper limb through this aperture. A supernumerary (extra cervical) rib or a fibrous connection extending from its tip to the first thoracic rib may elevate and place pressure on structures that emerge from the superior thoracic aperture, notably the subclavian artery or inferior trunk of the brachial plexus, and may cause symptoms. Other etiologies could include an elongated transverse process of the C7 vertebra and muscular abnormalities (e.g., in the scalenus anterior muscle, a sickle-shaped scalenus medius). Most common symptoms include discoloration of the hands, one hand colder than the other hand, weakness of the hand and arm muscles, and tingling

A 23-year-old right-handed pitcher for a Major League Baseball team presents to the clinic for evaluation concerning a constellation of symptoms. A CT scan demonstrates hypertrophied scalene muscles on the right side. The diagnosis is thoracic outlet syndrome. Which symptom is consistent with this diagnosis? A. Problems with respiration because of pressure on the phrenic nerve B. Reduced blood flow to the thoracic wall C. Reduced venous return from the head and neck D. Numbness in the upper extremity E. Distension of the internal jugular vein

A. The oculomotor nerve (CN III) is the efferent limb of the pupillary reflex. It carries the parasympathetic nerve with preganglionic cell bodies in the Edinger-Westphal nucleus and postganglionic cell bodies in the ciliary ganglion and supplies the sphincter pupillae of the iris. The oculomotor nerve is found between the superior cerebellar artery and the posterior cerebral artery. Aneurysm of the superior cerebellar artery can therefore compress on the oculomotor nerve resulting in pupillary dilatation (due to unopposed sympathetic innervation of the dilator pupillae) and unresponsiveness of the affected eye. The trigeminal, facial, vagus, and abducens nerves do not participate in the pupillary light reflex.

A 27-year-old unconscious woman is admitted to the emergency department. Examination reveals the right pupil is dilated and unresponsive to light. A CT scan reveals a large aneurysm of the superior cerebellar artery just after it branches off from the basilar, compressing the nerve responsible for the symptoms. Which nerve is being compressed? A. Oculomotor B. Trigeminal C. Facial D. Vagus E. Abducens

C. The lingual nerve is the most likely nerve damaged because there is loss both of taste and general sensory supply to the anterior two thirds of the tongue, which is innervated by the lingual nerve, which at this point has been joined by the chorda tympani. The chorda tympani would be a likely choice; however, it carries only taste and does not mediate other general sensation to the tongue. The auriculotemporal nerve is a posterior branch of the mandibular division of the trigeminal nerve and innervates skin near the ear and temporal region. The mental nerve is the terminal branch of the inferior alveolar nerve and innervates the skin of the chin.

A 24-year-old man had a third molar (wisdom tooth) extracted from his lower jaw. This resulted in the loss of general sense and taste sensation from the anterior two thirds of the tongue. This loss was most likely due to injury of which of the following nerves? A. Auriculotemporal B. Chorda tympani C. Lingual D. Mental E. Inferior alveolar

C. The maxillary sinus is located directly inferior to the orbit. Any trauma to the inferior bony wall of the orbit will likely displace the orbital structures in the compartment to the space below the orbit (maxillary sinus). In most people the sclera of the eyeball is stronger than in the orbital floor, so a blow-out fracture is more likely than rupture of the eyeball. The ethmoidal sinus is located superiorly and medially to the orbit, whereas the frontal sinus is located superiorly to the orbit. The nasal cavity is toward the midline and is not inferior to the orbit. The sphenoidal sinus is deeper into the facial region but is not inferior to the orbit.

A 24-year-old man is admitted to the hospital after a street fight. Radiographic examination reveals an inferior (blow-out) fracture of the orbit. Orbital structures would most likely be found inferiorly in which of the following spaces? A. Ethmoidal sinus B. Frontal sinus C. Maxillary sinus D. Nasal cavity E. Sphenoidal sinus

A. An inferior fracture of the orbit would likely damage the infraorbital nerve. A blow-out fracture often results in a displaced orbital wall, and in this case, the inferior wall. The infraorbital nerve leaves the skull immediately inferior to the inferior aspect of the orbit, via the infraorbital foramen. Thus, this nerve is the most likely to be damaged. The frontal nerve courses superiorly over the orbital contents before dividing into the supratrochlear and supraorbital nerves. The optic nerve is located behind the eyeball and travels posteriorly away from the orbit to enter the cranium. These nerves are therefore unlikely to be damaged.

A 24-year-old man is admitted to the hospital after a street fight. Radiographic examination reveals an inferior blow-out fracture of the orbit. Which of the following nerves is particularly vulnerable with this type of injury? A. Infraorbital B. Supratrochlear C. Frontal D. Inferior alveolar E. Optic

D. The area behind the foramen magnum consists of the squamous part of the occipital bone. The foramen magnum is in the basilar part of the occipital bone (basiocciput). The dura mater is attached to the margins of the foramen as it sweeps down from the posterior cranial fossa. Within the tube of dura mater, the lower medulla with the vertebral and spinal arteries and the spinal roots of the accessory nerves traverse the foramen in the subarachnoid space. CN 1 passes through the cribriform plate of the ethmoid bone. The glossopharyngeal, vagus, and accessory nerves arise from the side of the medulla oblongata. The three nerves run laterally across the occipital bone and pass through the jugular foramen

A 25-year-old man sustains a blow to the back of the head during a fight and is brought to the emergency department unconscious. A CT scan of the head reveals a fracture in the occipital bone extending superiorly from the foramen magnum. Which of the following is transmitted through the foramen magnum? A. Cranial nerve I B. Cranial nerve IX C. Cranial nerve X D. Cranial nerve XI E. Cranial nerve XII

B. Both the stapedius and tensor tympani normally function to dampen movements of the middle ear ossicles, thereby muting sound and preventing hyperacusis. A stapedius would be the source of hyperacusis in this problem because it receives its innervation from the facial nerve. The tensor tympani receives motor innervation from the mandibular division of the trigeminal nerve. The posterior belly of the digastric and the stylohyoid receive innervation from the facial nerve, but their paralysis would not cause hyperacusis. Damaged innervation of the cricothyroid, which is supplied by the external branch of the superior laryngeal nerve, would not result in hyperacusis.

A 27-year-old woman is admitted to the emergency department after she was thrown from a motor scooter. Radiographic evaluation reveals a type I LeFort fracture and comminuted fracture of the mandible and TMJ. Despite reconstructive surgery, the patient develops hyperacusis (sensitivity to loud sounds) due to facial nerve paralysis. Which of the following muscles is most likely paralyzed? A. Posterior belly of digastric B. Stapedius C. Tensor tympani D. Stylohyoid E. Cricothyroid

C. The two nerves found in the internal acoustic meatus are the facial nerve and vestibulocochlear nerve. Of the two, the facial nerve supplies facial muscles and can cause unilateral facial paralysis.

A 27-year-old woman is found to have unilateral facial paralysis. An MRI of her head reveals a small tumor located at the internal acoustic meatus. Which of the following cranial nerves is most likely being compressed by the tumor? A. V B. VI C. VII D. IX E. X

B. The patient described in the question has experienced vasovagal syncope after stimulation of his posterior external auditory canal by an otoscope speculum. In this form of syncope, parasympathetic outflow via the vagus nerve (CN X) leads to decreased heart rate and blood pressure. The posterior part of the external auditory canal is innervated by the small auricular branch of the vagus nerve. Most of the remainder of the external auditory canal, including the external portion of the tympanic membrane, is innervated by the mandibular division of the trigeminal nerve via its auriculotemporal branch. The inner surface of the tympanic membrane is innervated by the glossopharyngeal nerve (CN IX) via its tympanic branch.

A 28-year-old woman visits the ENT physician because of a painful infection of her outer ear. During physical examination the ENT physician inserts a speculum into the external auditory meatus in close contact with its posterior wall. The patient starts coughing and complains of feeling dizzy. Irritation of which of the following nerves was most likely stimulated by the speculum? A. Vestibulocochlear B. Vagus C. Trigeminal D. Facial E. Accessory

C. The recurrent laryngeal nerve supplies most of the motor innervation to the larynx and sensation below the true vocal folds. The thyroid gland and the recurrent laryngeal nerve are in close proximity and thus the nerve is the most likely to be injured with a thyroidectomy. Injury to the recurrent laryngeal nerve can result in speech defects, including hoarseness. The superior laryngeal nerve has two branches: the internal laryngeal nerve innervates the mucous membranes of the larynx above the vocal folds, and the external laryngeal nerve innervates the cricothyroid muscle, which tenses the vocal folds. The glossopharyngeal nerve is located superiorly to the true vocal folds and would not be affected by this procedure. GAS 1021; N 78;

A 29-year-old woman underwent a thyroidectomy. Postoperatively, the patient presented with hoarseness. Which of the following nerves was most likely injured during the operation? A. Internal laryngeal B. External laryngeal C. Recurrent laryngeal D. Superior laryngeal E. Glossopharyngeal

C. The neural cell bodies whose axons synapse in the pterygopalatine ganglion are located in the superior salivatory nucleus, which is in the pons; this nucleus provides the general visceral efferent (GVE) fibers of the facial nerve for lacrimal and salivary secretion. The SCG superior cervical ganglion is a sympathetic ganglion containing postganglionic neurons and is not concerned with the pterygopalatine ganglion, which is a parasympathetic ganglion. The Edinger-Westphal nucleus is located in the midbrain and contains the cell bodies of the GVE fibers of the oculomotor nerve, which are responsible for constriction of the pupil via synapse in the ciliary ganglion and supply to the sphincter pupillae muscle and accommodation via the ciliary muscle. The inferior salivatory nucleus is located in the medulla and gives origin to GVE fibers of the glossopharyngeal nerve to the otic ganglion for secretion of saliva from the parotid gland.

A 3-month-old male infant is admitted to the hospital because he cries continuously. During physical examination it is observed that the infant has a dry right eye. Upon the basis of imaging studies, the neuroophthalmologist diagnoses a lesion at the neural cell bodies of the preganglionic axons of the fibers that terminate in the pterygopalatine ganglion. Which of the following structures contains the neural cell bodies of the preganglionic axons? A. Superior cervical ganglion B. Edinger-Westphal nucleus C. Superior salivatory nucleus D. Inferior salivatory nucleus E. Nucleus ambiguus

B. The parotid gland receives visceromotor postganglionic parasympathetics and somatosensory (to the parotid capsule) from the auriculotemporal nerve. As the nerve passes between the neck of the mandible and the sphenomandibular ligament, it gives off parotid branches and then turns superiorly, posterior to its head and moving anteriorly, gives off anterior branches to the skin around the auricle.

A 3-year-old boy is being evaluated for painful swelling on the right side of his face. Imaging (Fig. 7-21) reveals an obstructed and enlarged structure (arrows) and inflammation of the associated salivary gland. Which of the following nerves is most likely responsible for the pain? A. Facial B. Auriculotemporal C. Lingual D. Vagus E. Chorda tympani

A. Pharyngeal (branchial) cleft cysts are the most common congenital cause of a neck mass. They are epithelial cysts that arise anterior to the superior third of the sternocleidomastoid muscle from a failure of obliteration of the second branchial cleft in embryonic development. The second arch grows caudally and, ultimately, covers the third and fourth arches. The buried clefts become ectoderm-lined cavities that normally involute. Occasionally this process is arrested and the entrapped remnant forms an epithelium-lined cyst, in some cases with a sinus tract to the overlying skin. Many branchial cleft cysts are asymptomatic; others may become tender, enlarged, or inflamed, or they may develop abscesses that rupture, resulting in a purulent draining sinus to the skin or pharynx. Surgery is indicated in these cases.

A 3-year-old boy is brought to the outpatient clinic with swelling of the side of his neck. Physical examination reveals a congenital mass of tissue anterior to the superior third of the sternocleidomastoid muscle (Fig. 7-10). The swelling is asymptomatic, nonpainful, and soft fluctuant. Which of the following is the most likely diagnosis? A. Branchial cleft cyst B. Ruptured sternocleidomastoid muscle C. Lymph node inflammation D. Torticollis E. External carotid artery aneurysm

C. The external surface of the tympanic membrane is innervated primarily by the auriculotemporal nerve, a branch of the mandibular division of the trigeminal nerve. Damage to this nerve would additionally result in painful movements of the TMJ because this joint receives innervation from the same nerve. Taste in the anterior two thirds of the tongue is supplied by the facial nerve and would be unaffected in this injury. (The chorda tympani could be injured, but its superior location on the medial side would make this unlikely.) The sensory innervation of the nasal cavity is supplied by the ophthalmic and maxillary divisions of the trigeminal nerve and would be unaffected by injury to the tympanic membrane. Sensory innervation to the larynx is provided by the vagus nerve, whereas the pharynx receives sensory fibers from the glossopharyngeal and vagus nerves. The palate is supplied by the maxillary divisions of the trigeminal nerve and would be unaffected by this injury.

A 3-year-old girl ruptured her tympanic membrane (eardrum) when she inserted a pencil into her external ear canal. Physical examination revealed pain in her ear and a few drops of blood in the external auditory meatus. There was the concern that there might possibly have been an injury to the nerve that principally innervates the external surface of the tympanic membrane. Which of the following tests is most likely to be performed during physical examination to check for injury to this nerve? A. Check the taste in the anterior two thirds of the tongue B. Check the sensation to the pharynx and palate C. Check if there is paraesthesia at the TMJ D. Check for sensation in the larynx E. Check for sensation in the nasal cavity

C. The signals that regulate these functions are carried by the chordae tympani nerve, which then joins the lingual nerve. As the lingual nerve makes its way to the tongue it passes between the medial pterygoid muscle and the ramus of the mandible, where it was likely affected by the fracture. The elevation of the jaw or mandible is a function performed by the masseter, temporalis, and medial pterygoid muscles. The lateral deviation is performed by the medial and lateral pterygoid muscles.

A 30-year-old man was brought to the emergency department after being involved in a head-on collision with another car. He did not wear a seat belt and was found unconscious in the driver's seat, from which he had to be extricated. Examination revealed a poorly responsive patient with a low BP, tachycardia, and several lacerations about his face and body. Examination revealed a bony deformity of the right mandible where abnormal mobility of the bone was palpated approximately along the ramus. Once the patient was stabilized, a CT of the skull was performed, revealing a displaced, transverse fracture of the ramus, just proximal to the angle of the mandible. Which of the following will most likely be affected? A. Elevation of the jaw B. Lateral deviation of the jaw to the left side C. Salivation; sensation and taste from the anterior tongue D. Sensation from skin over anterior cheek and tongue E. Salivation; sensation from posterior temporal skin and tragus of the ear

A. The nerves that control lacrimal secretion pass through the superior orbital fissure, which is found in the orbit between the lesser and greater wings of the sphenoid. The superior orbital fissure is divided into three parts that are oriented from lateral to medial. The lateral part transmits the lacrimal nerve, which is well away from the fracture site at the apex of the orbit. Fracture at the base of the orbit will more than likely affect structures in the medial and middle parts of the fissure. Through the medial part courses sympathetics from the internal carotid plexus and the superior ophthalmic vein. The abducens nerve, the nasociliary nerve, and divisions of the oculomotor nerve all pass through the middle part of the fissure, which results in options B, C, D, and E being likely consequences of the injury as they are closer to the apex of the orbit

A 30-year-old man was brought to the emergency department after being involved in a head-on collision with another car. He did not wear a seat belt and was found unconscious in the driver's seat, from which he had to be extricated. Examination revealed a poorly responsive patient with low BP, tachycardia, and several lacerations about his face and body. Once the patient was stabilized, a CT of the skull was done, revealing a fracture near the apex of the orbit with narrowing of the opening to the orbit in the area. Which of the following functionswill mostlikely be maintained or spared in this patient? A. Secretions from the right lacrimal gland B. Ability to detect a foreign body on the cornea C. Sensation over the anterior scalp D. Sensation to the upper eyelid E. Ability to abduct the eye

B. The ethmoid sinuses are groups of air cells located in the ethmoid bone. There are three groups of air cells: posterior (drain into the superior meatus), middle (drain into the middle meatus at the ethmoid bullae), and anterior (drain into the middle meatus by way of the ethmoid infundibulum at the semilunar hiatus). This can spread infections from the paranasal sinuses into the nasal cavity, leading to upper respiratory tract infections. The pharyngotympanic tube runs from the middle ear to the nasal cavity. The nasolacrimal duct indirectly drains the lacrimal gland, which is located on the lateral side of the orbit. The sphenoid sinus drains into the sphenoethmoidal recess.

A 30-year-old woman presented to her physician with complaints of nasal congestion and headache for 2 days. Her symptoms were associated with clear fluid from her nostrils (rhinorrhea). Examination revealed that she was febrile (38° C) and the nasal mucosa was erythematous with swelling of the conchae. There was tenderness over the forehead above the root of the nose and the areas on either side of the glabella. She was diagnosed with an upper respiratory tract infection. Which of the following structures is the most likely passage for the spread of infection in this patient? A. Pharyngotympanic tube B. Ethmoidal infundibulum C. Nasolacrimal duct D. Spheno-ethmoidal recess E. Superior nasal meatus

B. The order of tooth eruption is a follows: inferior medial incisors (6 to 8 months), superior medial incisors (8 to 10 months), first molar (6 to 8 months), superior lateral incisors (8 to 10 months), and finally inferior lateral incisors (12 to 14 months). Teeth tend to erupt earlier in girls than in boys, and quite a range exists in the normal distribution curve

A 31-year-old mother visits the pediatric outpatient clinic with her 6-month-old baby girl complaining that her baby is not developing quickly and has no teeth. Which of the following teeth are expected to appear first? A. Superior medial incisors at 8 to 10 months of age B. Inferior medial incisors at 6 to 8 months of age C. Superior lateral incisors at 8 to 10 months of age D. Inferior lateral incisors at 12 to 14 months of age E. First molar at 6 to 8 months of age

C. The superior cervical ganglion (SCG), which is the uppermost part of the sympathetic chain, supplies sympathetic innervation to the head and neck. The usual symptoms for SCG injury, commonly known as Horner's syndrome, are miosis, ptosis, and anhydrosis in the head and neck region. Postganglionic sympathetic nerves usually run alongside the arteries leading into the head and neck region. The submandibular ganglion does not carry sympathetic nerves to areas of the head and neck. The trigeminal ganglion includes only cell bodies from afferent sensory nerves from the head. The geniculate ganglion includes cell bodies for taste sensation from the anterior two thirds of the tongue, carried by the facial nerve; it also transmits parasympathetic innervation to many sections of the head and face. The ciliary ganglion provides parasympathetic innervation to the eye and also has some sympathetic fibers coursing through but not synapsing; thus, it would not account for the symptoms of the face.

A 31-year-old woman is admitted to the hospital after an automobile collision. An MRI examination reveals a large hematoma inferior to the right jugular foramen. Physical examination reveals right pupillary constriction (miosis), ptosis of the eyelid, and anhydrosis (loss of sweating) of the face. Which of the following ganglia is most likely affected by the hematoma? A. Submandibular B. Trigeminal (semilunar or Gasserian) C. Superior cervical D. Geniculate E. Ciliary

C. The cricothyroid artery is a small branch of the superior thyroid artery. It anastomoses with the cricothyroid artery of the opposite side at the upper end of the median cricothyroid ligament, a common site for establishing an emergency airway. The cricothyroid artery can be pushed into the airway during a cricothyroidotomy. The vessel(s) can bleed directly into the trachea, bleeding that can go unnoticed by medical personnel, with potentially fatal aspiration of blood by the patient.

A 32-year-old man is admitted to the emergency department unconscious after a serious car crash. During an emergency cricothyroidotomy an artery is accidentally injured. Two days later the patient shows signs of aspiration pneumonia. Which of the following arteries was most likely injured? A. Superior thyroid B. Inferior thyroid C. Cricothyroid D. Superior laryngeal E. Suprahyoid

B. Compression of the optic chiasm can cause bitemporal hemianopia due to compression of nerve fibers coming from the nasal (medial) hemiretinas of both eyes. The optic chiasm is located in very close proximity above the pituitary gland. Compression of an optic nerve would cause complete blindness in the affected eye. Compression of an optic tract would cause homonymous hemianopia. Compression of the oculomotor nerve would cause the eye to deviate "out and down" (paralysis of the four extraocular muscles innervated by this nerve), ptosis (paralysis of levator palpebrae), and mydriasis (paralysis of constrictor pupillae). Compression of the abducens nerve would cause paralysis of the lateral rectus muscle, leading to medial deviation (adduction) of the eye.

A 32-year-old man is admitted to the emergency department with visual problems. Radiographic examination reveals a tumor of the adenohypophysis (anterior pituitary gland). Physical examination reveals a loss of the lateral halves of the fields of vision of both eyes (bitemporal hemianopia or "tunnel vision"). Which of the following structures was most likely compressed by the tumor? A. Optic nerve B. Optic chiasm C. Optic tract D. Oculomotor E. Abducens nerve

A. An acoustic neuroma (vestibular schwannoma or neurolemmoma) is a benign tumor of the vestibulocochlear nerve (CN VIII), which causes compression of the facial nerve (CN VII). This nerve leads from the inner ear to the brain. Although many such tumors will not grow or grow very slowly, growth can in some cases result ultimately in brainstem compression (as in this example), hydrocephalus, brainstem herniation, and death. It is diagnosed on MRI with gadolinium contrast as shown. Extension of the neuroma into the right internal auditory meatus can be seen on the coronal MRI (see arrow in Fig. 7-9). The exact cause of the tumor is unknown; most people with acoustic neuromas are diagnosed between the ages of 30 and 60. Due to advances in microsurgery, including intraoperative monitoring of facial and cochlear function, the risks of facial paralysis and hearing loss have been greatly reduced. Many acoustic neuromas can now be treated effectively with both surgery and targeted radiation therapy (gamma knife). The outcomes for those with small acoustic neuromas are better, whereas those with neuromas larger than 2.5 cm are likely to experience significant hearing loss postsurgery.

A 32-year-old man is admitted to the hospital with nausea, vomiting, and severe headache. An MRI examination reveals an acoustic neuroma (Fig. 7-9). Which of the following nerves is most likely compressed by the tumor? A. Facial B. Oculomotor C. Vagus D. Hypoglossal E. Abducens

E. The dilator pupillae, levator palpebrae superioris, and smooth muscle cells of blood vessels in the ciliary body all receive sympathetic innervation. The postsynaptic cell bodies of the sympathetic neurons that innervate these structures are located in the superior cervical ganglion. The intermediolateral cell column contains presynaptic sympathetic neurons, but it is located only at spinal cord levels T1 to L2. The other structures listed do not contain sympathetic cell bodies.

A 32-year-old man is admitted to the hospital with severe headache and visual problems. The dilator pupillae muscle, the smooth muscle cell fibers of the superior tarsal muscle (of Müller, part of the levator palpebrae superioris), and the smooth muscle cells of the blood vessels of the ciliary body are supplied by efferent nerve fibers. Which of the following structures contains the neural cell bodies of these fibers? A. Pterygopalatine ganglion B. Intermediolateral nucleus (lateral horn) C1 to C4 C. Geniculate ganglion D. Nucleus solitarius E. Superior cervical ganglion

C. The parotid duct, also known as the Stensen's duct, crosses the masseter muscle transversely and extends to the oral cavity where it opens by the upper second molar. The facial artery can be palpated in the groove anterior to the mandibular angle. The facial vein lies anterior to the artery, passing toward the angle of the lips, but does not ascend in close proximity to the masseter. All of the other vessels are located more deeply and cannot be palpated.

A 32-year-old woman asks you what is the soft, thin ridge of tissue that she can feel running forward across the masseter muscle toward her upper lip. You reassure her that is perfectly normal. Which of the following is the most likely structure she is feeling? A. Facial artery B. Maxillary artery C. Parotid duct D. Marginal mandibular branch of the facial nerve E. Facial vein

D. The accommodation reflex is performed by constriction of the pupil when trying to focus on a near object. This function is controlled by the parasympathetic nerve fibers carried in the oculomotor nerve from the Edinger-Westphal nucleus of the midbrain that synapse in the ciliary ganglion. Postganglionic axons act on the sphincter pupillae muscle to cause reduction in pupil diameter and on the ciliary muscle to cause relaxation of the suspensory ligament, allowing the lens to adopt a more spherical shape for near focusing. If there is a lack of accommodation, it means the action of the ciliary muscle is compromised. The ciliary muscle also gets parasympathetic innervation by postganglionic neurons evoked from the ciliary ganglion by GVE fibers of oculomotor nerve whose cell bodies are located in the EdingerWestphal nucleus. The superior salivatory nucleus is involved with lacrimation and salivation, not the ciliary muscle and accommodation. The superior cervical ganglion is a sympathetic ganglion; its postganglionic axons innervate the dilator pupillae muscle, which causes mydriasis, but not the miosis of accommodation. The trigeminal ganglion does not have parasympathetic fibers and does not innervate the ciliary muscle for accommodation.

A 32-year-old woman is admitted to the hospital after losing consciousness and collapsing in the middle of the street. A neurologic examination reveals absence of the accommodation reflex of her right eye. Which of the following is most likely involved in the pathology in this patient? A. Superior salivatory nucleus B. Superior cervical ganglion C. Nervus intermedius D. Edinger-Westphal nucleus E. Trigeminal ganglion

E. The superior tarsal muscle (of Müller), innervated by sympathetics, is smooth muscle that assists in elevating the eyelids and maintaining this position. Loss of sympathetic innervation will result in partial ptosis of the eyelid. The superior oblique, innervated by the trochlear nerve, moves the pupil downward from the adducted position (for example, as when the right eye gazes down toward the left foot). To test the trochlear nerve, ask the patient to look with each eye toward the tip of the nose. The orbicularis oculi, innervated by the facial nerve, is responsible for closure of the eye. The palpebral part closes the eyelids ordinarily; the lacrimal part contracts when the eye is closed more forcibly, resulting in increased tear movement across the globe (perhaps to flow down the cheeks). Damage to the levator palpebrae superioris, innervated by the oculomotor nerve, would result in complete, rather than partial, ptosis.

A 32-year-old woman is admitted to the hospital with headaches and dizziness. During physical examination it is noted that the patient has partial ptosis (drooping eyelid). Which of the following muscles is most likely paralyzed? A. Orbicularis oculi, lacrimal part B. Orbicularis oculi, palpebral part C. Levator palpebrae superioris D. Superior oblique E. Superior tarsal (of Müller)

A. If there is an injury to the internal laryngeal nerve, there is a loss of sensation above the vocal cords. In this case, for internal laryngeal injury to occur, one must conclude that the operative field extended above the position of the thyroid gland to the level of the thyrohyoid membrane. The external laryngeal nerve can be injured during a thyroidectomy, but its injury would result in paralysis of the cricothyroid muscle and weakened voice/hoarseness. Injury of the glossopharyngeal nerve would result in more widespread symptoms, including loss of sensation from the pharynx, posterior tongue, and middle ear. Injury to the hypoglossal nerve would cause deficits in motor activity of the tongue. Damage to the recurrent laryngeal nerve would result in paralysis of most laryngeal muscles, with possible respiratory obstruction, hoarseness, and loss of sensation below the vocal cords.

A 32-year-old woman is undergoing a thyroidectomy. Postoperatively the patient suffers from loss of sensation within the larynx from the vocal folds upward to the entrance into the larynx, allowing for aspiration of liquids into the airway. Which of the following nerves is most likely injured? A. Internal laryngeal nerve B. External laryngeal nerve C. Glossopharyngeal nerve D. Hypoglossal nerve E. Recurrent laryngeal nerve

A. Damage to the internal laryngeal nerve would result in a general loss of sensation to the larynx above the vocal cords, leaving the patient with an inability to detect food or foreign objects in the laryngeal vestibule. The external laryngeal nerve and recurrent laryngeal nerve are both at risk during thyroidectomy. Damage to the recurrent laryngeal nerve would result in paralysis of all the laryngeal muscles except the cricothyroid; it would render the patient hoarse, with a loss of sensation below the vocal cords. Loss of the external laryngeal nerve would lead to paralysis of the cricothyroid muscle and vocal weakness. Injury to the hypoglossal nerve would result in weakness or paralysis of muscle movement of the tongue.

A 32-year-old woman underwent a thyroidectomy. Two months postoperatively, it was observed that the patient had lost the ability to notice the presence of foreign objects in the laryngeal vestibule. Which of the following nerves was most likely injured? A. Internal laryngeal nerve B. External laryngeal nerve C. Glossopharyngeal nerve D. Hypoglossal nerve E. Recurrent laryngeal nerve

B. Diploic veins are responsible for communication between the veins of the scalp and the venous sinuses of the brain. Diploic veins are situated within the layers of bone of the skull and connect the emissary veins of the scalp to the venous sinuses located between two layers of dura. The diploe are of clinical significance in that the diploic veins within this layer provide a pathway of communication between the veins of the scalp and underlying venous sinuses of the brain, by means of emissary veins. The emissary veins and diploe provide a potential vascular pathway of infection. The supratrochlear and supraorbital veins are located superficially on the scalp, immediately superior to the upper eyelid, and do not communicate directly with the venous sinuses of the brain. The anterior cerebral vein is an intracranial vein and, as such, does not maintain a direct communication with the external veins of the scalp. The superior sagittal sinus receives blood from the cerebral, diploic, and emissary veins; however, it does not provide a pathway of communication to the veins of the scalp.

A 34-year-old man is admitted to the emergency department after falling off his motorbike, suffering an injury to his head because he was not wearing a helmet. The patient has multiple lacerations in the skin over the frontal bone. Which of the following veins could most likely provide a pathway of transmission of infection from the veins of the scalp to the underlying dural venous sinuses? A. Supratrochlear vein B. Diploic veins C. Anterior cerebral veins D. Superior sagittal sinus E. Supraorbital vein

D. A tumor at the hypoglossal canal would compress the hypoglossal nerve (CN XII) and affect the genioglossus, a muscle it supplies. The palatoglossus is innervated by the vagus nerve (CN X), and the thyrohyoid is innervated by the ansa cervicalis (C1 to C3). The geniohyoid is supplied by C1, which runs with the hypoglossal nerve after it passes through the hypoglossal canal, and would therefore be unaffected. The mylohyoid is supplied by the nerve to mylohyoid, a branch of the mandibular division of the trigeminal nerve (CN V3).

A 34-year-old man is admitted to the hospital with severe headaches, dizziness, and vomiting. Imaging studies reveal a tumor at the hypoglossal canal. Which of the following muscles will most likely be affected? A. Geniohyoid B. Mylohyoid C. Palatoglossus D. Genioglossus E. Thyrohyoid

B. The vagus nerve innervates a part of the external auditory meatus and, when stimulated, can trigger a cough reflex in about 20% of people. This is thought to be due to "referred sensation" from the vestibule of the larynx, which is innervated by the vagus nerve. The vestibulocochlear nerve is associated only with the inner ear. The trigeminal nerve does provide some innervation to the external auditory meatus but does not affect the cough reflex as does the vagus nerve. The auricular branch of the facial nerve only provides a small amount of general sensory supply to the external ear; it is not associated with the cough reflex.

A 34-year-old man who swims regularly presents to your office with an external ear canal infection (otitis externa). The patient coughs during inspection of the external auditory meatus with a speculum. The cough results from the irritation of which nerve that innervates an area of the external auditory meatus? A. Vestibulocochlear B. Vagus C. Trigeminal D. Facial E. Accessory

B. Cricothyrotomy is an emergency airway procedure performed to ensure immediate airway ventilation in cases of laryngeal obstruction. It is performed by making an incision at the cricothyroid membrane, which is located between the thyroid cartilage and the cricoid cartilage. All the other options are incorrect because the cricothyroid membrane is located between the thyroid cartilage and the cricoid cartilage and this is where the incision is made.

A 34-year-old woman bursts through the doors of the emergency department. She is straining to take a breath but can only mouth, "I can't breathe" before collapsing. She is placed on a stretcher. Her tongue is swollen and protruding from her mouth. The patient has only minimal air movement with bag-mask ventilation. Oxygen saturation is approximately 80%. Attempts at oral intubation are unsuccessful due to massive soft tissue edema of her pharynx. A decision is made to perform a cricothyrotomy. After palpation of the neck to identify the appropriate landmarks, an incision should most likely be made at which of the following locations? A. The cricothyroid membrane, which is located at the junction of the clavicle and the sternum B. The cricothyroid membrane, which is located between the thyroid cartilage and the cricoid cartilage below C. The thyrohyoid membrane, which is located between the thyroid cartilage (Adam's apple) and the hyoid bone above D. The sternal notch, which is located at the junction of the clavicle and the sternum E. The trachea, which is located below the cricoid cartilage

A. Paralysis of the trochlear nerve results in loss of ability for the affected eye to be directed downward when the pupil is in the adducted position (the primary action of the superior oblique muscle). The patient must tilt her head toward the opposite side to allow the two pupils to converge on an object on the floor. Paralysis of the trochlear nerve is not unusual when a patient's head has hit the dashboard in an automobile crash—the delicate nerve is easily torn where it pierces the dura of the tentorium cerebelli in the tentorial notch because the brain and brainstem move forward and backward with the force of impact (a "coup-contrecoup" injury).

A 34-year-old woman is admitted to the emergency department after her right cheekbone and bony orbit hit the dashboard in an automobile crash. Physical examination reveals that the patient has lost the ability for the affected eye to be directed downward when the pupil is in the adducted position. An MRI examination reveals a torn nerve. What is the most common location at which this nerve will be injured? A. As it pierces the dura of the tentorium cerebelli in the tentorial notch B. At the cavernous sinus C. At the sella turcica D. At the inferior orbital fissure E. At the superior orbital fissure

B. A chalazion is caused by an obstructed tarsal gland of the eyelid. Swellings of the lacrimal gland usually present on the upper lateral eyelid and are not indicative of a chalazion. A chalazion is not an infection within the eye, so this excludes sclera and pupil from being the correct answers. The nasolacrimal duct runs from the medially located lacrimal sacs to the inferior meatus of the nose and would be unaffected in the case of a chalazion.

A 34-year-old woman is admitted to the emergency department with a painful eye. Physical examination reveals a lump in the lower eyelid that consists of debris (Fig. 7-13). A diagnosis of a chalazion is made. Which of the following structures is most likely blocked by the chalazion? A. Lacrimal ducts B. Tarsal glands C. Sclera D. Pupil E. Nasolacrimal duct

E. The middle of the vocal cord would be the most likely location of the tumor because there is no direct lymph drainage from this region. All other locations mentioned are drained by the lymphatics. Areas above the vocal cords are drained by the superior deep cervical nodes, and areas below the vocal cords drain to the pretracheal nodes before draining into the inferior deep cervical nodes.

A 34-year-old woman is admitted to the hospital because of hoarseness for the past 3 months. Radiologic examination reveals a cancerous growth in her larynx with no evidence of metastasis. In addition, the area in which the tumor is growing is characterized by very limited lymphatic drainage. Which of the following locations is most likely to contain a tumor with these characteristics? A. Anterior commissure of the vocal ligaments B. Interarytenoid fold C. Laryngeal ventricle D. Cricothyroid ligament E. Middle segment of the vocal cord

D. The recurrent laryngeal nerve is the most likely nerve damaged during the surgery because it runs in close proximity to the inferior thyroid artery and is easily injured or transected with the artery if extreme care is not exercised during operative procedures. The recurrent laryngeal nerve innervates the majority of the vocal muscles that open and close the rima glottidis, in addition to providing sensory supply to the larynx below the vocal folds. Even relatively mild trauma to the nerve can result in hoarseness. The internal branch of the superior laryngeal nerve is not in close proximity to the inferior thyroid artery and pierces the thyrohyoid membrane to enter the pharynx. The ansa cervicalis lies lateral to the site of surgery and does not innervate any structures that, if paralyzed, would cause hoarseness.

A 34-year-old woman is admitted to the hospital with a large mass at her thyroid gland. Ultrasound examination suggests a benign tumor, which is confirmed with a biopsy. Twenty-four hours following a partial thyroidectomy, in which the inferior thyroid artery was also ligated, the patient speaks with a hoarse voice and has difficulty in breathing on exertion. Which of the following nerves was most likely injured during the surgical procedure? A. Internal branch of superior laryngeal B. Ansa cervicalis C. Ansa subclavia D. Recurrent laryngeal E. External branch of superior laryngeal

C. During removal of the tumor, the internal branch of the superior laryngeal nerve was injured. Injury to this nerve results in loss of sensation above the vocal cords, at the entrance to the larynx, and loss of taste on the epiglottis. Loss of sensation in the laryngeal vestibule can precipitate aspiration of fluid into the larynx, trachea, and lungs. The pharyngeal nerve from the vagus nerve supplies motor innervation to the muscles of the pharynx, except the stylopharyngeus (glossopharyngeal nerve). Injury to the hypoglossal nerve would result in protrusion of the tongue toward the affected side and moderate dysarthria. The lingual nerve conveys parasympathetic preganglionic fibers to the submandibular ganglion and general sensation and taste fibers for the anterior two thirds of the tongue. The recurrent laryngeal provides sensory fibers to the larynx below the vocal cords and motor fibers to all of the muscles of the larynx except for the cricothyroid.

A 34-year-old woman is admitted to the hospital with a large mass in her lower anterior neck. Ultrasound examination reveals a benign tumor of her thyroid gland. Twenty-four hours following a partial thyroidectomy, it was noted that the patient frequently aspirates fluid into her lungs. Upon examination it was determined that the area of the piriform recess above the vocal fold of the larynx was anesthetized. Which of the following nerves was most likely iatrogenically injured? A. External branch of the superior pharyngeal B. Hypoglossal C. Internal branch of the superior laryngeal D. Lingual E. Recurrent laryngeal

A. The chorda tympani joins the lingual nerve in the infratemporal fossa, and a lesion to the lingual nerve before it joins the chorda tympani would account for the loss of general sensation, with no loss to the special sense of taste and saliva production. If the chorda tympani were injured, the patient would present with a loss of taste (from the anterior two thirds of tongue) and a decrease in saliva production because the submandibular and sublingual salivary glands would be denervated. The inferior alveolar nerve provides sensory innervation to the mandibular teeth, but no such loss is present. The lesser petrosal nerve innervates postganglionic neurons supplying the parotid gland, but no loss of salivation is present. The glossopharyngeal nerve provides taste innervation to the posterior third of the tongue and sensation related to the gag reflex, but there is no deficit present in this patient.

A 35-year-old man is admitted to the hospital with severe headaches. A CT scan evaluation reveals a tumor in the infratemporal fossa. Physical examination reveals loss of general sensation from the anterior two thirds of his tongue, but taste and salivation are intact. Which of the following nerves is most likely affected by the tumor? A. Lingual proximal to its junction with the chorda tympani B. Chorda tympani C. Inferior alveolar D. Lesser petrosal E. Glossopharyngeal

B. The lingual nerve initially courses directly underneath the mucosa of the floor of the mouth and superficial to the submandibular gland, specifically the submandibular duct. This nerve is therefore at risk for ligation, division, or trauma during excision of the gland and duct. The lingual nerve is part of the mandibular division of the trigeminal nerve and carries fibers from the chorda tympani, a branch of the facial nerve (CN VII). These latter fibers supply taste to the anterior two thirds of the tongue and preganglionic parasympathetic axons involved in salivary gland secretion. Fibers of the mandibular division of the trigeminal nerve (CN V3) give rise to the lingual nerve and supply general sensation to the anterior two thirds of the tongue. The lingual nerve passes deep both to the lateral pterygoid muscle and the ramus of the mandible and subsequently travels deep to the submandibular gland itself. The buccal nerve, also a branch of the mandibular division of the trigeminal nerve, supplies the mucosa of the cheek and is not in close proximity to the gland or duct. The inferior alveolar nerve, though close in proximity to the submandibular gland, travels deep to the lateral pterygoid muscle and later enters the mandibular canal to supply the lower teeth. The nerve to the mylohyoid, a branch of the inferior alveolar nerve, supplies the mylohyoid muscle and the anterior belly of the digastric. Neither of these nerves is at risk for damage during excision of the submandibular gland and duct.

A 35-year-old man is admitted to the hospital with severe pain in the area of his right submandibular gland. Radiographic examination reveals a tumor of the gland. The submandibular gland and its duct are removed. Which of the following nerves is most prone to injury in this type of procedure? A. Buccal B. Lingual C. Inferior alveolar D. Nerve to mylohyoid E. Glossopharyngeal

C. The most likely structures one would encounter while performing a midline incision below the isthmus of the thyroid gland would be the inferior thyroid vein and the thyroidea ima artery. The inferior thyroid vein drains typically to the left brachiocephalic vein, which crosses superficially, just inferior to the isthmus. The thyroidea ima artery arises from the aortic arch, vertebral artery, or other source but is not a constant structure. The middle thyroid veins drain the thyroid gland to the internal jugular vein and are superior to the incision site. The inferior thyroid arteries branch from either subclavian artery and meet the thyroid gland at an oblique angle. They would not be ligated with a midline incision. The brachiocephalic veins are inferior to the site of incision.

A 35-year-old woman is admitted to the emergency department after a violent automobile crash. The patient's upper airway is obstructed with blood and mucus, and a midline tracheostomy inferior to the thyroid isthmus is performed. Which of the following vessels are most likely to be present at the site of incision and will need to be cauterized? A. Middle thyroid vein and inferior thyroid artery B. Inferior thyroid artery and inferior thyroid vein C. Inferior thyroid vein and thyroidea ima artery D. Cricothyroid artery and inferior thyroid vein E. Left brachiocephalic vein and inferior thyroid artery

B. The superior ophthalmic vein drains directly into the cavernous sinus. The danger area of the face is located in the triangular region from the lateral angle of the eye to the middle of the upper lip, near the nose, and is drained by the facial vein. The facial vein communicates directly with the cavernous sinus through the superior ophthalmic vein. The pterygoid venous plexus communicates with the cavernous sinus through the inferior ophthalmic vein, but it is not directly connected to the cavernous sinus. The basilar venous plexus connects the inferior petrosal sinuses and communicates with the internal vertebral venous plexus. The parietal emissary veins and frontal venous plexus do not communicate directly with the cavernous sinus.

A 35-year-old woman is hospitalized due to cavernous sinus thrombosis resulting from an infection on her face. Which of the following is the most direct route for spread of infection from the face to the cavernous sinus? A. Pterygoid venous plexus B. Superior ophthalmic vein C. Frontal venous plexus D. Basilar venous plexus E. Parietal emissary vein

A. The posterior cricoarytenoid muscles lie on the superoposterior aspect of the lamina of the cricoid cartilage. When these muscles contract, they cause lateral rotation (abduction) of the vocal processes of the arytenoid cartilages, thereby opening the space between the vocal folds, the rima glottidis. The lateral cricoarytenoid is involved with adducting the arytenoid cartilage and closing the rima glottidis. The thyroarytenoid muscles lie alongside either vocal ligament and are also involved in adducting the vocal folds. The transverse arytenoid muscle connects both arytenoid cartilages and also aids in closing the rima glottidis. Finally, the cricothyroid muscle is located on the anterior aspect of the cricoid cartilage and aids in elongation and tensing of the vocal folds, thus raising the pitch of the voice.

A 35-year-old woman is under general anesthesia. Before laryngeal intubation the rima glottidis is opened by which pair of muscles? A. Posterior cricoarytenoids B. Lateral cricoarytenoids C. Thyroarytenoids D. Transverse arytenoids E. Cricothyroids

C. A blow-out fracture of the medial wall of the orbit would likely render the medial rectus muscle nonfunctional by entrapment of the muscle between the fracture fragments of the cracked medial wall. The medial rectus is responsible for adduction of the eye, but in this case the muscle acts as a tether or anchor on the eyeball, preventing lateral excursion (abduction) of the eye. There is no nerve damage here, and the muscle is not paralyzed. The lateral rectus is responsible for abduction of the eye, and the inferior rectus rotates the eyeball downward. Damage to these muscles or their nerve supply would result in an inability to move the eye laterally and inferiorly, respectively.

A 36-year-old female racquetball player is admitted to the hospital after being struck in the orbital region. Radiographic examination reveals a blow-out fracture of the medial wall of the orbit. Physical examination also reveals that the pupil of the affected eye cannot be turned laterally. Which of the following muscles is most likely injured or trapped? A. Lateral rectus B. Medial and inferior recti C. Medial rectus D. Medial rectus and superior oblique E. Inferior rectus

A. The petrous part of the temporal bone houses the middle and inner ear and has the facial canal with the facial nerve passing through it. Damage to the infratemporal fossa will not cause hearing loss or vertigo but will produce additional symptoms. The middle ear could account for the hearing loss, and the inner ear for hearing loss and vertigo, but not the facial palsy or lack of lacrimation.

A 40-year-old woman presented to her family practitioner with complaints of vertigo and nausea and reduced hearing through her left ear for 3 months. She also complained of a dry mouth and dryness of the left eye. CT scan of the head showed thickening of the bones of the skull. Which of the following areas was most likely affected to produce this patient's symptoms? A. Petrous temporal bone B. Infratemporal fossa C. Facial canal D. Middle ear E. Inner ear

A. A herpes rash on the dorsum of the nose is known as Hutchinson's sign. This indicates that the virus is located in cell bodies of the ophthalmic division of the trigeminal nerve. This nerve branches into nasociliary, frontal, and lacrimal branches. The nasociliary nerve has direct branches that carry sensory innervation from the eye. The nasociliary nerve also gives off the ethmoidal nerves that innervate the superior nasal mucosa, in addition to providing the origin of the dorsal nasal nerve. The supratrochlear nerve is a branch of the frontal nerve and carries sensory innervation from the skin superior to the orbit. The infraorbital nerve is a branch from the maxillary division of the trigeminal nerve and carries sensory innervation from the skin of the face between the orbit and the upper lips.

A 36-year-old man is admitted to the emergency department with a painful skin rash on the dorsum of his nose. Physical examination reveals that a herpetic lesion is affecting the dorsum of the nose and the eyeball. Which of the following nerves is most likely to be responsible for transmission of the virus to the eye? A. Nasociliary B. Supratrochlear C. Infraorbital D. Posterior ethmoidal E. Anterior ethmoidal

A. An injury to the left vagus nerve (CN X) would cause the uvula to become deviated to the right. This is because the vagus nerve innervates the musculus uvulae muscle that makes up the core of the uvula. If only one side is effectively innervated, contraction of the active muscle will deviate the uvula to the contralateral side of the injury (ipsilateral side of the uninjured vagus nerve). In addition, the intact levator veli palatini will pull the uvula to the intact side. The right and left hypoglossal nerves innervate the tongue muscles and would not affect the uvula. The glossopharyngeal nerve supplies sensory innervation to the oropharynx and nasopharynx, but not motor innervation to these areas.

A 36-year-old woman is admitted to the hospital with severe head injuries after a car crash. During neurologic examination her uvula is found to be deviated to the right. Which nerve is most likely affected to result in this deviation? A. Left vagus B. Right vagus C. Right hypoglossal D. Left glossopharyngeal E. Right glossopharyngeal

A. The uvula would move toward the intact right side. This is because the intact levator veli palatini would be unopposed by the opposite, paralyzed left levator veli palatini.

A 36-year-old woman is admitted to the hospital with severe head injuries after a car crash. During neurologic examination it is noted that her palate elevates asymmetrically, being pulled up toward the right. Which of the following muscles is paralyzed? A. Left levator veli palatini B. Left tensor veli palatini C. Right levator veli palatini D. Right tensor veli palatini E. Right tensor veli palatini and left levator veli palatini

A. The posterior cricoarytenoids are the only muscles of the larynx that abducts the vocal cords. The remaining answer choices are muscles that act in adduction of the vocal cords.

A 38-year-old man is admitted to the hospital with a large mass in his lower anterior neck. Ultrasound examination reveals a benign tumor of his thyroid gland. Twenty-four hours following a partial thyroidectomy, it was noted that the patient could not abduct the true vocal cords due to a nerve injury during the operation. Which of the following muscles was most likely denervated? A. Posterior cricoarytenoid B. Lateral cricoarytenoid C. Thyroarytenoid D. Arytenoid E. Cricothyroid

B. The inferior alveolar branch of the mandibular division of the trigeminal nerve provides sensory innervation to the mandibular teeth and would require anesthesia to abolish painful sensation. The lingual nerve provides taste and sensation to the anterior two thirds of the tongue and carries general sensory fibers, taste fibers, and parasympathetic fibers. It does not provide sensory innervation to the teeth. The buccal nerve provides sensory innervation to the inner surface of the cheek. The mental nerve is the distal continuation of the inferior alveolar nerve as it exits the mental foramen of the mandible and does not affect the teeth. The nerve to the mylohyoid is a motor branch of the inferior alveolar nerve that supplies the mylohyoid and the anterior belly of the digastric.

A 38-year-old woman is admitted to the dental clinic with acute dental pain. The attending dentist found penetrating dental caries (tooth decay) affecting one of the mandibular molar teeth. Which of the following nerves would the dentist need to anesthetize to treat the caries in that tooth? A. Lingual B. Inferior alveolar C. Buccal D. Mental E. Mylohyoid

A. The function of the cricoarytenoid muscle is to abduct the vocal cords. A transverse arytenoid and lateral cricoarytenoid muscle adducts the vocal cords. Cricothyroid and thyroarytenoid muscles lengthen the vocal cords and narrow the laryngeal cavity, respectively

A 39-year-old man presents to an ENT specialist with hoarseness. Laryngoscopy shows nodules on the vocal cords with normal movement. Which muscle functions to open the rima glottidis? A. Posterior cricoarytenoids B. Lateral cricoarytenoids C. Thyroarytenoids D. Transverse arytenoids E. Cricothyroids

C. Ankyloglossia (tongue-tie) is characterized by a lingual frenulum that extends all the way to the tip of the tongue. This condition can cause problems with speech, feeding, and oral hygiene as a result of the low range of motion of the tongue. Ankyloglossia can be treated surgically by cutting the lingual frenulum. None of the other procedures described would treat this condition.

A 4-year-old boy suffering from ankyloglossia is brought to the speech therapist. The examining physician recommends that the child be admitted for operation by a pediatric surgeon. Which of the following surgical procedures would be most appropriate for this condition? A. Removal of pterygomandibular raphe B. Resection of the pterygoid hamulus bilaterally C. Cutting the lingual frenulum D. Repair of the palate E. Removal of the central segment of the hyoid bone

D. The nerves involved are the lingual and buccal nerves, which are branches of the mandibular branch of the trigeminal nerve. These branches provide sensation to mucosa of the anterior two thirds of the tongue (general sensation), adjacent gums, cheek mucosa, and overlying skin.

A 40-year-old man was brought to the emergency department after falling through a glass window to the ground 5 feet below. He complained of pain from multiple lacerations across his body, particularly from a laceration to the left cheek. On examination, he was comfortable with stable vital signs. There were multiple superficial lacerations on his upper and lower limbs but no deformities of bones or joints were evident. There was a 5-cm longitudinal laceration over the left jaw. Gentle exploration of the wound revealed that it was deep and a foreign body was present. A CT scan of the head confirmed that a 3 × 2 cm foreign body, possibly glass, was lodged between the two heads of the lateral pterygoid muscle. Which of the following clinical findings are most likely to be found in this patient? A. Weak elevation of the jaw B. Weak deviation of the jaw to the right side C. Numbness over the skin of the tragus and helix of the left ear D. Numbness over the skin and mucosa of the anterior cheek E. Decreased volume of saliva

E. The middle meningeal artery is a branch of the maxillary artery and courses between the dura mater and skull close to the area of the pterion. Any fracture or impact trauma to this location typically results in a laceration of the middle meningeal artery resulting in an epidural hematoma. The external carotid artery ends behind the mandible by dividing into the maxillary and the superficial temporal arteries, and neither of these arteries directly affects the meninges of the brain. The deep temporal arteries do not penetrate the bony skull and thus would not contribute to an epidural hematoma.

A 40-year-old unconscious man is admitted to the emergency department after being hit in the head with a baseball. A CT scan examination reveals a fractured pterion and an epidural hematoma. Branches of which of the following arteries are most likely to be injured? A. External carotid B. Superficial temporal C. Maxillary D. Deep temporal E. Middle meningeal

B. The masseter muscle originates on the zygomatic bone and inserts on the lateral surface of the ramus of the mandible. It is a powerful elevator of the mandible. Grinding of the teeth can lead to hypertrophy of the muscles of mastication, especially the masseter. Patients will have difficulty in depressing the mandible due to spasms of this enlarged muscle.

A 40-year-old woman comes to the physician because of pain in the region of her left jaw, left-sided earache, and headache for 3 days. The patient has not had any trauma to her face or jaw but says she often grinds her teeth. She is concerned because she is a singer, and it is painful when she opens her mouth wide to sing. There is also a clicking sound when she opens her mouth. Physical examination shows the left side of the jaw deviating slightly to the left on elevation. The area around the left mandibular condyle is painful on palpation. Mandibular depression is difficult to perform because of pain. This movement also elicits an audible clicking sound. There is tightness indicative of a muscle spasm along the left mandibular ramus. Palpation shows no other area of tightness. Spasms of which of the following muscle are most likely associated with this condition? A. Buccinator B. Masseter C. Mylohyoid D. Posterior belly of the digastric E. Lateral pterygoid

E. Due to the mass in the lumen of the parotid duct and distended parotid gland, nerves passing through the gland can be affected. The trunk of the facial nerve divides into temporofacial and cervicofacial divisions, which then further divide into temporal (supplies temple, forehead, and supraorbital muscles), zygomatic (infraorbital, lateral nasal, and upper lip muscles), buccal (upper lip and muscles at the corner of mouth), marginal mandibular (lower lip and chin muscles), and cervical branches (platysma muscle). Compression of any or all of the branches affects corresponding muscles and their functions. Pain sensation over the anterior auricle, tragus, and anterior helix is due to compression of the auriculotemporal nerve, which passes through the parotid gland and ascends just anterior to the ear, supplying the external acoustic meatus, surface of the tympanic membrane, and large area of the temple.

A 40-year-old woman presented to the emergency department with complaints of swelling and pain over the left side of her face. On examination, there was tender, warm swelling over the left mandible anterior to the ear. Ultrasound revealed a hyperechoic mass within the lumen of the parotid duct, which was distended proximally. What other clinical features are most likely present in this patient? A. Weak deviation of the mandible to the right B. Complete ptosis of the left eyelid C. Numbness of the skin over the left lower mandible D. Excessive tearing E. Pain over the anterior auricle, tragus, and anterior helix

B. The superior cerebellar artery originates immediately before the termination of the basilar artery. It passes lateral below the oculomotor nerve to give blood supply to the anterior and medial parts of the cerebellum. An aneurysm of the superior cerebellar artery would result in oculomotor nerve palsy. This will affect the parasympathetics to the ciliary body and pupillary constrictor resulting in blurry vision, the levator palpebrae superioris resulting in ptosis, and the extrinsic muscles of the eye with the exception of the lateral rectus and superior oblique, resulting in lateral deviation (abduction) of the eye.

A 40-year-old woman presented to the neighborhood clinic with complaints of a constant, worsening headache for 4 months that was unresponsive to pain medication. The pain was worse on the right side. She denied having fever, neck stiffness, or aversion to light. On examination, there was no motor weakness of the upper or lower limbs. A CT scan of the brain with contrast revealed a berry aneurysm of the right superior cerebellar artery. Which of the following will most likely be identified during physical examination of this patient? A. Lack of accommodation, adduction of the right eye B. Blurred vision, complete ptosis, abduction of the right eye C. Partial ptosis, adduction and downward rotation of the right eye D. Complete ptosis, abduction and upward rotation of the right eye E. Complete ptosis, abduction and downward rotation of the right eye

C. Melanocytes in the pigmented layer of the retina are a potential source of malignant melanoma cells. The tumor spreads hematogenously directly to the brain and has a very poor prognosis. None of the other listed structures contains melanocytes

A 40-year-old woman presents with severe headaches and dizziness. An MRI reveals a brain tumor, and a biopsy confirms it as an advanced melanoma. She dies 2 months later. Pigmented lesions are not seen on her skin or scalp at the time of diagnosis or during postmortem examination. Which of the following is the most likely source of the malignant melanoma cells? A. Superior sagittal sinus B. Sphenoidal sinus C. Retina of the eye D. Pituitary gland E. Thymus

D. The greater petrosal nerve is a branch of the facial nerve that ultimately supplies the lacrimal gland. This branch comes off the facial nerve (CN VII) at the geniculate ganglion proximal to the chorda tympani. The greater petrosal nerve is unlikely to be involved in a lesion of the facial nerve as described. The other listed functions of the facial nerve would be affected by the lesion.

A 40-year-old woman suffers severe head trauma in a car crash. After radiographic examination she is diagnosed with a fracture of the temporal bone resulting in a lesion of the facial nerve proximal to the origin of the chorda tympani in the posterior wall of the tympanic cavity. Which of the following functions would most likely remain intact in this patient? A. Control of muscles in lower half of face B. Control of secretions by submandibular gland C. Taste sensation from anterior two thirds of tongue D. Tear production by the lacrimal gland E. Voluntary closure of the eyelid

C. The greater petrosal nerve carries parasympathetic fibers that are involved in the innervation of the lacrimal gland, as well as the mucosal glands of the nose, palate, and pharynx. As a result, an injury to the right greater petrosal nerve would be expected to result in decreased lacrimal secretions for the right eye. The sublingual and submandibular glands receive their parasympathetic fibers from the facial nerve via the chorda tympani and the lingual nerve. They would be unaffected by this lesion. The parotid gland receives its parasympathetic secretory innervation from the glossopharyngeal nerve via the lesser petrosal and auriculotemporal nerves and would be unaffected. Taste to the anterior tongue is provided by the facial nerve via the chorda tympani, and general sensation to the anterior tongue is provided by the mandibular division of the trigeminal nerve via the lingual nerve.

A 43-year-old man is admitted to the emergency department with a fracture of the base of his skull. A thorough physical examination reveals that a number of structures have been injured, suggesting that the right greater petrosal nerve has been injured. Which of the following symptoms needs to be identified during physical examination to confirm the diagnosis of greater petrosal nerve injury? A. Partial dryness of the mouth due to lack of salivary secretions from the submandibular and sublingual glands B. Partial dryness of the mouth due to lack of salivary secretions from the parotid gland C. Dryness of the right cornea due to lack of lacrimal gland secretion D. Loss of taste sensation from the right anterior two thirds of the tongue E. Loss of general sensation from the right anterior two thirds of the tongue

E. The superior oblique muscle turns the pupil downward from the adducted position. Inability to perform this motion, in conjunction with diplopia when walking down stairs, indicates damage to the trochlear nerve. The abducens innervates the lateral rectus, resulting in abduction of the eye. The oculomotor nerve supplies the superior, inferior, and medial rectus as well as the inferior oblique muscles. Overall, innervation from the oculomotor nerve results in upward and downward movements of the eye. Damage to this nerve would not induce diplopia when an affected individual walks down stairs. In addition, inability to gaze downward in the adducted position does not indicate oculomotor nerve damage. In this position the oculomotor nerve would be responsible for upward movement. The nasociliary nerve is a sensory nerve originating from the ophthalmic branch of the trigeminal nerve.

A 43-year-old man is admitted to the hospital complaining of diplopia (double vision) when walking down stairs. During physical examination of the extraocular muscles the patient experiences diplopia, and when he is asked to turn his right eye inward toward his nose and look down, he is able to look inward but not down. Which nerve is most likely involved? A. Abducens B. Nasociliary C. Oculomotor, inferior division D. Oculomotor, superior division E. Trochlear

C. Because of the surgical division of the ansa cervicalis, the sternohyoid muscle will most likely be paralyzed following this tumor resection. The ansa cervicalis innervates the strap muscles, including the sternohyoid, sternothyroid, and omohyoid muscles. The sternocleidomastoid is innervated by the spinal accessory nerve and will not be involved with this surgery. The platysma is located most superficially on the neck and is innervated by the cervical branch of the facial nerve. The trapezius muscle is also innervated by the spinal accessory nerve and plays no role in ansa cervicalis functions. Finally, the cricothyroid muscle is innervated by the external laryngeal branch of the vagus and would not be affected by the surgery.

A 43-year-old man is diagnosed with laryngeal carcinoma. A surgical procedure is performed and the tumor is successfully removed from the larynx. The right ansa cervicalis is anastomosed with the right recurrent laryngeal nerve to reinnervate the muscles of the larynx and restore phonation. Which of the following muscles will most likely be paralyzed after this operation? A. Sternocleidomastoid B. Platysma C. Sternohyoid D. Trapezius E. Cricothyroid

E. Ptosis and miosis occur in response to blocking of sympathetic innervation. Ptosis (drooping of the eyelid) results from lack of innervation of the superior tarsal muscle (of Müller), and miosis (pupillary constriction) results from unopposed parasympathetic innervation of the pupil. A dilated pupil would not occur because this requires the action of the sympathetically innervated dilator pupillae. Dry eye would occur due to lacrimal gland insufficiency, but because this is mediated by parasympathetic fibers, it would remain unaffected in this case. The same holds true for the parasympathetically mediated accommodation pathway. Depth perception involves the visual pathway and is not mediated by the sympathetic system.

A 44-year-old woman is being treated for Raynaud's disease. A sympathetic blocking drug is administered in high doses. Which of the following conditions will most likely be expected to occur as an adverse effect of the drug? A. Exophthalmos and dilated pupil B. Enophthalmos and dry eye C. Dry eye and inability to accommodate for reading D. Wide open eyelids and loss of depth perception E. Ptosis and miosis

E. During a puncture wound as described in this case, passing up from below the chin, the nail would first pierce the platysma, then the anterior belly of the digastric, then the mylohyoid, then the geniohyoid, and finally the genioglossus.

A 45-year-old male construction worker slips and falls on a nail protruding from a board. The nail penetrates the skin overlying the submental triangle lateral to the midline. Which of the following muscles would be the last to be penetrated? A. Platysma B. Mylohyoid C. Anterior belly of the digastric D. Geniohyoid E. Genioglossus

B. The preauricular lymph nodes are also known as the deep parotid nodes. They are located deep to the parotid gland and drain lymph from the posterior orbit. The submandibular nodes drain the side of the cheek and lateral aspects of the nose and lips. The superficial parotid lymph nodes lie superficially to the parotid gland and drain the lateral angles of the eyelids, aspects of the nose, and the external acoustic meatus. The jugulodigastric nodes receive drainage from all of the superior nodes of the face and also drain the tonsils. The submental lymph nodes drain the tip of the tongue and chin.

A 45-year-old man is admitted to the emergency department with a red, painful eye. During physical examination it is noted that the conjunctiva of the affected eye is infected. Which of the following lymph node groups would be first involved if the infection spread? A. Submandibular B. Parotid C. Jugulodigastric D. Submental E. Preauricular

E. An infection of the submandibular space is usually the result of a dental infection in the mandibular molar area in the floor of the mouth (Ludwig's angina). If the patient is not treated with antibiotics promptly, the pharyngeal and submandibular swelling can lead to asphyxiation. Quinsy, also known as peritonsillar abscess, is a pus-filled inflammation of the tonsils that can occur due to tonsillitis. Ankyloglossia, which is also known as tongue-tie, is a congenital defect that results in a shortened lingual frenulum that restricts movement of the tongue. The affected person will usually have a speech impediment. Torus palatinus is a benign bony growth on the hard palate; a torus mandibularis is a similar growth on the inside of the mandible. Such growths are usually benign and would not typically cause pain. A ranula is a mucocele found on the floor of the mouth, often resulting from dehydration in older individuals, with coagulation (inspissation) of salivary secretions. It can be caused by acute local trauma; however, they are usually asymptomatic.

A 45-year-old man is admitted to the emergency department with severe dyspnea. During physical examination there is swelling in the floor of his mouth and pharynx and his airway is nearly totally occluded. In addition, there is a swelling in his lower jaw and upper neck. His physical history indicates that one of his lower molars was extracted a week ago and he had been feeling worse every day since. Which of the following conditions will be the most likely diagnosis? A. Quinsy B. Torus palatinus C. Ankyloglossia D. Ranula E. Ludwig's angina

B. The nasal polyp also involved the maxillary sinus, located immediately laterally to the nasal cavity. The sphenoid sinus, located posterosuperiorly to the nasopharynx, is unlikely to be affected by a nasal polyp. The ethmoidal sinuses, located medially to the orbit and lateral to the nasal cavity, are also unlikely to be affected by a nasal polyp, although this possibility cannot be ruled out. The frontal sinuses located superomedially to the eyes are unlikely to be affected by the nasal polyp. The frontonasal ducts, the communication between the frontal sinus and the nasal cavity, are also unlikely to be affected.

A 45-year-old man is admitted to the hospital with breathing problems. During physical examination the patient shows signs of unilateral nasal airway obstruction. A CT scan reveals a nasal polyp obstructing the airway (Fig. 7-14). Drainage from which of the following structures is also obstructed? A. Sphenoid sinus B. Maxillary sinus and nasolacrimal duct C. Ethmoidal sinus D. Frontal sinus E. Nasolacrimal duct

B. If the left abducens nerve is injured, there will be a loss of function of the left lateral rectus muscle so the patient will be unable to abduct his left eye. The trochlear nerve supplies the superior oblique muscle, which if injured would cause the patient to lose the ability to turn the pupil downward when it is in the adducted position. As an example, the affected patient could not turn the pupil to look downward to the left if the right trochlear nerve were paralyzed. This deficiency can make it difficult for individuals to descend stairs if they have trochlear nerve palsy. If the oculomotor nerve were injured, the pupil would be directed "down and out" due to unopposed actions of the lateral rectus and superior oblique, which are innervated by the abducens and trochlear nerves, respectively. If the optic nerve were injured, the patient would have blindness in the affected eye. If the oculomotor and abducens nerves were injured, the patient would have only the actions of the superior oblique muscle, and the eye would be directed downward and outward from the position of forward gaze.

A 45-year-old man presents to the outpatient clinic after stumbling and hitting his head on a table in a restaurant. Photographs were taken of the patient's eyes during neurologic examination (Fig. 7-8). Which of the following nerves to the left eye was most likely injured? A. Trochlear B. Abducens C. Oculomotor D. Optic E. Oculomotor and abducens

C. The infraorbital branch of the maxillary division of the trigeminal nerve exits the front of the skull below the orbit through the infraorbital foramen. A needle inserted into the infraorbital foramen and directed posteriorly will pass through the foramen rotundum to reach the trigeminal ganglion and the beginning of the maxillary division of the trigeminal nerve. The mandibular division of the trigeminal nerve exits the skull through the foramen ovale. The middle meningeal artery exits the infratemporal fossa through the foramen spinosum to enter the cranial cavity. The inferior alveolar branch of the mandibular division passes into the mandibular foramen to then descend in the jaw to supply the mandibular teeth. The foramen magnum is where the spinal cord exits the skull and where the spinal accessory nerve ascends into the skull after arising from the cervical spinal cord and brainstem.

A 45-year-old man was suffering from trigeminal neuralgia (tic douloureux). The pain was so severe that the patient had considered suicide as a way to escape the pain. Even light, gentle stimuli to the skin between the lower eyelid and the upper lip resulted in severe, agonizing pain. It was decided to lesion the nerve branch involved by injecting alcohol into the nerve. To reach the nerve, the needle will most likely need to be inserted through which of the following openings? A. Foramen ovale B. Foramen spinosum C. Infraorbital foramen D. Mandibular foramen E. Foramen magnum

C. The pharyngeal tonsil is situated in a slitlike space, the pharyngeal recess, in the nasopharynx behind the opening of the auditory (eustachian) tube, and a pharyngeal tonsil in this location can lead to blockage of the drainage of the auditory tube. The lingual tonsil is located in the posterior aspect of the tongue, whereas the palatine tonsil is contained within the tonsillar fossa between the palatoglossal and palatopharyngeal arches. An enlargement of the lingual tonsil or the palatine tonsil will not occlude the auditory tube due to their location in the oropharynx. The superior pharyngeal constrictor would not be involved in occlusion of the auditory tube because it is located more posteriorly. The uvula is drawn upward during deglutition and prevents food from entering the nasopharynx; it does not block the auditory tube.

A 45-year-old man with a complaint of ear pain, difficulty hearing in one ear, nose bleeds, and difficulty breathing through the nose is diagnosed with post nasal carcinoma. Otoscopic examination reveals fluid in the middle ear cavity. Hypertrophy of which of the following structures would be most likely to compromise the drainage of the auditory tube A. Lingual tonsil B. Palatine tonsil C. Pharyngeal tonsil D. Superior constrictor muscle E. Uvula

E. A lesion of the trochlear nerve causes weakness of downward medial gaze. As a result, patients with trochlear nerve lesions commonly have difficulty walking down stairs. The superior cerebellar artery branches from the basilar artery just before it bifurcates into the posterior cerebral arteries. The trochlear nerve emerges from the dorsal aspect of the midbrain and can easily be compressed by an aneurysm of the superior cerebellar artery as it wraps around the midbrain. Aneurysms of the other arteries mentioned are not likely to compress the trochlear nerve, and lesions of the nerves listed are not likely to cause problems walking down stairs.

A 45-year-old woman is admitted to the emergency department with vision problems when she walks down stairs. During physical examination the patient exhibits weakness of her downward medial gaze. Cerebral arteriography and an MRI indicate that a nerve is being compressed by an arterial aneurysm just inferior to the tentorium cerebelli. Which of the following arteries and nerves is most likely being compressed? A. Internal carotid artery/abducens nerve B. Middle cerebral artery/oculomotor nerve C. Posterior cerebral artery/ophthalmic nerve D. Basilar artery/ophthalmic nerve E. Superior cerebellar artery/trochlear nerve

E. The sigmoid venous sinus empties into the internal jugular vein and drains the cranial vault. It runs along the posterior cranial fossa near the suture between the temporal and occipital bones just lateral to the mastoid air cells. The superior sagittal sinus lies within the superior aspect of the longitudinal fissure, between the two cerebral hemispheres. The inferior sagittal sinus runs inferior to the superior sagittal sinus within the falx cerebri and joins the great cerebral vein (of Galen) to form the straight sinus. The straight sinus drains the great cerebral vein (of Galen) into the confluence of sinuses. The cavernous sinus is located within the middle cranial fossa and receives the ophthalmic veins, the greater petrosal sinus, and other venous vessels.

A 45-year-old woman is admitted to the hospital for severe ear pain. Physical examination reveals chronic infection of the mastoid air cells (mastoiditis). The infection can erode the thin layer of the bone between the mastoid air cells and the posterior cranial fossa and spread most commonly into which of the following venous structures? A. Superior sagittal sinus B. Inferior sagittal sinus C. Straight sinus D. Cavernous sinus E. Sigmoid sinus

A. The carotid sinus is a baroreceptor that can be targeted for carotid massage to decrease blood pressure. The carotid sinus receptors are sensitive to changes in pressure. For this reason, sustained compression of the carotid sinuses can lead to unconsciousness or death as the heart rate is reflexively reduced. The carotid body is a chemoreceptor, responsive to the balance of oxygen and carbon dioxide. Neither the thyroid gland nor the parathyroid gland has anything to do with acute control of blood pressure due to mechanical stimuli. The inferior cervical ganglion fuses with the first thoracic ganglion to form the stellate ganglion. It gives rise to the inferior cervical cardiac nerve and provides postganglionic sympathetic supply to the upper limb.

A 45-year-old woman is admitted to the hospital with a severe headache. The patient is diagnosed with hypertension and arrhythmias. To reduce the patient's blood pressure, massage is initiated at a pressure point located deep to the anterior border of the sternocleidomastoid muscle at the level of the superior border of the thyroid cartilage. Which of the following structures is targeted by the massage? A. Carotid sinus B. Carotid body C. Thyroid gland D. Parathyroid gland E. Inferior cervical ganglion

C. The internal acoustic meatus is a canal in the petrous part of the temporal bone. The facial nerve (CN VII) and VIII course through this canal. The vestibulocochlear nerve (CN VIII) consists of the cochlear nerve, carrying special sensory hearing, and the vestibular nerve, carrying signals for balance and equilibrium. The facial nerve (CN VII) carries special sensory taste to the anterior one third of the tongue and motor fibers that control the muscles of facial expression.

A 51-year-old man complains of hearing loss in his left ear, poor balance, a loss of taste, and drooling from the left side of his mouth. A CT scan shows a tumor on the left side of the posterior cranial fossa. Where would the tumor be located to result in the symptoms? A. Foramen ovale B. Foramen rotundum C. Internal acoustic meatus D. Jugular foramen E. Superior orbital fissure

C. The superior salivatory nucleus is the autonomic nucleus for the facial nerve (CN VII). Parasympathetic fibers carried by the greater petrosal branch of the facial nerve are responsible for supply of the lacrimal gland and sinuses, via the pterygopalatine ganglion. The geniculate ganglion contains the cell bodies for taste from the anterior two thirds of the tongue carried by the chorda tympani branch of the facial nerve. This branch also carries the parasympathetic supply for the submandibular and sublingual salivary glands. The auriculotemporal nerve provides sensory innervation to the temporal regions of the head, the TMJ, and general sensation from the ear. The inferior salivatory nucleus provides preganglionic parasympathetic fibers carried by the glossopharyngeal nerve (CN IX) that synapse in the otic ganglion, providing parotid stimulation. The pterygopalatine ganglion includes fibers that innervate only lacrimation and the nasal sinuses, but not taste on the anterior two thirds of the tongue

A 45-year-old woman is admitted to the hospital with severe headaches, dizziness, and vomiting. Radiologic examination reveals an intracranial tumor. Upon physical examination the patient has dryness of the nasal and paranasal sinuses, loss of lacrimation, and loss of taste from the anterior two thirds of the tongue. Which of the following structures is most likely involved with the tumor? A. Auriculotemporal nerve B. Lesser petrosal nerve C. Facial nerve D. Inferior salivatory nucleus E. Pterygopalatine ganglion

C. Frey's syndrome occurs when parasympathetic axons in the auriculotemporal nerve are cut during a parotidectomy. When these postganglionic cholinergic axons grow peripherally after parotid surgery, they establish synapses upon the cholinergic sweat glands, which are innervated normally only by sympathetic fibers. As the peripheral nerves make new connections, aberrant connections can be formed between the auriculotemporal nerve and other glands (not usually innervated by the auriculotemporal nerve). This results in flushing and sweating in response to the thought, smell, or taste of food, instead of the previous, normal salivary secretion by the parotid gland.

A 45-year-old woman is admitted to the hospital with swelling on the side of her face of 2 months duration. Radiographic examination reveals a parotid gland tumor. An operative procedure is performed in which the tumor is removed. Three months postoperatively the patient complains that her face sweats profusely when she tastes or smells food, and a diagnosis is made of Frey's syndrome (gustatory sweating). Which of the following nerves was most likely injured during the procedure? A. Buccal B. Inferior alveolar C. Auriculotemporal D. Facial E. Lingual

D. The pharyngeal (Zenker's) diverticulum is usually located between the cricopharyngeal and thyropharyngeal portions of the inferior pharyngeal constrictor. This is the most common site for development of a pharyngeal diverticulum due to the inherent weakness between the pharyngeal muscles in this location. Stasis of materials within this diverticulum can lead to inflammation, infection, and abscess. This site is also known as Killian's triangle.

A 45-year-old woman visits the outpatient clinic with past history of dysphagia, nighttime fits of coughing, repeated chest infections, and a palpable swelling in her neck. Radiographic examination reveals the presence of a persistent congenital pharyngeal pouch. Between which muscles is this pouch most likely located? A. Between styloglossus and stylopharyngeus B. Between palatoglossal arch and median glossoepiglottic fold C. Between upper and middle pharyngeal constrictors D. Between the cricopharyngeal and thyropharyngeal portions of inferior pharyngeal constrictor E. Between the middle and inferior pharyngeal constrictors

B. The external branch of the superior laryngeal nerve courses together with the superior thyroid artery for much of its route. The cervical sympathetic trunk is located more laterally and quite posteriorly to this location. The inferior root of the ansa cervicalis is located more superficially in the anterior neck. The internal branch of the superior laryngeal nerve takes a route superior to that of the external branch and the superior thyroid artery and would be unlikely to be injured in this case. The recurrent laryngeal nerve terminates inferiorly, passing into the larynx in relation to the inferior thyroid artery or its branches

A 46-year-old woman is admitted to the hospital with a large mass in her lower anterior neck. Ultrasound examination suggests a benign tumor of her thyroid gland, which is confirmed with a biopsy. During the procedure to remove the tumor the superior thyroid artery is identified and used as a landmark in order not to damage its small companion nerve. Which of the following nerves is most likely to accompany the superior thyroid artery? A. Cervical sympathetic trunk B. External branch of the superior laryngeal C. Inferior root of the ansa cervicalis D. Internal branch of the superior laryngeal E. Recurrent laryngeal

B. The right abducens nerve innervates the right lateral rectus, which mediates outward movement (abduction) of the right eye. Inward movement is accomplished by the medial rectus, supplied by the oculomotor nerve. Downward movement in the midline is accomplished by joint activation of the superior oblique and inferior rectus muscle. Downward movement of the pupil from the adducted position is a function of the superior oblique alone, which is supplied by the trochlear nerve. Down and out motion is mediated by the combined actions of the lateral rectus and inferior rectus muscles, which are innervated by the abducens and oculomotor nerves. Downward movement of the pupil from a forward gaze is a result of combined actions of inferior rectus and superior oblique muscles, supplied by oculomotor and trochlear nerves, respectively.

A 47-year-old woman is admitted to the hospital with signs of cavernous sinus thrombosis. Radiographic examination reveals a pituitary tumor involving the cavernous sinus, confirming the initial diagnosis (Fig. 7-3). During physical examination it is suspected that the right abducens nerve of the patient has been damaged by the tumor. In which direction will the physician most likely ask the patient to turn her right eye to confirm the abducens nerve damage, assuming she is unable to perform this task? A. Inward B. Outward C. Downward D. Down and out E. Down and in

C. The superior orbital fissure is the opening that allows the passage of the oculomotor nerve and the trochlear nerve; the lacrimal, frontal, and nasociliary branches of ophthalmic division of the trigeminal nerve; the abducens nerve; the superior and inferior divisions of the ophthalmic vein; and the sympathetic fibers from the cavernous plexus. The sensory and motor components of the corneal reflex are the ophthalmic division of the trigeminal nerve and the oculomotor nerve, whereas the eye impairment is due to a lesion to the oculomotor nerve, all of which are transmitted through the superior orbital fissure. The inferior orbital fissure contains the maxillary division of the trigeminal nerve, infraorbital vessels, and branches of the pterygopalatine ganglion. The optic canal contains the ophthalmic artery and optic nerve, in addition to sympathetic fibers. The foramen rotundum contains the maxillary nerve. The foramen ovale contains the lesser petrosal nerve, the mandibular division of the trigeminal nerve, the accessory middle meningeal artery, and the emissary veins.

A 48-year-old man complains of diplopia (double vision). On neurologic examination he is unable to adduct his left eye and lacks a corneal reflex on the left side. Where is the most likely location of the lesion resulting in the symptoms? A. Inferior orbital fissure B. Optic canal C. Superior orbital fissure D. Foramen rotundum E. Foramen ovale

B. The superior cervical ganglion provides sympathetic innervation to the face and neck regions. Sympathetics travel along the branches of the internal carotid artery, and one result of stimulation of these nerves is to dilate the pupil during a sympathetic response ("flight or fight"). The oculomotor nerve would not affect the dilation of the pupil; rather, its stimulation results in the constriction (parasympathetic nerves). The nervus intermedius is the parasympathetic component to the facial nerve and affects only lacrimation of the eye. The Edinger-Westphal nucleus is the location of the cell bodies of the preganglionic parasympathetic neurons that are carried by the oculomotor nerve (not sympathetics). The trigeminal ganglion only provides sensory innervation to the face and eye but has no motor effect on the pupil.

A 48-year-old man presents with a constricted right pupil that does not react to light. His left pupil and vision in both eyes are normal. These findings are most likely due to a lesion involving which of the following right-sided structures? A. Oculomotor nerve B. Superior cervical ganglion C. Nervus intermedius D. Edinger-Westphal nucleus E. Trigeminal (semilunar, Gasserian) ganglion

E. The patient is suffering from an aneurysm of the anterior communicating artery. This artery connects the two anterior cerebral arteries across the beginning of the longitudinal fissure superior to the optic chiasm. An aneurysm in this area compresses the optic chiasm, resulting in blindness in the outer half of both right and left visual fields (bitemporal hemianopsia). An aneurysm of the anterior inferior cerebellar artery compresses the abducens nerve, which results in an inability to abduct the eye. Ptosis is as a result of compression to the oculomotor nerve, which can be compressed by an aneurysm in the superior cerebellar or the posterior cerebral arteries. An aneurysm of the superior cerebellar artery can also cause compression of the trochlear nerve and an inability to depress the adducted eye. Loss of corneal sensation is due to damage to the trigeminal nerve.

A 49-year-old man is seen in the emergency department because of headaches and dizziness for the past couple of months. Imaging studies reveal a saccular (berry) aneurysm (Fig. 7-26, arrow). An aneurysm at this location would most likely cause nerve compression resulting in which of the following additional findings on neurologic examination? A. Inability to abduct the eye B. Inability to depress the adducted eye C. Loss of corneal sensation D. Ptosis E. Visual field deficits

C. The glossopharyngeal nerve is located in the tonsillar bed as it runs with the stylopharyngeus muscle, which it innervates. The vagus nerve forms part of the pharyngeal plexus for supply of the pharynx and continues in the carotid sheath. The hypoglossal nerve is found inferior to the tongue. The external and internal laryngeal nerves are found in the neck.

A 49-year-old woman is admitted to the ENT department. Physical examination reveals swollen palatine tonsils and a tonsillectomy is performed. Which nerve coursing along the tonsillar bed needs to be spared during the surgery? A. Vagus B. Hypoglossal C. Glossopharyngeal D. Internal laryngeal E. External laryngeal

E. A tumor of the jugular canal would likely affect the glossopharyngeal, vagus, and accessory nerves as they exit the cranium through the jugular foramen. The uvula deviates toward the unaffected side of the pharyngeal muscles because of the pull of the unopposed levator veli palatini. In this case, the uvula deviates to the left, indicating that the left palatal muscles are unaffected whereas the right muscles are not working properly. The pharyngeal wall on the left side is also drawn upward by the nonparalyzed stylopharyngeus, supplied by the left glossopharyngeal nerve. The pharyngeal constrictor muscles, as well as muscles of the palate, are all innervated by the vagus nerve, except for the tensor veli palatini, which is supplied by the mandibular division of the trigeminal nerve. The right mandibular nerve (of the mandibular division of trigeminal nerve) provides sensory innervation to the face and motor supply to the masticatory muscles and does not innervate the muscles of the pharynx. The left hypoglossal nerve innervates the intrinsic and extrinsic muscles of the left side of the tongue. Compression or injury of this nerve would not lead to uvula deviation.

A 49-year-old woman is admitted to the hospital with headaches and dizziness. Radiologic examination reveals a tumor in the jugular canal. Upon physical examination, when the right side of the pharyngeal wall is touched with a tongue depressor, the uvula deviates to the left and the left pharyngeal wall contracts upward. When the left pharyngeal wall is touched, the response is similar. Which of the following nerves is most likely to have been injured by the tumor? A. Right glossopharyngeal B. Left glossopharyngeal C. Right mandibular D. Left hypoglossal E. Right vagus

B. The maxillary sinus drains via the middle meatus, specifically into the semilunar hiatus. The middle meatus and semilunar hiatus are located under the middle nasal concha. The inferior meatus drains the lacrimal secretions carried by the nasolacrimal duct, whereas the superior meatus drains the posterior ethmoidal and sphenoid sinuses. The nasopharynx and sphenoethmoidal recess are not situated in close proximity to the maxillary sinus and are therefore not involved in its drainage.

A 54-year-old man is admitted to the hospital with severe pain in his nasal cavity. Radiographic examination reveals a carcinoma in his nasal cavity. In which of the following locations would the carcinoma block the hiatus of the maxillary sinus? A. Inferior meatus B. Middle meatus C. Superior meatus D. Nasopharynx E. Sphenoethmoidal recess

C. The left brachiocephalic vein is the most likely vein punctured in the procedure because it extends across the trachea from the left side of the body, joining the right brachiocephalic vein to form the superior vena cava, which is located just to the right of the midline. The superior thyroid veins drain the superior aspects of the thyroid glands and join the internal jugular veins bilaterally and superiorly to the site of incision. The middle thyroid veins drain the middle portions of the thyroid glands and also terminate in the internal jugular veins laterally, superior to the incision site. The inferior thyroid veins drain the inferior aspects of the thyroid glands and descend bilaterally to the trachea to join the right and left brachiocephalic veins, respectively. Finally, the jugular arch connecting the anterior jugular veins is quite superficial and is not typically a source of concern if encountered surgically.

A 5-year-old boy fell from a tree and was admitted to the emergency department unconscious. When an emergency tracheostomy was performed, profuse dark venous bleeding suddenly occurred from the midline incision over the trachea. Which of the following vessels was most likely accidentally cut? A. Superior thyroid vein B. Inferior thyroid vein C. Left brachiocephalic vein D. Middle thyroid vein E. Jugular arch connecting the anterior jugular veins

D. The anterior inferior quadrant of the eardrum is the only portion of the tympanic membrane that would allow for an incision with minimal or no damage to adjacent important structures. Incision in the anterior and posterior superior quadrants of the eardrum would likely damage the malleus, which is situated immediately superior and medially to the tympanic membrane. The umbo is situated in close proximity to the handle of the malleus and might be damaged during incision. A vertical incision through the eardrum would almost certainly damage the malleus of the middle ear. Damage to the malleus from surgical incision would interfere with the auditory conduction through the middle ear cavity, and this should be avoided to prevent conductive hearing loss.

A 5-year-old boy is admitted to the hospital with otitis media. Otoscopic examination reveals a bulging and inflamed eardrum. It is decided to incise the tympanic membrane to relieve the painful pressure and allow drainage of the infectious fluid associated with otitis media. Which of the following is the best location to make an opening (myringotomy) for drainage? A. The anterior superior quadrant of the eardrum B. The posterior superior quadrant of the eardrum C. Directly through the site of the umbo D. The anterior inferior quadrant of the eardrum E. A vertical incision should be made in the eardrum, from the 12 o'clock position of the rim of the eardrum to the 6 o'clock position of the rim

A. The auditory (eustachian or pharyngotympanic) tube is a mucosal-lined tube that provides a direct connection from the nasopharynx to the middle ear cavity. A respiratory infection can travel from the upper respiratory tract to the oropharynx or nasopharynx and then on into the middle ear via the auditory tube. The choanae are paired openings from the nasal cavity into the nasopharynx and do not connect with the auditory tube or the middle ear. The facial canal and the internal acoustic meatus are passages for facial and vestibulocochlear nerves, respectively. Neither of these is a likely site for the spread of infection from the upper respiratory tract to the middle ear.

A 5-year-old girl is admitted to the hospital with an upper respiratory tract infection. During physical examination her sense of hearing appears to be poor. Her right ear is painful, and upon otoscopic examination a golden brown fluid can be observed through the tympanic membrane. Which is the most likely direct route for the spread of an infection from the upper respiratory tract to the middle ear cavity? A. Pharyngotympanic tube B. Choanae C. Nostrils D. Facial canal E. Internal acoustic meatus

D. The chin and lower lip area are supplied by the mental nerve, a branch of the inferior alveolar nerve, which in turn is a branch of the mandibular division of the trigeminal nerve (CN V3). The auriculotemporal nerve supplies the TMJ, the temporal region, the parotid gland, and the ear. The buccal nerve is sensory to the internal surface of the cheek. The lesser petrosal nerve is a parasympathetic nerve and would not be affected by herpes zoster, a disease of the dorsal root ganglia. The infraorbital nerve provides sensory innervation to the upper lip

A 50-year-old woman complained of pain over her chin and lower lip. A few days later, small vesicles appeared over the same area and soon began erupting. She was diagnosed with a dermatomal herpes zoster inflammation (shingles). Which of the following nerves was most likely to contain the virus in this case? A. Auriculotemporal B. Buccal C. Lesser petrosal D. Mental E. Infraorbital

E. The submental nodes are located between the anterior bellies of the digastric muscles. The central portions of the lower lip and floor of the mouth and the apex of the tongue drain into these nodes. Efferent lymphatic from these nodes go to the submandibular lymph nodes and partly to a gland of the deep cervical group of nodes. The occipital nodes drain the occipital region of the scalp. The parotid nodes drain the root of the nose, the eyelids, the frontotemporal region, and the external acoustic meatus, and a deep group drains the nasal part of the pharynx and the posterior parts of the nasal cavities. The retropharyngeal nodes are found in the buccopharyngeal fascia and drain the nasal cavities, the nasal part of the pharynx, and the auditory tubes. The jugulodigastric nodes drain primarily the tonsils.

A 50-year-old woman with a history of cigarette smoking and alcohol abuse presents to the clinic with a lower lip lesion. Biopsy of her lower lip reveals a squamous cell carcinoma. Which lymph nodes will most likely be first affected by the spread of the tumor cells? A. Occipital B. Parotid C. Retropharyngeal D. Jugulodigastric E. Submental

E. The lingual nerve supplies sensory innervation to the mucous membrane of the anterior two thirds of the tongue, taste sensation to the anterior part of the tongue, and parasympathetic fibers to the oral salivary glands. The chorda tympani branch of the facial nerve is responsible for carrying taste fibers from the anterior two thirds of the tongue and preganglionic parasympathetic fibers for the submandibular ganglion. Injury to the lingual nerve at its origin, before it joins with the chorda tympani, will result in loss of general sensation of the tongue, but with preservation of taste and salivary function. Injury to the glossopharyngeal nerve would result in loss of general sensory and taste fibers from the posterior third of the tongue and parasympathetic supply for the parotid gland. Injury to the superior laryngeal nerve, a branch of the vagus nerve, will result in loss of sensation from the larynx above the vocal folds.

A 52-year-old man is admitted to the emergency department with a bullet wound in the infratemporal fossa. During physical examination it is observed that the patient has lost unilateral sensation of hot, cold, pain, and pressure from the front part of the tongue, but taste and salivary function are preserved. Which of the following is the most likely diagnosis? A. The facial nerve was transected distal to the origin of the chorda tympani B. Receptors for hot, cold, pain, and pressure are absent in the patient's tongue C. The glossopharyngeal nerve has been injured in the pharynx D. The superior laryngeal nerve was obviously severed by the bullet E. The lingual nerve was injured at its origin near the foramen ovale

E. The location of the growing lump is close to the external acoustic meatus and is likely compressing on the facial nerve as it exits the stylomastoid foramen. The facial nerve is responsible for tight closure of the eyelids. Protraction of the tongue is due to innervation of the hypoglossal nerve and elevation of the mandible by the mandibular nerve. Pupillary constriction and partial ptosis result from damage to the oculomotor nerve.

A 52-year-old man with a growing lump on the side of his face and neck for the past several months visits his doctor because of the recent development of pain along with other new neurologic symptoms. Based on the mass in the CT image (Fig. 7-22), which of the following neurologic abnormalities will most likely be present on examination as a direct result of the growing mass? A. Contralateral deviation of the uvula during elevation of the soft palate B. Ipsilateral deviation on protraction of the tongue C. Ipsilateral pupillary constriction and partial ptosis D. Ipsilateral weakness in elevation of the mandible E. Ipsilateral weakness in tight closure of the eyelids

D. It is necessary to anesthetize the conjunctival covering of the sclera, which is supplied by the nasociliary branch of the ophthalmic division of the trigeminal nerve. To do that, the needle should be placed through the upper eyelid deeply toward the orbital apex to infiltrate the nasociliary nerve, and also between the orbital septum and the palpebral musculature laterally to anesthetize lateral sensory supply from the lacrimal nerve and (perhaps) twigs from the maxillary division of the trigeminal nerve. The lacrimal fossa, which is occupied by the lacrimal sac portion of the nasolacrimal duct, is too medial, whereas the supraorbital foramen is above the eye. Injections into either location would not result in anesthetizing of the sclera. Answers A and E both result in puncturing of the sclera and would most likely cause further damage to the eye.

A 54-year-old man is admitted to the emergency department with a fracture at the frontozygomatic suture suffered in an automobile collision. During physical examination the upper eyelid of the patient exhibits multiple lacerations and the sclera contains small fragments from his broken glasses. What site would be preferable for needle insertion to anesthetize the orbital contents and then the area of the eyelid injury? A. Into the sclera in the limbic region and also into the infraorbital foramen B. Into the lacrimal fossa and also beneath the lateral bulbar conjunctiva C. Into the supraorbital foramen and also into the lacrimal caruncle D. Through the upper eyelid deeply toward the orbital apex and also between the orbital septum and the palpebral musculature laterally E. Directly posteriorly through the anulus tendineus and superior orbital fissure

A. The abducens nerve would be affected first due to aneurysmal dilation of the internal carotid artery (ICA) because the nerve runs in closest proximity to the artery within the cavernous sinus. The other nerves running in the wall of the cavernous sinus are the oculomotor nerve, trochlear nerve, and both the maxillary and ophthalmic branches of the trigeminal nerve. Each of these nerves, however, courses along, or within, the lateral walls of the cavernous sinus and may not be immediately affected by an aneurysm of the ICA.

A 54-year-old man is admitted to the hospital due to severe headaches. A CT examination reveals an internal carotid artery aneurysm inside the cavernous sinus. Which of the following nerves would be typically affected first? A. Abducens nerve B. Oculomotor nerve C. Ophthalmic nerve D. Maxillary nerve E. Trochlear nerve

C. A tumor involving the meningeal branches of the ethmoidal nerves that originate from the ophthalmic division of the trigeminal nerve is likely to cause pain from pressure and nerve injury in the anterior cranial fossa. The maxillary and mandibular divisions of the trigeminal nerve provide sensory innervation to the middle and posterior aspects of the meninges, respectively. Spinal nerve C2 and C3 fibers do not provide meningeal innervation. The tentorial nerve, a branch of the ophthalmic division of the trigeminal nerve, supplies the tentorium and the supratentorial falx cerebri.

A 54-year-old man is admitted to the hospital with severe headaches. A CT scan reveals a tumor in his brain occupying a portion of the anterior cranial fossa. Which of the following is responsible for the sensation of pain from headache in this case? A. Meningeal branches of the maxillary nerve B. Meningeal branches of the mandibular nerve C. Meningeal branches of the ethmoidal nerve D. Tentorial nerve E. C2 and C3 fibers

E. The sphenoidal sinus provides the most direct access to the pituitary gland, which is situated directly above this sinus. Neither the frontal sinus nor maxillary sinus has any direct communication with the interior of the cranial vault and would therefore not allow the surgeon a potential access point to the pituitary gland. The cribriform plate could offer a point of entry into the cranium; entry at that site would lead to damage of the olfactory cells and nerve, but it would also lead to entry into the subarachnoid space, with leakage of CSF and potential meningitis. The cribriform plates are also located too far anteriorly from the pituitary gland. The cavernous sinus is situated within the cranial vault and surrounds the pituitary gland; it is not a site for surgical entrance to the cranial cavity.

A 54-year-old man is diagnosed with an aneurysm of the basilar artery close to the cavernous sinus. An anterior approach to the sella turcica through the nasal cavity is performed. Through which of the following routes is the surgeon most likely to gain access to the cranial cavity? A. Cribriform plate B. Cavernous sinus C. Frontal sinus D. Maxillary sinus E. Sphenoidal sinus

B. The posterior cricoarytenoid muscle is the only abductor of the larynx that opens the rima glottidis and rotates the arytenoid cartilages laterally. All of the other listed muscles have adduction as part of their function and thus are not required to maintain the airway.

A 54-year-old man is to undergo bilateral thyroidectomy. During this procedure there is the possibility of bilateral paralysis of muscles that can open the airway. If a particular nerve is injured bilaterally, there is significant risk of asphyxiation postoperatively unless the patient is intubated or the airway is opened surgically. Which of the following muscle pairs opens the airway? A. Cricothyroids B. Posterior cricoarytenoids C. Arytenoids D. Thyroarytenoids E. Lateral cricoarytenoids

C. The superior sagittal sinus would most likely be the source of the bleeding because it attaches anteriorly to the crista galli and because of the slow nature of the bleed. The middle meningeal artery would not be a good answer because its location is near the pterion on the temporal aspect of the skull but its bleeding would be profuse, not slow. The great cerebral vein (of Galen) is located posteriorly in the cranial cavity and is not in the right location for an injury of this type to disrupt it. The straight dural venous sinus is also posterior, receiving the draining of the inferior sagittal sinus and the great vein (of Galen). It drains posteriorly to the confluence of sinuses (eponym: Torcular of Herophilis). The superior ophthalmic vein drains from the orbit to the cavernous sinus; further, it is located inferiorly to the crista galli and is not directly related to the superior sagittal sinus.

A 54-year-old man was admitted to the emergency department after he was struck by an automobile. Radiographic examination revealed a fracture through the crista galli of the anterior cranial fossa, resulting in slow, local bleeding. Which of the following is the most likely source of bleeding? A. Middle meningeal artery B. The great cerebral vein of Galen C. Superior sagittal sinus D. Straight sinus E. Superior ophthalmic vein

B. The anterior communicating artery, the portion of the arterial circle (of Willis), is directly superior to the optic chiasm, and an aneurysm of this artery would likely compress the chiasm, as in this patient.

A 54-year-old woman is admitted to the emergency department after experiencing sudden problems with vision over the past 5 days. MRI reveals that an aneurysm of one of the arteries at the base of the brain is compressing the optic chiasm. Which of the following arteries will most likely be involved? A. Middle cerebral B. Anterior communicating C. Anterior cerebral D. Superior cerebellar E. Posterior superior cerebellar

A. CSF is mostly secreted from the choroid plexuses of the lateral, third, and fourth ventricles of the brain. The CSF enters the subarachnoid space from the fourth ventricle, via the foramina of Luschka and Magendie. The CSF then circulates in the subarachnoid space until it is finally resorbed back into the venous side of the circulation through the arachnoid granulations into the superior sagittal sinus. A thrombus of the superior sagittal sinus can to lead to an obstruction of CSF (communicating hydrocephalus) in which all of the ventricles of the brain are enlarged and the intracranial pressure is increased.

A 55-year-old male farmer is admitted to the emergency department after falling from the hayloft in his barn. Radiographic examination reveals a small, depressed fracture of the skull vertex and thrombosis of the superior sagittal sinus. A day later the patient loses consciousness. What is the most likely cause of his loss of consciousness? A. Obstruction of CSF resorption B. Obstruction of the cerebral aqueduct (of Sylvius) C. Laceration of the middle meningeal artery D. Fracture of the cribriform plate with CSF rhinorrhea E. Aneurysm of the middle cerebral artery

D. An acoustic neuroma is an intracranial tumor that arises from the Schwann cell sheath investing the vestibulocochlear nerve. As this tumor grows, it eventually occupies a large portion of the cerebellopontine angle. Since cranial nerves VII and V are in close proximity to this location, these nerves are also usually affected, with subsequent manifestation of symptoms of impaired hearing, vertigo, loss of balance and nystagmus, paralysis of muscles of facial expression, hyperacusis, loss of taste sensation on the anterior two thirds of the tongue, loss of corneal reflex and sensation around the mouth and nose, and paralysis of muscles of mastication.

A 55-year-old man complains of progressive hearing loss and a continuous strange noise in his right ear that he noticed several months ago. On physical examination he has right-sided hearing loss, asymmetric smile, and decreased corneal reflex in his right eye. A CT scan reveals a large intracranial tumor. Which of the following will most likely be the location of the tumor? A. Between the medulla and the cerebellar hemisphere B. Above the diaphragma sellae C. Over the lateral hemispheric fissure D. Between the cerebellum and the lateral pons E. Between the cerebellar peduncles

D. Maxillary sinusitis is an infection of the maxillary sinus, which is located in the body of the maxillary bone. Sharp pain can be a major symptom of maxillary sinusitis. The difference between the remaining answer choices is the location of the sinus. The sphenoid sinus is located in the posterior nasopharynx. The ethmoidal sinuses are located laterosuperiorly to the nasal septum. The frontal sinus is located in the frontal bone in the anterior part of the face.

A 55-year-old man has a complaint of left-sided tooth pain in his upper jaw. A dental examination reveals no abnormalities of his teeth. During physical examination tapping on his left maxilla elicits sharp pain on the left side of his face, including his maxillary teeth on that side. The patient reports that he has no allergies. Which of the following conditions will be the most likely diagnosis? A. Sphenoid sinusitis B. Anterior ethmoidal sinusitis C. Posterior ethmoidal sinusitis D. Maxillary sinusitis E. Frontal sinusitis

A. The loose areolar connective tissue layer is known as the "danger zone" because hematoma can spread easily from this layer into the skull by means of emissary veins that pass into and through the bones of the skull. None of the other scalp layers listed is referred to as the "danger zone."

A 55-year-old man is admitted to the emergency department after slipping on wet pavement and falling. Physical examination reveals that the patient has a hematoma that formed in the danger zone of the scalp, spreading to the area of the eyelids. Which of the following layers is regarded as the "danger zone"? A. Loose, areolar layer B. Skin C. Galea aponeurotica D. Pericranium E. Subcutaneous layer

A. The right and left recurrent laryngeal nerves loop around the right subclavian artery and the arch of the aorta, respectively. These nerves then travel superiorly in the tracheoesophageal groove to the larynx. Damage to the recurrent laryngeal nerve as a result of surgical intervention or the presence of a tumor in the tracheoesophageal groove would render the patient hoarse. This hoarseness is due to a lack of innervation by the recurrent laryngeal nerve to most of the muscles of the larynx. Damage to the internal laryngeal nerve would cause a loss of sensation above the vocal cords, in addition to a loss of taste on the epiglottis. Damage to the external laryngeal nerve, which can occur during thyroidectomy, will result in a loss of innervation to the cricothyroid muscle, with resultant vocal weakness. Patients with this lesion will often present with a fatigued voice. The vagus nerve gives rise to the recurrent laryngeal nerves; damage to this nerve, however, would result in numerous symptoms beyond just hoarseness.

A 55-year-old man is admitted to the emergency department with a complaint of pain when chewing over the previous 3 months. Physical examination reveals the patient suffers from odynophagia and hoarseness in his speech. Radiographic examination reveals a tumor at the tracheoesophageal groove. Which of the following nerves is most likely affected by the tumor? A. Recurrent laryngeal B. Internal laryngeal C. Vagus D. External laryngeal E. Phrenic

A. The retropharyngeal space extends from the inferior aspect of the skull to the posterior mediastinum behind the esophagus. An infection or abscess in this space could thus travel toward the posterior mediastinum. The retropharyngeal space is enclosed between the visceral fascia covering the posterior wall of the pharynx and the alar layer of the prevertebral fascia. The alar fascia is formed from bilateral anterior extensions of the prevertebral fascia. Between the alar fascia and the more posterior prevertebral fascia covering the skeletal musculature is the so-called danger space of the neck. This space is continuous superiorly to the base of the skull and continues inferiorly through the posterior mediastinum to the level of the respiratory diaphragm. The alar fascia is continuous with the carotid sheath and provides the posterior boundary for the retropharyngeal space. Attachments of the alar fascia to the retropharyngeal fascia result in separation of the pretracheal space from the retropharyngeal space. The prevertebral fascia invests the vertebral column and the intrinsic muscles of the back. The pretracheal fascia encloses the trachea and larynx, whereas the buccopharyngeal fascia invests the superior pharyngeal constrictor and buccinator muscles.

A 55-year-old man is admitted to the emergency department with fever of 4 days' duration. Radiographic examination reveals the presence of an infection that is spreading from the retropharyngeal space to the posterior mediastinum. Between which of the following fascial layers is the infection most likely located? A. Between alar and prevertebral B. Between alar and pretracheal C. Between pretracheal and prevertebral D. Between buccopharyngeal and alar E. Between buccopharyngeal and prevertebral

B. When a berry aneurysm ruptures, the blood flows into the subarachnoid space and therefore mixes with CSF; thus, blood would be present in the CSF when a lumbar puncture is performed. The pterion overlies the anterior branch of the middle meningeal vessels, and damage to these vessels would result in an epidural hematoma, with compression of the brain. Leakage of branches of the middle meningeal artery within the temporal bone would cause blood vessels within the bone to leak, without direct connection to the CSF fluid. A tear of the cerebral vein in the superior sagittal sinus would lead to a subdural hematoma, in which the blood collects in the subdural space, without entry to CSF. The occlusion of the internal carotid artery by way of clot would not lead to leakage of blood into the CSF.

A 55-year-old man is admitted to the hospital with a complaint of severe headaches. A lumbar puncture reveals traces of blood in the CSF. Which of the following conditions has most likely occurred in this patient? A. Fracture of the pterion with vascular injury B. A ruptured "berry" aneurysm C. Leakage of branches of the middle meningeal vein within the temporal bone D. A tear of the cerebral vein at the entrance to the superior sagittal sinus E. Occlusion of the internal carotid artery by a clot generated in the left atrium

B. The auriculotemporal nerve, a branch of the mandibular division of the trigeminal nerve (CV V3) leads into the parotid gland, and its compression in mumps can be associated with severe pain. The compressive effects are due in large part to the continuity of the facial capsule of the parotid gland with the tough layer of superficial investing fascia of the neck, a layer that is almost nondistensible. When the gland swells, sensory fibers for pain are triggered rapidly, and can be referred to the ear. None of the other nerves listed supply the parotid gland.

A 6-year-old boy is admitted to the hospital with high fever and pain over the parotid gland (Fig. 7-7). A diagnosis of parotiditis (mumps) is established, and the boy is sent back home. Which of the following nerves is responsible for painful sensations from the region of the parotid gland? A. Facial B. Auriculotemporal C. Lesser petrosal D. Lingual E. Chorda tympani

D. The nucleus ambiguus gives rise to efferent motor fibers of the vagus nerve, which supply the laryngeal and pharyngeal muscles. If supply to this region is interrupted, an individual loses the swallowing, cough, and gag reflexes. The nucleus solitarius is located in the brainstem and is responsible for receiving general visceral sensation and taste from the facial, glossopharyngeal, and vagus nerves. The trigeminal motor nucleus contains motor neurons that innervate muscles of mastication, the tensor tympani, tensor veli palatini, mylohyoid, and anterior belly of the digastric. The dorsal motor nucleus contains the cell bodies of preganglionic parasympathetic fibers of the vagus nerve innervating the heart muscle and smooth musculature and glands of the respiratory and intestinal tract. The superior ganglion of the vagus contains cell bodies of general somatic afferent fibers, and the inferior ganglion of the vagus is chiefly visceral afferent in function concerning sensations (with the exclusion of painful sensation) from the heart, lungs, larynx, and alimentary tract.

A 55-year-old man is admitted to the neurosurgical clinic for a scheduled removal of a tumor in the left jugular canal. Postoperatively, the patient has no gag reflex when the ipsilateral pharyngeal wall is stimulated, although the pharynx moved upward and a gag reflex resulted when the right pharyngeal wall was stimulated. The uvula was deviated to the right and the left vocal cord had drifted toward the midline. Which of the following structures will contain the neural cell bodies for the motor supply of the paralyzed muscles? A. Nucleus solitarius B. Trigeminal motor nucleus C. Dorsal motor nucleus D. Nucleus ambiguus E. Superior or inferior ganglia of vagus

D. Branches of the sphenopalatine, superior labial, and anterior ethmoidal anastomose anteriorly on the nasal septum are prone to bleeding. The descending palatine and ascending pharyngeal arteries are located posteriorly in the pharynx. The posterior superior alveolar artery travels within the maxilla and the accessory meningeal artery is found within the cranial cavity.

A 55-year-old man with a history of cocaine abuse is brought to the emergency department because of severe uncontrolled epistaxis. Flexible nasal endoscopy reveals the source of bleeding to be in Kiesselbach's area. Anastomotic connections between which arteries are involved in this condition? A. Descending palatine and ascending pharyngeal B. Posterior superior alveolar and accessory meningeal C. Lateral branches of posterior ethmoidal and middle meningeal D. Branches of sphenopalatine, superior labial, and anterior ethmoidal E. Descending palatine and tonsillar branches of the ascending pharyngeal

A. The inner surface of the tympanic membrane is supplied by the glossopharyngeal nerve. The auricular branches of the facial and vagus nerves and the auriculotemporal branch of the trigeminal nerve innervate the external surface of the tympanic membrane. The great auricular nerve arises from C2 and C3 and supplies the posterior auricle and skin over the parotid gland. The lingual nerve does not have anything to do with sensory supply of the tympanic membrane.

A 55-year-old man with severe ear pain visits an otorhinolaryngologist because his tympanic membrane has been ruptured by a foreign object. Which of the following nerves is responsible for the sensory innervation of the inner surface of the tympanic membrane? A. Glossopharyngeal B. Auricular branch of facial C. Auricular branch of vagus D. Great auricular E. Lingual

A. The anterior division of the facial nerve passes through the parotid gland and is therefore at risk during surgery of the parotid gland. Since this patient's symptoms involved paralysis of the muscles of the lower lip, the branch of the facial nerve that supplies these muscles, the marginal mandibular branch, is the one that has suffered the iatrogenic injury

A 55-year-old woman has undergone facial surgery for the excision of a malignant parotid tumor. Postoperatively, marked weakness is seen in the musculature of the patient's lower lip. Which of the following nerves was most likely injured during the parotidectomy? A. Marginal mandibular branch of facial B. Zygomatic branch of facial C. Mandibular division of the trigeminal nerve D. Buccal branch of facial E. Buccal nerve

A. The deep lingual vein is located most superficially on the underside of the tongue. It is therefore the most direct route for absorption of the administered nitroglycerin. The submandibular and sublingual ducts are excretory in function and do not function to absorb a drug, such as nitroglycerin. The lingual and sublingual vein are located more deeply within the floor of the mouth and do not provide the most direct route for absorption.

A 55-year-old woman is admitted to the emergency department with chest pain (angina). An electrocardiogram (ECG) examination reveals an acute myocardial infarction (heart attack). A series of medications is administered to the patient, including sublingual nitroglycerin for reducing her blood pressure. Which of the following structures is most likely to be the route of absorption of this drug? A. Deep lingual vein B. Submandibular duct C. Sublingual duct D. Lingual vein E. Sublingual vein

E. The vagus nerve exits the skull at the jugular foramen and is responsible for motor innervation to the smooth muscles of the trachea, bronchi, and digestive tract, in addition to the muscles of the palate, pharynx, larynx, and superior two thirds of the esophagus. The ansa cervicalis innervates the strap muscles of the neck, with the exception of the thyrohyoid muscle. The cervical sympathetic trunk does not enter into the jugular foramen; it runs behind the carotid sheath, parallel with the internal carotid artery; its carotid branch accompanies the artery into the carotid canal and carries sympathetic fibers to deep areas of the head. Damage to the external laryngeal nerve would result in paralysis of the cricothyroid muscle, presenting as an easily fatigued voice with hoarseness. Injury to the hypoglossal nerve would result in protrusion of the tongue toward the affected side and moderate dysarthria.

A 55-year-old woman is admitted to the hospital with difficulty swallowing. Physical examination reveals that the patient has episodes of severe headaches and frequently aspirates fluids when drinking them. A radiographic examination reveals a skull base tumor occupying the space behind the jugular foramen. Involvement of which of the following structures is most likely responsible for the findings in the patient? A. Ansa cervicalis B. Cervical sympathetic trunk C. External laryngeal nerve D. Hypoglossal nerve E. Vagus nerve

B. The hypoglossal nerve provides motor innervation to the muscles of the tongue, with the exception of the palatoglossus. Injury to the hypoglossal nerve would result in deviation of the tongue toward the affected side when the tongue is protruded (in this case the right side), due mainly to the unilateral contraction of left genioglossus, and moderate dysarthria. Injury to the glossopharyngeal nerve would result in loss of taste in the posterior third of the tongue and a loss of soft palate sensation and gag reflex on the affected side. The inferior alveolar nerve supplies the tissues of the chin and lower teeth. The lingual nerve conveys parasympathetic preganglionic fibers to the submandibular ganglion and general sensation and taste fibers for the anterior two thirds of the tongue. Injury to the vagus nerve would cause sagging of the soft palate, deviation of the uvula to the unaffected side, hoarseness, and difficulty in swallowing and speaking.

A 55-year-old woman is admitted to the hospital with severe hypertension. Examination reveals hypertension (190/110 mm Hg) and hypercholesterolemia (250 mg/dL). During physical examination she complains of headaches and dizziness. An arteriogram reveals 90% occlusion of both common carotid arteries. A carotid endarterectomy is performed and large atherosclerotic plaques are removed. During a postoperative physical examination on the right side, it was noted that her tongue deviated toward the right when she was asked to protrude it. Which of the following nerves was most likely injured during the procedure? A. Right glossopharyngeal B. Right hypoglossal C. Left hypoglossal D. Left lingual E. Left vagus

D. The basilar artery is formed by the two vertebral arteries at the inferior part of the brainstem. At the medullopontine junction, the basilar artery gives two branches: the anterior inferior cerebellar artery first and then the labyrinthine artery. The abducens nerve (cranial nerve VI) emerges at the medullopontine junction and is usually found between the anterior inferior cerebellar and the labyrinthine arteries. The abducens nerve supplies the lateral rectus muscle, which abducts the eyes. This function of the nerve can be impaired if the nerve is compressed by a nearby aneurysm. If this condition occurs on the right side, it can result in weak abduction of the right eye.

A 55-year-old woman presented to the emergency department with complaints of severe headaches for 2 weeks. There was no history of fever, neck stiffness, or photophobia. On examination, she was in pain and distress. Vital signs revealed blood pressure (BP) 160/89 mm Hg, pulse 110 bpm, respiration rate 16/ min, temperature 36° C. Range of movement of the neck was normal, and funduscopy showed no abnormalities on the retina. A CT scan of the brain with contrast was performed and revealed a dilated branch of the basilar artery at the junction of the pons and medulla on the right side. She was referred to the neurosurgeon for assessment and further management. Which of the following statements describes clinical features with which this patient might present? A. Weak abduction and depression of the right eye B. Weak adduction of the left eye C. Weak adduction and elevation of the right eye D. Weak abduction of the right eye E. Weak abduction of the left eye

B. Hydrops (edema) results from accumulation of excessive fluid in the endolymphatic sac. Labyrinthine hydrops or endolymphatic hydrops is known as Ménière's disease. This disease can result in hearing loss, roaring noises in the ear, and episodic dizziness (vertigo) associated with nausea and vomiting. About 10% of patients require surgical intervention for persistent, incapacitating vertigo; others are treated with diuretics, low salt intake, and reduction of stimulants like caffeine to lower the volume of body fluids and alleviate the symptoms of Ménière's disease.

A 55-year-old woman presents to the ENT clinic with episodes of vertigo, fluctuating hearing loss, ringing of the right ear, and feeling or fullness or pressure in her ear. The clinical diagnosis suggests Ménière's disease. Which of the following structures is most likely affected by the edema associated with Ménière's disease? A. Middle ear B. Endolymphatic sac C. Semicircular canals D. Cochlea E. Helicotrema

C. The glossopharyngeal nerve (CN IX) enters the posterior oropharynx by coursing between the stylohyoid ligament and the stylopharyngeus muscle. Calcification of the stylohyoid ligament can readily affect this nerve by irritation or compression. The other nerves listed are not in close proximity to the styloid process or stylohyoid ligament. The glossopharyngeal nerve carries sensory nerve fibers from the posterior third of the tongue and the pharynx. A lesion of this nerve could cause loss of both general sensation and taste sensation from the posterior third of the tongue.

A 55-year-old woman visits the outpatient clinic complaining of unilateral neck pain, sore throat, and ear pain. Radiographic examination reveals Eagle's syndrome, in which the styloid process and stylohyoid ligament are elongated and calcified. Which of the following nerves is most likely affected by Eagle's syndrome in this patient? A. Vagus B. Facial C. Glossopharyngeal D. Hypoglossal E. Vestibulocochlear

D. The superior thyroid vein is a tributary to the internal jugular vein; it accompanies the superior thyroid artery. The middle thyroid vein is typically a short, direct tributary to the internal jugular vein. The inferior thyroid vein usually drains vertically downward to one or both brachiocephalic veins. The superior and middle thyroid veins can be torn in thyroid surgery, perhaps admitting an air bubble (due to negative pressure in the veins) that can ascend in the internal jugular vein into the skull, with deleterious or lethal results.

A 55-year-old woman visits the outpatient clinic with swelling in her neck. Ultrasound examinations reveal a thyroid gland tumor. Three days after thyroidectomy air bubbles are seen on the CT of her brain. Which of the following is the most likely cause of the air bubbles? A. Injury to inferior thyroid artery B. Injury to inferior and superior thyroid arteries C. Injury to superior thyroid artery and vein D. Injury to superior and middle thyroid veins E. Injury to superior, middle, and inferior thyroid veins

C. Because of its size and vulnerable position during birth, the sternocleidomastoid muscle is injured more often than other muscles of the head and neck during birth especially if the delivery is difficult. When acting alone, the action of this muscle is to turn the head to the opposite side and bend it toward the ipsilateral shoulder. When using both muscles, the head will flex toward the chest. Therefore, the most likely muscle to have been injured here is the left sternocleidomastoid muscle

A 6-year-old boy, whose medical history includes a complicated delivery, has a continuously tilted head posture, with the right ear near the right shoulder and the face turned upward and to the left. Which of the following muscles was most likely damaged during birth? A. Anterior scalene B. Omohyoid C. Sternocleidomastoid D. Trapezius E. Platysma

A. The hypoglossal nerve innervates the muscles of the tongue and is therefore directly involved in alteration of shape and movement of the tongue. A lesion in this nerve would cause deviation of the tongue toward the injured side, which could be observed upon protrusion of the tongue. The genioglossus is the major muscle involved in protrusion of the tongue. The genioglossus muscles arise from the inside of the mandible and pass posteriorly to insert into the deep aspect of the tongue. When the genioglossus muscles contract, they pull the tongue forward, and out of the mouth, in protrusion. If one genioglossus is paralyzed, it acts like a brake on one side of the tongue when the tongue is pulled forward, causing the tip of the tongue to point to the nonmoving side. The styloglossus muscle is responsible for retraction and elevation of the tongue.

A 56-year-old man is diagnosed with an extradural tumor in the posterior cranial fossa. When the patient protruded his tongue during physical examination, the tongue deviated to the right. Which of the following muscles and nerves are most likely injured? A. Right hypoglossal nerve and right genioglossus B. Left hypoglossal nerve and left genioglossus C. Right hyoglossus and left styloglossus D. Right geniohyoid and first cervical nerve E. Contralateral vagus and hypoglossal nerves

D. The sigmoid sinus collects venous blood from the transverse sinuses and empties it into a small cavity known as the jugular bulb, the inferior portion of which is located beneath the bony floor of the middle ear cavity. A paraganglioma is a tumor that may originate from paraganglia cells found in the middle ear and on the jugular bulb. Tumors that originate from the jugular bulb can grow to fill the entire bulb and may effectively block blood returning to the heart from that side of the brain. Blood flow from the brain is gradually diverted toward the opposite sigmoid sinus and jugular bulb, causing the opposite venous system to expand and accommodate increased blood flow. The cochlea and lateral semicircular canals are located in the inner ear and are not directly affected by such a tumor. The internal carotid artery is related to the anterior wall of the middle ear cavity and is not likely to be affected by a tumor penetrating the middle ear. The sigmoid venous sinus collects venous blood beneath the temporal bone and follows a tortuous (S-shaped) course to the jugular foramen where it becomes continuous with the internal jugular vein at the jugular bulb. The aditus ad antrum is the entrance to the mastoid antrum, which is the common cavity in the mastoid bone into which mastoid air cells open. Below the aditus ad antrum is an elevation of bone, the pyramid of the stapes, which is occupied by the stapedius muscle.

A 56-year-old man visits the outpatient clinic with a complaint of severe headaches and ear pain. Radiologic examination reveals a tumor in the middle ear cavity, invading through the bony floor. Which of the following structures will most likely be affected? A. The cochlea and lateral semicircular canal B. The internal carotid artery C. The sigmoid venous sinus D. The internal jugular bulb E. The aditus ad antrum of the mastoid region and the facial nerve

D. The trochlear nerve innervates the superior oblique muscle, which acts to move the pupil downward and laterally. It is the only muscle that can depress the pupil when the eye is adducted. When an individual walks down stairs, this eye motion is initiated, and diplopia results if it is not functioning properly. The optic nerve provides vision, and a lesion of this nerve would not result in diplopia when an affected individual walks down the stairs, but rather diminished vision or blindness. The oculomotor nerve supplies the superior, inferior, and medial rectus as well as the inferior oblique. Overall, innervation from the oculomotor nerve results in upward and inward movements of the eye, and a lesion of this nerve would not induce diplopia in an individual walking down stairs. The abducens nerve innervates the lateral rectus muscle, which abducts the eye, and damage would not induce the diplopia presented in this problem. The frontal nerve is a branch of the

A 56-year-old woman complains of diplopia (double vision) when walking down stairs. A lesion of which of the following nerves is most likely responsible for this patient's complaint? A. Optic B. Oculomotor C. Abducens D. Trochlear E. Frontal

B. The recurrent laryngeal nerve is often at risk of being damaged during a thyroidectomy. Patients who have a transected or damaged recurrent laryngeal will often present with a characteristic hoarseness following surgery. The posterior cricoarytenoid is supplied by the recurrent laryngeal nerve and would thus be impaired following damage to the nerve. The posterior cricoarytenoid is the only muscle responsible for abduction of the vocal cords, and paralysis of this muscle would result in a permanently adducted position of the involved vocal cord. The other muscles listed are all adductors of the vocal cords, and paralysis of these would not lead to closure of the airway.

A 56-year-old woman has just undergone a complete thyroidectomy. After she recovers from the anesthesia a hoarseness of her voice is noted that persists for 3 weeks. Subsequent examination shows a permanently adducted vocal fold on the right side. Surgical trauma to the innervation of which of the following muscles is most likely to be responsible for the position of the right vocal fold? A. Aryepiglottic B. Posterior cricoarytenoid C. Thyroarytenoid D. Transverse arytenoids E. Vocalis

A. The dural covering of the optic nerve is connected to the anular tendon which serves as the origin of the rectus muscles; therefore, when there is an inflammation of the optic nerve, contractions of the rectus muscles can evoke severe pain.

A 56-year-old woman is admitted to the hospital with eye pain. During physical examination the patient complains of excruciating pain when she performs any movement of the eye. An MRI examination reveals that the optic nerve is inflamed. What is the most likely explanation? A. The anular tendon (of Zinn) is inflamed B. The inflammation has affected the nerves innervating the eye muscles C. The muscles are contracting due to generalized inflammation D. The nasociliary nerve is affected E. The ophthalmic artery is constricted

E. The submental lymph nodes drain approximately the anterior two thirds of the mouth and tongue, including the lower lips. The occipital nodes serve the inferoposterior aspect of the head. The parotid nodes lie anterior to the ear and serve the region of the lateral aspect of the eye, the parotid gland, and anterior ear. The retropharyngeal nodes lie posterior to the pharynx and drain the posterior aspect of the throat and pharynx. The jugulodigastric node is a large node posterior to the parotid gland and just below the angle of the mandible, and it receives lymph from much of the face and scalp and is commonly enlarged in tonsillitis.

A 70-year-old man has a biopsy of a growth on his lower lip. The biopsy reveals a squamous cell carcinoma. Which lymph nodes will most likely be first involved in the spread of the cancer cells? A. Occipital B. Parotid C. Retropharyngeal D. Jugulodigastric E. Submental

B. The auriculotemporal nerve is a posterior branch of the mandibular division of the trigeminal nerve. It encircles the middle meningeal artery and courses medially to the TMJ and then ascends up near the auricle. Because this nerve supplies the TMJ and skin of the external auditory canal, pain from the joint can be referred to the ear as in this case. The facial nerve courses over the ascending ramus of the mandible, passing superficial to the masseter muscle and below the TMJ through the parotid gland, and would not be involved in this problem. The lesser petrosal nerve courses through the middle cranial fossa and exits through the foramen ovale, where it joins the otic ganglion. The vestibulocochlear nerve exits the cranial cavity through the internal acoustic meatus and innervates structures in the inner ear. Finally, the chorda tympani is a branch of the facial nerve and joins the mandibular division of the trigeminal nerve anterior to the TMJ.

A 56-year-old woman is admitted to the hospital with rheumatoid arthritis of her temporomandibular joint (TMJ) and severe ear pain. An image from her radiographic examination is shown in Fig. 7-2. Which of the following nerves is most likely responsible for conducting the pain sensation? A. Facial B. Auriculotemporal C. Lesser petrosal D. Vestibulocochlear E. Chorda tympani

B. A lesion of the oculomotor nerve will cause the eye to remain in a "down and out" position. This is due to the actions of the unopposed lateral rectus (supplied by the abducens nerve) and the superior oblique (supplied by the trochlear nerve). The tertiary function of the superior oblique is to cause intorsion (internal rotation) of the eyeball, a function that is not usually seen unless the oculomotor nerve is paralyzed. The patient is also likely to present with a full or partial ptosis due to paralysis of the levator palpebrae muscle. The pupil will remain dilated because of loss of stimulation by parasympathetic fibers that innervate the constrictor pupillae muscle. Damage to the other nerves listed will not lead to the conditions described.

A 57-year-old man is admitted to the emergency department with dizziness and severe headaches. A CT scan evaluation reveals a tumor in the superior orbital fissure. Upon physical examination the patient's eyeball is fixed in an abducted position, slightly depressed, and the pupil is dilated. In addition, the upper lid is droopy. When the patient is asked to move the pupil toward the nose, the pupil rotates medially. Consensual corneal reflexes are normal. Which of the following nerves is most likely affected? A. Trochlear nerve B. Oculomotor nerve C. Abducens nerve and sympathetic nerve plexus accompanying the ophthalmic artery D. Ophthalmic nerve and short ciliary nerve E. Superior division of oculomotor nerve and the nasociliary nerve

B. The axons of olfactory nerves run directly through the cribriform plate to synapse in the olfactory bulb. Damage to this plate can damage the nerve axons, causing anosmia (loss of the sense of smell). A fracture of the cribriform plate is not likely to entrap the eyeball. Hyperacusis can occur following paralysis of the stapedius muscle. A lesion of the vestibulocochlear nerve can cause tinnitus and/or deafness.

A 57-year-old man is transported to the emergency department after falling from a tree. A CT scan reveals a fracture of the cribriform plate (Fig. 7-5). Which of the following conditions will most likely be present during the physical examination? A. Entrapment of the eyeball B. Anosmia C. Hyperacusis D. Tinnitus E. Deafness

C. A tumor at the cerebellopontine angle, such as an acoustic schwannoma, is most likely to affect first the vestibulocochlear nerve and then the facial nerve. This excludes the vagus, hypoglossal, glossopharyngeal, and trigeminal nerves from being the correct answers.

A 58-year-old man is admitted to the ENT clinic with progressive unilateral hearing loss and ringing in the affected ear (tinnitus) of 4 months duration. MRI reveals a tumor at the cerebellopontine angle. Which of the following nerves is most likely affected? A. Vagus B. Hypoglossal C. Vestibulocochlear D. Glossopharyngeal E. Trigeminal

D. Part of the lateral pterygoid muscle has its insertion on the articular disc within the TMJ and would be most affected by the inflammation of this joint. The temporalis muscle inserts upon the coronoid process and retracts the jaw. The medial pterygoid muscle extends from the medial surface of the lateral pterygoid plate to the mandible and functions in elevation of the jaw. The masseter extends from the zygomatic arch to the lateral ramus of the mandible and elevates the jaw. The buccinator pulls back the angle of the mouth and flattens the cheek.

A 59-year-old man is admitted to the emergency department with acute pain on his mandible. An MRI examination reveals an acute inflammation of the temporomandibular joint due to arthritis. Which of the following muscles will most likely be affected by the inflammatory process of this joint? A. Temporalis B. Medial pterygoid C. Masseter D. Lateral pterygoid E. Buccinator

B. A nonobstructive hydrocephalus occurs as a result of decreased reabsorption of cerebrospinal fluid (CSF) via the arachnoid villi. The subdural and epidural spaces are potential spaces, and these do not contain CSF. The choroid plexus produces the CSF, which flows into the subarachnoid space and is reabsorbed into the superior sagittal sinus via the arachnoid villi.

A 59-year-old man is recovering after resection of a brainstem tumor. Postoperative imaging reveals a nonobstructive (communicating) hydrocephalus. Between which structures is malabsorption most likely to occur? A. Choroid plexus and subdural space B. Subarachnoid space and superior sagittal sinus C. Subdural space and cavernous sinus D. Superior sagittal sinus and jugular vein E. Epidural and subdural space

C. The thyroidea ima artery is present in about 10% of people; when present it supplies the thyroid gland and ascends in the front of the trachea; therefore, it would be easily injured in an emergency tracheostomy with a midline incision over the trachea. The inferior thyroid branch of the thyrocervical trunk does not run along the front of the trachea in such a position that a midline incision could damage it. The cricothyroid branch of the superior thyroid artery passes across the cricothyroid ligament, well above the site of incision. Arterial bleeding would not result from damage to either the middle thyroid vein or the jugular venous arch.

A 59-year-old painter fell from the scaffolding and was admitted to the emergency department unconscious. An emergency tracheostomy is performed, and brisk arterial bleeding suddenly occurs from the midline incision over the trachea. Which of the following vessels was most likely cut accidentally? A. Inferior thyroid branch of thyrocervical trunk B. Cricothyroid branch of the superior thyroid artery C. Thyroidea ima artery D. Middle thyroid vein E. Jugular arch connecting the anterior jugular veins

D. The infratemporal fossa is a wedge-shaped region. It is positioned inferior to the temporal fossa and between the ramus of the mandibular laterally and the wall of the pharynx medially. The contents of the fossa include the temporalis, masseter, and lateral and medial pterygoid muscles. The pterygoid venous plexus and branches of the maxillary artery are also found in this fossa. Nerves passing through the fossa include the mandibular, inferior alveolar, lingual, buccal, and chorda tympani nerves. Tumors involving the infratemporal fossa present with a variety of symptoms depending on the structures involved.

A 60-year-old man presented to his family practitioner with complaints of numbness to the right side of the lower jaw and the inside of the cheek. Examination revealed loss of sensation over the scalp in the posterior temporal region and skin over the anterior auricle. There was weak lateral deviation of the mandible to the right. He was referred for a CT scan of the brain, which showed a mass in the medial temporal fossa. What other functions are most likely affected in this patient? A. Weak elevation of the corner of the right side of the mouth B. Loss of sensation over the upper lip and to the upper teeth C. Inability to sense a foreign body in the right eye D. Decreased secretion from the parotid gland E. Inability to abduct the right eye

E. The internal branch of the superior laryngeal nerve, often called the internal laryngeal nerve, supplies the mucosa of the larynx above the vocal folds (which includes the vestibule of the larynx) and the piriform recess. The external branch of the superior laryngeal nerve (external laryngeal nerve) is motor to the cricothyroid muscle. The inferior laryngeal nerve supplies the mucosa of the larynx below the vocal folds. The glossopharyngeal nerve (CN IX) supplies sensation to the posterior third of the tongue and to the pharynx. The hypoglossal nerve (CN XII) is motor

A 60-year-old man presents with swelling in his neck. A biopsy shows a benign tumor in the piriform recess of his larynx. The mucosa of the piriform recess must be anesthetized during removal of the tumor. Which nerve supplies general sensation to the mucous membrane of the laryngeal vestibule and piriform recesses? A. External laryngeal B. Glossopharyngeal C. Hypoglossal D. Inferior laryngeal E. Internal laryngeal

D. Sternocleidomastoid and trapezius muscles are supplied by the spinal accessory nerve, which enters the skull via the foramen magnum. The posterior inferior cerebellar artery is the artery nearest the accessory nerve

A 61-year-old man was admitted to the hospital after being found unconscious by his daughter. Shortly afterwards, he regained consciousness and physical examination showed that he was unable to shrug his shoulder on the right side and turn his head to the left. A CT scan demonstrated a large aneurysm of an artery compressing the nerve responsible for his symptoms. In which artery is the aneurysm? A. Superior cerebellar B. Posterior cerebral C. Anterior inferior cerebellar D. Posterior inferior cerebellar E. Basilar

B. The posterior chamber receives ciliary body secretions first. The ciliary body produces aqueous humor and is located in the posterior chamber. Increased production of fluid from this site would cause an increase in intraocular pressure if drainage is inadequate. The iridoscleral angle of the anterior chamber is the location of drainage of the aqueous humor; therefore, a blockage of drainage in this location can cause increased intraocular pressure. The pupil is the connection between the anterior and posterior chamber; a collection of fluid does not occur here, for this is simply an aperture to allow light onto the retina. The vitreous body is not directly connected to the production of aqueous humor. The lacrimal sac is the upper dilated end of the nasolacrimal duct and opens up into the inferior meatus of the nasal cavity. The nasolacrimal duct has nothing to do with increased intraocular pressure

A 62-year-old man is admitted to the hospital with blurred vision. Taking of his history and performance of a physical examination reveals a long history of gradual loss of his visual field. The intraocular pressure is high, and a diagnosis of glaucoma is made. Which of the following spaces first receives the aqueous humor secreted by the epithelium of the ciliary body? A. Anterior chamber B. Posterior chamber C. Pupil D. Vitreous E. Lacrimal sac

A. There is a lesion of the facial nerve (CN VII) proximal to the geniculate ganglion. At the geniculate ganglion the greater petrosal nerve branches from the facial nerve and ultimately runs to the pterygopalatine ganglion where preganglionic fibers synapse on postganglionic neurons that innervate the lacrimal gland. There is a disruption of the facial nerve proximal to this branch that allows the greater petrosal nerve to be stimulated by factors that would normally stimulate the submandibular and sublingual glands. These glands are innervated via the chorda tympani that comes off the facial nerve distal to the geniculate ganglion.

A 62-year-old man visits the outpatient clinic complaining of spontaneous lacrimation during eating. Which of the following nerves has developed a lesion to cause this condition? A. Facial nerve proximal to the geniculate ganglion B. Greater petrosal nerve C. Lesser petrosal nerve D. Lacrimal nerve E. Chorda tympani

D. Paralysis of the right accessory and hypoglossal nerves is present in this patient. Drooping of the right shoulder occurs as a result of paralysis of the trapezius as a result of injury to the right accessory nerve, which supplies that muscle. Loss of the right accessory nerve would also result in weakness in turning the head to the left, a function of the right sternocleidomastoid muscle, which is supplied by this nerve. The tongue deviation to the right is due to the unopposed activity of the left tongue muscles since the right hypoglossal nerve (which innervates the right tongue muscles) is affected. The other combinations of affected cranial nerves would not produce the specific symptoms described here.

A 63-year-old man had his prostate gland tumor removed 2 years before his present admission to the hospital, complaining of various neurologic problems including headaches. Radiographic examination reveals that the cancer has spread from the pelvis to the posterior cranial fossa by way of the internal vertebral venous plexus (of Batson). During physical examination the patient's right shoulder droops noticeably lower than the left, he exhibits considerable weakness in turning his head to the left, and his tongue points to the right when he attempts to protrude it directly from his mouth. There are no other significant findings. Which of the following nerves are most likely affected? A. Right vagus, right accessory, and right hypoglossal nerves B. Left accessory, right glossopharyngeal, right vagus, and left hypoglossal nerves C. Left hypoglossal, right trigeminal, and left glossopharyngeal nerves D. Right accessory and right hypoglossal nerves E. Left facial, left accessory, right accessory, and vagus nerves

C. The tumor is compressing the facial nerve, which runs through the internal acoustic meatus along with the vestibulocochlear nerve. The facial nerve provides the sensation of taste to the anterior two thirds of the tongue via the chorda tympani and also mediates all of the facial muscles, except the muscles of mastication. The mandibular branch of the trigeminal nerve courses through the foramen ovale and mediates motor to the muscles of mastication and sensory to the lower third of the face. The maxillary branch of the trigeminal branch passes through the foramen rotundum and is sensory to the middle third of the face. The jugular foramen has the glossopharyngeal, vagus, and accessory nerves coursing through it. Finally, the superior orbital fissure has the ophthalmic branch of the trigeminal nerve coursing through it, along with the oculomotor, trochlear, and abducens nerves.

A 63-year-old man with hearing loss in his left ear complains of a loss of taste and drooling from the left side of his mouth. A CT scan shows a tumor compressing the nerve exiting the skull through which of the following openings? A. Foramen ovale B. Foramen rotundum C. Internal acoustic meatus D. Jugular foramen E. Superior orbital fissure

E. The aneurysm is located near the basilar artery between the left superior cerebellar and posterior cerebral arteries. This location is the origin of the oculomotor nerve, which innervates the extraocular eye muscles with the exception of the lateral rectus (innervated by the abducens nerve) and the superior oblique (done by the trochlear nerve). Compression of this nerve by an aneurysm will result in the eye being abducted (lateral rectus) and depressed (superior oblique).

A 63-year-old woman presents with ptosis of the left eye. On examination the left eye is abducted and depressed and the light reflex is absent. Angiography reveals an aneurysm (arrow, Fig. 7-24) of an artery compressing the nerve responsible for the symptoms. Which of the following describes the location of the aneurysm? A. ICA in the cavernous sinus B. Division of the ICA into middle and anterior cerebral arteries C. Left vertebral artery at the junction with the basilar artery D. Union of left and right vertebral arteries forming basilar artery E. Basilar artery between the left superior cerebellar and posterior cerebral arteries

C. This patient has gustatory sweating, also called Frey's syndrome. When his parotid gland was removed the postganglionic nerves that innervate the glandular epithelium of his parotid gland were cut, but the nerve cell bodies of those nerves, which are located in the otic ganglion, were undamaged and able to regenerate fibers. These fibers travel with the auriculotemporal nerve, and they "sought out" glandular epithelium. The closest glands are sweat glands in the skin of the face. Under circumstances that normally induce salivation (seeing appetizing food in this case), sweating occurs instead.

A 64-year-old man has a parotidectomy because of a malignant tumor in his parotid gland. About a year after the surgery he notices that when he sees appetizing food he sweats on the side of his face where the tumor was removed. Where are the nerve cell bodies for the nerve fibers that are now innervating his sweat glands? A. Ciliary ganglion B. Pterygopalatine ganglion C. Otic ganglion D. Submandibular ganglion E. Superior salivatory nucleus

C. The oculomotor nerve passes between the posterior cerebral artery (PCA) and the superior cerebellar artery near the junction of the midbrain and pons. The optic nerve arises near the arterial circle of Willis close to the internal carotid artery. Its location would thus prevent compression following an aneurysm at the PCA and superior cerebellar artery. Although the trochlear nerve could be compressed by the superior cerebellar artery, it would not likely be damaged by an aneurysm of the PCA. The abducens nerve is located in the pons, and the vagus is situated near the postolivary sulcus in the medulla. Neither of these nerves is likely to be compressed by the arteries mentioned here due to their more distal location.

A 65-year-old man is admitted to the emergency department after an episode of a transient ischemic attack. Radiographic examination reveals an aneurysm in the region between the posterior cerebral artery and superior cerebellar artery. Which of the following nerves will most likely be compressed from the aneurysm? A. Trochlear B. Abducens C. Oculomotor D. Vagus E. Optic

C. Just before it passes into the mandible to supply the lower teeth and chin, the inferior alveolar nerve gives rise to the mylohyoid nerve, a motor nerve supplying the mylohyoid and anterior belly of the digastric. The geniohyoid muscle is innervated by motor fibers from spinal nerve C1 that run with the hypoglossal nerve. The hyoglossus muscle is innervated by the hypoglossal nerve. The stylohyoid muscle is innervated by the facial nerve. The palatoglossus muscle is innervated by the vagus nerve.

A 65-year-old man is admitted to the emergency department after his head hit the dashboard in an automobile collision. Radiographic and physical examinations reveal that the inferior alveolar nerve is injured at its origin. Which of the following muscles would most likely be paralyzed as a result? A. Geniohyoid B. Hyoglossus C. Mylohyoid D. Stylohyoid E. Palatoglossus

D. Subdural bleeding usually results from tears in veins that cross the subdural space, between the dura and the arachnoid. This bleeding may cause a gradual increase in intracranial pressure and may result in leakage of venous blood over the right cerebral hemisphere with a variable rate of progression. A subarachnoid bleed is due to rupture of an artery into the subarachnoid space surrounding the brain, between the arachnoid membrane and the pia mater. Hydrocephalus may result if the subarachnoid bleeding or subsequent fibrosis creates obstructions to CSF flow through the subarachnoid space or drainage of the CSF. Epidural bleeding results in most cases from tearing of the middle meningeal artery, and this rapidly expanding, space-occupying lesion can cause death within 12 hours. Intracerebral bleeding into the brain parenchyma is focal bleeding from a blood vessel into the brain parenchyma, most likely caused by hypertension and/or atherosclerosis. Typical symptoms include focal neurologic deficits, with abrupt onset of headache, nausea, and impairment of consciousness. Bleeding into the cerebral ventricular system may be due to trauma or hemorrhage of blood from nearby arteries, especially those related to the supply of the choroid plexus.

A 65-year-old man is admitted to the hospital 3 weeks after a "small bump of his head," according to his narrative. He suffered the accidental bump from a low-hanging branch while driving his tractor through the apple orchard during harvesting season. During physical examination the patient displays mental confusion and poor physical coordination. Radiologic examination reveals an intracranial thrombus probably due to leakage from a cerebral vein over the right cerebral hemisphere. From what type of bleeding is the patient most likely suffering? A. Subarachnoid bleeding B. Epidural bleeding C. Intracerebral bleeding into the brain parenchyma D. Subdural bleeding E. Bleeding into the cerebral ventricular system

E. Within the cavernous sinus the abducens nerve is in intimate contact with the internal carotid artery. Therefore, an aneurysm of the internal carotid artery could quickly cause tension or compression on the abducens nerve. This would result in ipsilateral paralysis of abduction of the pupil. Inability to gaze downward and medially would be due to the trochlear nerve, which is not in the cavernous sinus. Complete ptosis would be a result of a complete lesion in the oculomotor nerve, which is not apparent here. Bilateral loss of accommodation and loss of pupillary reflex would be the result of bilateral loss of the oculomotor nerve, which is not likely in this situation. Finally, ipsilateral loss of the consensual corneal reflex is a result of loss of both the ophthalmic division of the trigeminal nerve and the facial nerve, supplying the afferent and efferent limbs of the reflex, respectively.

A 65-year-old woman is admitted to the hospital with signs of cavernous sinus thrombosis. Radiographic examination reveals an aneurysm of the internal carotid artery within the cavernous sinus. During physical examination what sign would one expect to see first if nerve compression has occurred within the cavernous sinus? A. Inability to gaze downward and medially on the affected side B. Complete ptosis of the superior palpebra (upper eyelid) C. Bilateral loss of accommodation and loss of direct pupillary reflex D. Ipsilateral loss of the consensual corneal reflex E. Ipsilateral paralysis of abduction of the pupil

B. The cranial nerve emerging anterolaterally at the junction of the pons and midbrain is the oculomotor (CN III). It passes between the superior cerebellar and posterior cerebral arteries (above). Both arteries are branches of the basilar artery. An aneurysm of the PCA could result in compression of the nerve and lead to oculomotor nerve palsy. This will result in the individual being unable to move the eye normally. The affected eye will be in a down and out position. The outward location of the eye is due to the lateral rectus (innervated by the sixth cranial nerve), which maintains muscle tone in comparison to the paralyzed medial rectus. The downward location is because the superior oblique muscle (innervated by the fourth cranial or trochlear nerve) is not antagonized by the paralyzed superior rectus, inferior rectus, and inferior oblique muscles.

A 65-year-old woman presented to the emergency department with complaints of severe headaches for 2 weeks. There was no history of fever, neck stiffness, or photophobia. On examination, she was in painful distress. Vital signs revealed BP 160/89 mm Hg, pulse 110 bpm, respiration rate 16/min, temperature 36° C. Range of movement of the neck was normal and funduscopy showed no abnormalities of the retina. A CT scan of the brain with contrast revealed a dilated branch of the basilar artery at the junction of the pons and midbrain on the right side. She was referred to the neurosurgeon for assessment and further management. Which of the following statements describes clinical features with which this patient might present? A. Weak abduction of the right eye B. Blurred vision from the right eye and full ptosis of the right upper eyelid C. Loss of vision from lateral fields right eye and medial fields of left eye D. Inability to detect odors through the right nostril E. Loss of sensation of the skin over the right forehead, cheek, and mandible

B. Aqueous humor is secreted by the ciliary body into the posterior chamber of the eye, just behind the iris. The humor flows through the pupil into the anterior chamber and then is filtered by a trabecular meshwork, then drained by the scleral venous sinus (also called the canal of Schlemm). The pupil is the opening in the iris, which leads from the posterior chamber to the anterior chamber. Vitreous humor, not aqueous humor, is found in the vitreous body. The lacrimal sac is involved with tears, not the secretion of aqueous humor.

A 67-year-old man visits the outpatient clinic with complaints of deteriorating vision. A form of glaucoma is diagnosed in which the aqueous humor does not drain properly into the scleral venous sinus at the iridoscleral angle of the eyeball. The aqueous fluid is secreted by the epithelium of the ciliary body directly into which of the following spaces? A. Iridoscleral angle B. Posterior chamber C. Pupil D. Vitreous body E. Lacrimal sac

C. Rinne's test is often employed during physical examination to determine possible conduction hearing loss. A tuning fork is struck and placed on the mastoid process. It is then placed near the external ear until the patient can no longer detect vibrations. In a normal healthy patient the air conduction will be better than the bone conduction. Rinne's test is often used in conjunction with the Weber test to rule out sensorineural hearing loss.

A 67-year-old man visits the outpatient clinic with hearing problems. During physical examination a Rinne's test for hearing is performed by placing a tuning fork on his head to test for bone conduction. Upon what specific point should the tuning fork be placed to test conduction? A. Temporal bone B. Frontal bone C. Mastoid process D. External occipital protuberance E. Vertex of the head

A. The spinal accessory nerve passes across the posterior triangle of the neck and innervates both the trapezius muscle and the sternocleidomastoid muscle for the respective side of the body. Upon surgical division of the nerve, the patient will lose the ability to raise the ipsilateral shoulder and will demonstrate weakness in turning the head to the opposite side. The trapezius will also lose tone and the shoulder will droop. The ansa cervicalis innervates strap muscles of the neck and, if cut, would not produce drooping of the shoulder. The facial nerve does not pass through any of the triangles of the neck; however, if it were divided, paralysis would result in the muscles of facial expression. The hypoglossal nerve innervates the intrinsic muscles of the tongue, plus the genioglossus, hyoglossus, and styloglossus and, if injured, would not result in any of the patient's symptoms.

A 67-year-old woman is admitted to the emergency department with a severe swelling on the right side of her neck. An MRI examination reveals an abscess. The abscess is surgically removed from the middle of the posterior cervical triangle on the right side. During recovery the patient notices that her shoulder droops and she can no longer raise her right hand above her head to brush her hair. Which of the following nerves has most likely been iatrogenically injured? A. Accessory B. Ansa cervicalis C. Facial D. Hypoglossal E. Suprascapular

E. The anterior inferior cerebellar artery (AICA) is a major supplier of the anterior inferior portion of the cerebellum. Nerves located in close proximity would likely be affected by hemorrhage of this artery. The optic, oculomotor, and trochlear nerves are all associated with the midbrain region and would likely not suffer any damage with a possible hemorrhage. The trigeminal nerve is situated in the pons and is thus located too far rostrally to be affected. The abducens nerve is situated at the pontomedullary junction and is therefore most likely to be damaged following hemorrhage of the AICA.

A 68-year-old man is admitted to the emergency department after an acute cerebral vascular accident (stroke). Radiologic studies reveal that the primary damage was to the anterior inferior cerebellar artery, resulting in a small hemorrhage of the artery at its origin from the main trunk. Which of the following nerves will most likely be immediately affected by the hemorrhage? A. Optic nerve B. Oculomotor nerve C. Trochlear nerve D. Trigeminal nerve E. Abducens nerve

B. The trigeminal ganglion, also known as the semilunar or Gasserian ganglion, is the location of the sensory neuron cell bodies of the trigeminal nerve (CN V). Tic douloureux (trigeminal neuralgia) is a condition in which pain occurs over the area of distribution of trigeminal nerve branches. The geniculate ganglion is found on the facial nerve (CN VII) and receives sensory fibers for taste and transmits preganglionic parasympathetic fibers. The inferior glossopharyngeal ganglion is part of the glossopharyngeal nerve (CN IX), not the trigeminal nerve, and is not the site of the cell bodies mediating the pain. The otic ganglion, located on the mandibular division of the trigeminal nerve, contains postganglionic parasympathetic cell bodies for parotid secretion. The pterygopalatine ganglion, located in the pterygopalatine fossa, also contains postganglionic parasympathetic cell bodies for lacrimation and mucosal secretion.

A 68-year-old woman is suffering from excruciating, sudden bouts of pain over the area of her midface. Physical examination indicates that she has tic douloureux (trigeminal neuralgia). Which ganglion is the location of the neural cell bodies of the nerve mediating the pain? A. Geniculate B. Trigeminal (semilunar or Gasserian) C. Inferior glossopharyngeal D. Otic E. Pterygopalatine

C. The angiograph provided clearly shows that the radiopaque medium injected into the patient did not completely fill the common carotid artery. Portions of the internal and external carotid arteries are filled above the common carotid due to "back fill" provided by the collateral circulation. However, vascular supply to the brain is still compromised in this patient, leading to her symptoms.

A 68-year-old woman visits the outpatient clinic with a complaint of chronic dizziness and headaches and a history of multiple transient ischemic attacks. Cranial and cervical angiography (Fig. 7-11) reveals an occluded vessel. Which of the following vessels is most likely occluded? A. External carotid B. Internal carotid C. Common carotid D. Vertebral E. Superior thyroid

D. Toxoplasmosis infection is caused by the parasite Toxoplasma gondii, which is associated with undercooked meat and the feces of cats. Whereas it is a relatively common infection, once one has been exposed, one has immunity. The biggest concern is when a pregnant woman is exposed who has not been previously exposed. Congenital malformation, microphthalmia being one of the more common, can occur if the infection is passed on to the fetus.

A 7-day-old infant is admitted to the pediatric intensive care unit with microphthalmia. Which of the following is the most likely cause of this condition? A. Infection with rubella virus B. Choroid fissure failed to close C. Persistent hyaloid artery D. Toxoplasmosis infection E. Epstein-Barr virus infection

C. A lesion of the facial nerve is likely to lead to the symptoms described (drooping mouth, unable to close right eye, and food collection in the oral vestibule) because the muscles of facial expression are paralyzed. There is a bony prominence over the facial nerve located on the medial wall of the middle ear. Because of its close proximity, the facial nerve can be very rarely damaged due to otitis media. The other nerves listed are not located in close proximity to the middle ear and, if injured, would not present with the symptoms described.

A 7-year-old boy was suffering from a severe infection of the middle ear (otitis media), which spread to the mastoid air cells (mastoiditis). Surgery was required but resulted in the following: right corner of the mouth drooping, unable to close his right eye, food collection in his right oral vestibule. Which nerve was injured? A. Glossopharyngeal B. Vagus C. Facial D. Maxillary division of the trigeminal nerve E. Mandibular division of the trigeminal nerve

D. The pharyngotympanic (eustachian) tube connects the middle ear and the nasopharynx and is the conduit for spreading infections. The choanae are the openings between the nasal cavity and the nasopharynx, but they are not involved in spreading infection. The internal and external auditory meatuses are not directly associated with the middle ear but are associated with the inner and outer ear, respectively. The pharyngeal recess is a slitlike opening located behind the entrance to the auditory tube in the nasopharynx. Adenoids, enlarged masses of lymphoid tissue, can develop there.

A 7-year-old boy with a high fever is brought to the pediatrician. During physical examination the patient complains of pain in his ear. His throat appears red and inflamed, confirming the diagnosis of pharyngitis. Which of the following structures provided a pathway for the infection to spread to the tympanic cavity (middle ear)? A. Choanae B. Internal acoustic meatus C. External acoustic meatus D. Pharyngotympanic tube E. Pharyngeal recess

E. The jugular foramen is the route of exit for three nerves (glossopharyngeal [CN IX], vagus [CN X], and spinal accessory [CN XI] nerves) and one vein (internal jugular vein) from the cranial cavity. The glossopharyngeal nerve provides the sensory input for the gag reflex, whereas the vagus nerve provides the motor output. Nerve compression within this foramen would lead to a loss of both systems and thus no gag reflex. Tongue movements are almost entirely supplied by the hypoglossal nerve (CN XII), which exits the skull through the hypoglossal canal. The facial nerve (CN VII) innervates the muscles of the face and would not be affected by this injury. Loss of sensation from the face and scalp would be present only if there was involvement of the trigeminal nerve. Loss of hearing would be present with any compression of the vestibulocochlear nerve (CN VIII). The vestibulocochlear nerve was formally called the auditory nerve but vestibulocochlear clearly refers to its dual function and hearing

A 70-year-old man is admitted to the hospital with chronic headache and enlarged lymph nodes. A CT scan shows a tumor at the jugular foramen. Which of the following would be the most likely neurologic deficit? A. Loss of tongue movements B. Loss of facial expression C. Loss of sensation from the face and the scalp D. Loss of hearing E. Loss of gag reflex

D. The vagus nerve is responsible for sensation in the mucosa of the larynx down to the level of the vocal folds, and also motor innervation of the muscles that initiate a cough reflex and swallowing (motor). The mandibular division of the trigeminal nerve provides sensory innervation to the mouth and lower and lateral face and motor innervation to the muscles of mastication. The maxillary division of the trigeminal nerve provides only sensory innervation to the midfacial region surrounding the maxillary bone. The glossopharyngeal nerve provides sensory innervation to the pharynx (gag reflex) and motor innervation to the stylopharyngeus muscle. The hypoglossal nerve innervates most of the muscles of the tongue and is not associated with the cough reflex.

A 70-year-old man is admitted to the hospital with severe headaches. During the physical examination he has difficulty coughing and swallowing. An MRI scan shows a tumor affecting a cranial nerve. Which nerve is most likely affected? A. Mandibular B. Maxillary C. Glossopharyngeal D. Vagus E. Hypoglossal

B. The vagus nerve (CN X) is one of the major sensory nerves to the external auditory meatus and is the afferent limb of the cough reflex. The vestibulocochlear nerves mediates hearing and balance. Although the auriculotemporal branch of the trigeminal contributes in the innervations of the external acoustic meatus, it does not participate in the cough reflex. The facial nerve, which also contributes in the innervations of the external acoustic meatus, has no role in the cough reflex. The accessory nerve is motor to the sternocleidomastoid and trapezius nerves.

A 70-year-old man presents to his otologist for evaluation of sudden hearing loss. Upon examination the physician visualizes a large amount of cerumen (earwax) in the external auditory meatus and removes it to fully evaluate the tympanic membrane. During this process the patient begins to cough. The cough results from stimulation of an area of the meatus that is innervated by which nerve? A. Vestibulocochlear B. Vagus C. Trigeminal D. Facial E. Accessory

E. The vocalis muscles lie medial to the thyroarytenoid muscles and lateral to the vocal ligaments within the vocal folds. The vocalis muscles produce minute adjustments of the vocal ligaments, selectively tensing and relaxing the anterior and posterior parts, respectively, of the vocal folds during animated speech and singing.

A 70-year-old patient with sudden-onset hoarseness is found to have a small carcinoma of the mucosa overlying the right vocalis muscle. Which of the following is most likely located immediately lateral to this muscle? A. Lateral cricoarytenoid muscle B. Aryepiglottic muscle C. Posterior cricoarytenoid muscle D. Oblique arytenoids muscle E. Thyroarytenoid muscle

D. The hypoglossal nerve (CN XII) passes through hypoglossal canal. All the muscles of the tongue, intrinsic and extrinsic, are supplied by the hypoglossal nerve (except palatoglossus). Extrinsic muscle supplies the genioglossus, hyoglossus, and styloglossus. Thus compression of the nerve will affect intrinsic and extrinsic muscles of the tongue.

A 72-year-old man presents at the clinic with difficulty when eating. An MRI showed a tumor of the brainstem affecting the contents of the hypoglossal canal. Which of the following muscles would be affected by such a tumor? A. Geniohyoid B. Mylohyoid C. Palatoglossus D. Genioglossus E. Stylohyoid

B. The supraclavicular lymph node on the left side is associated with the thoracic duct. The thoracic duct receives lymph from below the diaphragm, including the gastrointestinal tract. Malignant cells that travel up the thoracic duct are known to involve the left supraclavicular lymph node. Historically this is called Virchow's node or Troissier's sign when enlargement is found on palpation.

A 72-year-old man with enlarged cervical lymph nodes has a malignant tumor of the cecum in the right lower quadrant of his abdomen. Which of the following lymph nodes of the neck is most frequently associated with malignant tumors of the gastrointestinal tract? A. Left inferior deep cervical B. Left supraclavicular C. Right inferior deep cervical D. Right supraclavicular E. Jugulodigastric

E. A Pancoast tumor is located in the pulmonary apex, usually in the right lung. (This is because inhaled gases tend to collect preferentially in the upper right lung, in part because of the manner of branching of the tertiary bronchi.) These tumors can involve the sympathetic chain ganglia and cause Horner's syndrome (slight ptosis and miosis). The other conditions listed are not likely to cause symptoms of Horner's syndrome. Raynaud's disease, a vascular disorder that affects the extremities, is caused by excessive tone of sympathetic vasoconstriction. Frey's syndrome, a rare malady resulting from parotidectomy, is characterized by excessive facial sweating in the presence of food or when thinking about it. Bell's palsy is characterized by a lesion of the facial nerve, with weakness or paralysis of mimetic muscles. Quinsy is characterized by painful, pus-filled inflammation of the tonsils.

A 72-year-old woman is admitted to the emergency department with tenderness in the upper right thorax, painful to compression. During physical examination the patient presents with slight ptosis of her right eyelid. The right pupil is constricted more distinctly than the contralateral pupil. Which of the following is the most likely diagnosis? A. Raynaud's disease B. Frey's syndrome C. Bell's palsy D. Quinsy E. Pancoast tumor

C. The thrombus may pass through the central vein of the retina to reach the cavernous sinus. The patient would suffer blindness because the central vein is the only vein draining the retina and if it is occluded, blindness will ensue. The subarachnoid space would not be associated with the blindness experienced. Thrombus of the central artery would not cause cavernous sinus thrombophlebitis. The optic chiasm is a neural structure that does not transmit thrombi. The ciliary ganglion is a parasympathetic ganglion; a thrombus in the cavernous sinus would not pass through it.

A 73-year-old man visits the outpatient clinic with a complaint of progressive, painless loss of vision. Radiographic examination reveals thrombophlebitis of the cavernous sinus. Through which of the following structures must a thrombus pass to cause the symptoms of this patient? A. Subarachnoid space B. Central artery of the retina C. Central vein of the retina D. Optic chiasm E. Ciliary ganglion

E. The rima glottidis is the opening between the true vocal cords. The diameter of this opening is regulated by the laryngeal muscles to modulate pitch of sound. The piriform fossa is a shallow space found on the lateral side of the laryngeal orifice bounded laterally by the thyroid cartilage and medially by the aryepiglottic fold. While "vestibule" can mean any opening, in the head and neck it strictly is used to mean the anterior-most portion of the oral cavity, making B an incorrect option. In the head and neck region of the human body, the ventricles are cavities above the midbrain where the CSF is produced. The vallecula is a depression behind the root of the tongue that serves to hold saliva for lubrication and prevention of premature initiation of deglutition reflex.

A 73-year-old patient presents at the clinic with neck pain, hoarseness, and weight loss of 6 months duration. During laryngoscopy a large tumor is identified on one of the true vocal folds. The ENT specialist is unable to pass the scope through the opening between the folds. What is the name of this opening? A. Piriform recess B. Vestibule C. Ventricle D. Vallecula E. Rima glottidis

B. The internal carotid is the artery that supplies the majority of the blood supply to the brain. It has several parts including the cervical, petrous, lacerum, cavernous, clinoid, ophthalmic, and communicating segments. The cavernous sinuses are paired dural sinuses that are located on the lateral wall of the sphenoid bone on either side of the sella turcica. The internal carotid artery and the abducens nerve run through it. Damage to this part of the internal carotid would affect the abducens nerve and the patient's ability to abduct the eye.

A 75-year-old woman presents to the emergency department complaining of double vision. Physical examination reveals inability to abduct her right eye. In which of the locations indicated in the arteriogram (Fig. 7-25) would an aneurysm most likely be located to cause the nerve compression resulting in these symptoms/signs? A. PCA B. ICA C. External carotid artery D. Anterior cerebral artery E. Posterior communicating artery

A. Mastoiditis is an infection of the air cells within the mastoid process of the temporal bone, often caused by untreated acute otitis media. A known complication of mastoiditis is inflammation of the transverse sinus. Necrosis of the bone due to untreated infection will often affect the transverse sinus. The petrous part of the temporal bone is unlikely to experience inflammation. Infection in the middle ear is usually the preceding event to mastoiditis rather than occurring as a result of it. The occipital sinus is located far posteriorly to the mastoid process and is unlikely to be affected. Because of its position, the internal carotid artery will not be affected by this inflammation.

A 9-year-old girl is admitted to the emergency department with painful swelling behind her ear. An MRI examination reveals mastoiditis (Fig. 7-12). Which of the following structures is most likely to be affected by the inflammation? A. Transverse sinus B. Petrous part of the temporal bone C. Inner ear D. Occipital sinus E. Internal carotid artery

D. Cephalohematoma is hemorrhage between the cranium and the periosteum of the cranial bones by rupture of vessels crossing the periosteum. In contrast, subgaleal (subaponeurotic) hemorrhage is hemorrhage between the periosteum and the aponeurotic layer and more extensive with more complications. The swelling does not cross suture lines in cephalhematoma but does in subgaleal hemorrhage. Caput succedaneum is collection of edema fluid above the periosteum of the cranial bones.

A neonate is observed by the pediatric team to have bilateral swelling on both sides of the head (Fig.7-16). On physical examination the swellings do not cross the suture lines and transillumination of the skull appears to be normal. Neurologic examination is normal. Which of the following is most likely responsible for this patient's symptoms? A. Intracranial bleeding B. Blockage of the interventricular foramen C. Failure of cleavage of the prosencephalon D. Collection of blood beneath the periosteum of the bone E. Collection of edema fluid above the periosteum of the bone

C. The trigeminal nerve is the major sensory nerve of the face. It has three branches: ophthalmic, maxillary, and mandibular. The neurologist was trying to elicit the corneal reflex where the afferent limb (sensation on the eyeball) is via the trigeminal nerve and the efferent limb is the facial nerve (closing the eye). The optic nerve is responsible for vision, and the oculomotor and abducens nerves innervate extraocular muscles.

A neurologist strokes a wisp of cotton across a patient's left cornea without a response. The neurologist then strokes the cotton across the patient's right cornea and both eyes blink. The most likely explanation of these findings is damage to which of the following cranial nerve on the left? A. Optic B. Oculomotor C. Trigeminal D. Abducens E. Facial

D. Rupture of the periosteal arteries resulting in a cephalohematoma is defined as a collection of blood underneath the periosteum. On the head, it is located between the pericranium (periosteum of the skull) and the calvaria (skull). The galea aponeurotica, skin and areolar connective tissue are all located superficial to the site of bleeding and hematoma.

A newborn infant is delivered with forceps after a difficult delivery. Upon physical examination of the newborn a cephalohematoma is noted from rupture of small periosteal arteries. Between which of the following layers of tissue does the blood accumulate? A. Between skin and dense connective tissue layer B. Between loose connective tissue layer and galea aponeurotica C. Between galea aponeurotica and pericranium D. Between pericranium and calvaria E. In the subcutaneous layer

A. The larynx receives sensory innervation from the vagus nerve via the internal laryngeal nerve. The glossopharyngeal nerve does not innervate the epiglottis. The vestibulocochlear nerve mediates hearing and balance. The hypoglossal nerve supplies motor fibers to the tongue muscles. The facial nerve, while performing many functions including motor supply to muscles of facial expression and taste in the anterior two thirds of the tongue, does not supply sensory nerves to the epiglottis.

A patient is rushed to the emergency department and requires insertion of an endotracheal tube. Because the physician needs to insert the tube immediately, the patient is not given anesthesia. As the physician inserts the tube, he strikes the epiglottis, which causes the patient to grimace. Which of the following cranial nerves most likely allows the patient to sense pain from the larynx? A. Vagus B. Glossopharyngeal C. Vestibulocochlear D. Hypoglossal E. Facial

D. The olfactory nerves arise from cells in the superior part of the lateral and septal walls of the nasal cavity. The processes of these cells (forming the olfactory nerve) pass through the cribriform plate and end in the olfactory bulbs, which lie on either side of the crista galli. Therefore a tumor here compresses the nerves, and the sense of smell will be affected. The optic tract and chiasm are not likely to be affected. Similarly, the vagus, vestibulocochlear, and facial nerves are not in close proximity.

A patient with a brain tumor near the crista galli and cribriform plate of the ethmoid bone most likely experiences which of the following symptoms? A. Paralysis of facial muscles B. Loss of vision C. Difficulty swallowing D. Loss of smell E. Loss of hearing

A. The vagus nerve is responsible for innervation to the soft palate; if damaged, the uvula would deviate from the side of the lesion. Glossopharyngeal nerve damage could reduce the gag reflex because it is the afferent limb but would not produce the other symptoms. Damage to the trigeminal nerve would not produce any of these symptoms.

A physician is performing a cranial nerve examination on a 49-year-old patient. Oral examination revealed that the patient's uvula is deviated to the right. When the left side of the pharyngeal mucosa is touched, the gag reflex is weaker than when the mucosa on the right side is touched. Which nerve is most likely injured to cause these symptoms? A. Left vagus B. Left glossopharyngeal C. Right vagus D. Left V3 E. Right glossopharyngeal

C. The temporal bone parts include mastoid, petrous, squamous, and tympanic portions. The mastoid process is part of the mastoid portion of the temporal bone.

A physician palpates the mastoid process of an adolescent complaining of pain behind the ear. Which bone is the physician palpating? A. Occipital B. Zygomatic C. Temporal D. Parietal E. Sphenoid

D. The mental nerve and vessels pass through the mental foramen in the mandible. The mental nerve supplies the skin and mucous membrane of the lower lip from the mental foramen to the midline, including the skin of the chin. It is a branch of the inferior alveolar nerve, which comes off the posterior division of the mandibular division of the trigeminal nerve. The buccal nerve is a branch of the anterior division of the mandibular nerve, which supplies the skin over the buccal membrane. The auriculotemporal nerve and the great auricular nerve, a branch of the cervical plexus composed of fibers from C2 and C3 spinal nerves, innervates the parotid sheath as well as the overlying skin. The lesser petrosal nerve is the visceral motor component of the glossopharyngeal nerve (CN IX), carrying parasympathetic fibers from the tympanic plexus to the parotid gland. The infraorbital nerve innervates (sensory) the lower eyelid, upper lip, and part of the nasal vestibule and exits the infraorbital foramen of the maxilla.

A severe mandibular fracture in a 22-year-old woman requires surgical reconstruction. Postoperatively, she complains of numbness of the lower lip and chin. Which nerve was most likely injured? A. Auriculotemporal B. Buccal C. Lesser petrosal D. Mental E. Infraorbital

D. A fractured hyoid bone is evidence of strangulation. A fall from a height and subdural hematoma would likely be accompanied by fractured bones. Whereas myocardial infarction or poison remain possibilities, the medical examiner would have a high index of suspicion for strangulation because of the fractured hyoid bone.

A young man and woman are hiking in a wilderness area and discover the body of a man apparently in his 20s. He appears to have been dead for a few days, but animal predation was minimal. A postmortem examination was performed by the county medical examiner, and no evidence of penetrating wounds (bullet, lacerations, etc.) was found. A plain radiograph showed a fractured hyoid bone, but the calvaria and other bones appeared to be intact. Which of the following is the most likely cause of death? A. Myocardial infarction (heart attack) B. A fall from a height that resulted in fatal internal bleeding C. Subdural hematoma D. Strangulation E. Ingestion of a poisonous substance

B. The palatine tonsils lie in tonsillar beds with muscular (covered with mucosa) anterior and posterior pillars forming the boundaries of the bed. These pillars are formed by the palatoglossal arch, anteriorly, and the palatopharyngeal arch, posteriorly. The anterior pillar, part of the palatoglossal arch, contains the palatoglossus muscle; the posterior pillar, provided by the palatopharyngeal arch, is formed by the palatopharyngeus muscle.

An 11-year-old boy is examined by an otorhinolaryngologist for his swollen palatine tonsils. The palatine tonsils are located between the anterior and posterior tonsillar pillars. Which of the following muscles form these pillars? A. Levator veli palatini and tensor veli palatini B. Palatoglossus and palatopharyngeus C. Styloglossus and stylopharyngeus D. Palatopharyngeus and salpingopharyngeus E. Superior and middle pharyngeal constrictors

B. The palatine tonsils are highly vascular and are primarily supplied by the tonsillar branch of the facial artery; therefore, care is taken to identify and ligate or cauterize this artery while performing a tonsillectomy. The palatine tonsil also receives arterial supply from the ascending pharyngeal, the dorsal lingual, and the lesser palatine, but the supply from the facial artery is by far the most significant.

An 11-year-old boy with swollen palatine tonsils is examined by an otolaryngologist. Which of the following arteries supplies most of the blood to these tonsils? A. Ascending pharyngeal B. Facial C. Lingual D. Descending palatine E. Superior thyroid

C. Normally the tonus of the buccinator muscle prevents the accumulation of saliva and foodstuffs in the oral vestibule. Although a lesion of the facial nerve would paralyze the other muscles listed, the buccinator is the predominant muscle of the cheek.

An 8-year-old boy had a mastoidectomy due to an infection that did not respond to antibiotics. Postoperatively he had Bell's palsy (facial paralysis), and one of the features was the accumulation of saliva in the vestibule of his oral cavity and dribble from the corner of his mouth. Which of the following muscles was most likely paralyzed? A. Zygomaticus major B. Orbicularis oculi C. Buccinator D. Levator palpebrae superioris E. Orbicularis oris

A. Horner's syndrome involves interruption of sympathetic supply to the face. This results in ptosis (drooping eyelid), miosis (constricted pupil), and anhydrosis (lack of sweating) of the face. The eye is lubricated by the lacrimal gland, which secretes in response to parasympathetic stimulation, and would be unaffected. Exophthalmos (protrusion of the globe) is frequently caused by hyperthyroidism and is not present in Horner's syndrome. Loss of sympathetic innervation leads to unopposed vasodilatation of the vessels to the face, leading to flushing rather than paleness.

An 8-year-old boy is admitted to the hospital with a drooping right eyelid (ptosis) (Fig. 7-4). The initial diagnosis is Horner's syndrome. Which of the following additional signs on the right side would confirm the diagnosis? A. Constricted pupil B. Dry eye C. Exophthalmos D. Pale, blanched face E. Sweaty face

D. Of the answer choices listed, the left facial nerve of the patient is the most likely to be damaged during the mastoidectomy. The facial nerve exits the skull via the stylomastoid foramen, just anterior to the mastoid process. A lesion of the facial nerve is likely to cause the symptoms described as a result of paralysis of the facial muscles. Depending upon the site of injury, the patient could also lose the chorda tympani branch of the facial nerve, leading to loss of taste from the anterior two thirds of the tongue ipsilaterally as well as loss of functions of the submandibular and sublingual salivary glands. The other nerves listed are not likely to be damaged during a mastoidectomy.

An 8-year-old boy was suffering from a severe infection of the right middle ear. Within the course of a week, the infection had spread to the mastoid antrum and the mastoid air cells. The organisms did not respond to antibiotics, so the surgeon decided to perform a radical mastoid operation. Following the operation, it was noticed that the boy's face was distorted. The mouth was drawn upward to the left, and he was unable to close his right eye. Saliva tended to accumulate in his right cheek and dribble from the corner of his mouth. What structure was most likely damaged during the operation? A. Mandibular nerve B. Parotid duct C. Vagus nerve D. Facial nerve E. Glossopharyngeal nerve

E. Perineural spread of a tumor (or spread of tumor along a nerve) is one of the more insidious forms of tumor metastasis. This form of spread is more commonly found in malignant rather than benign lesions. In this case tumor spreads along tissue at the jugular foramen, thus involving nerves glossopharyngeal (CN IX), vagus (CN X), and spinal accessory nerves (CN XI). Therefore in case of the pharyngeal reflex (gag reflex), sensory limb is mediated predominantly by CN IX (glossopharyngeal nerve) and motor limb by CN X (vagus nerve). Tongue, facial expression and sensation and hearing are mediated by cranial nerves XII, VII, V, and VIII respectively

An 82-year-old man is being evaluated preoperatively for a tumor in the posterior cranial fossa. MRI demonstrated perineural spread of the tumor along nervous tissue at the jugular foramen. Which deficit would you expect? A. Loss of tongue movements B. Loss of facial expression C. Loss of sensation from the face and the scalp D. Loss of hearing E. Loss of gag reflex

D. The mucosa of the piriform fossa receives sensory innervation from the internal laryngeal nerve. This nerve also supplies somatic sensory fibers to the larynx above the vocal cords, epiglottis, and valleculae. Owing to its superficial location in the mucosa of the piriform fossa, it could be damaged during procedures involving the piriform fossa with loss of sensation in the areas it innervates and loss of protective reflex of the larynx. The mandibular division of the trigeminal nerve (CN V3) supplies somatic sensation the lower face and motor innervation to muscles of mastication. It also sends sensory fibers to the meninges including the external acoustic meatus. The maxillary division of the trigeminal nerve (CN V2), although a sensory component of the trigeminal, does not innervate the piriform fossa. The glossopharyngeal nerve (CN IX) supplies special visceral efferent to the stylopharyngeus muscle, parasympathetic to the parotid gland, general visceral afferent to the carotid sinus and body, special sensory to the posterior one third of the tongue, and general somatic efferent to the external ear, internal part of the tympanic membrane, posterior tongue, and upper pharynx.

An 82-year-old man presents to the ENT clinic for evaluation of a neck mass. Visual examination with a flexible laryngoscope reveals a tumor in the piriform recess. In taking a biopsy, the physician must be careful not to injure which nerve? A. Mandibular B. Maxillary C. Glossopharyngeal D. Internal laryngeal E. Hypoglossal

C. The pupillary light reflex is used to test the ability of the eyes to perceive light sensation. The afferent limb of this reflex arc is the optic nerve while the oculomotor is the efferent limb. By shining light into the eye from a penlight, each eye is tested individually for a direct constriction of the pupil on the ipsilateral side and then for a consensual reflex on the contralateral side. The startle reflex is a kind of acoustic reflex that occurs in reaction to sudden auditory stimulation. The blink or corneal reflex is a reflex that is produced in reaction to stimulation of the cornea of the eye. This reflex is mediated by the ophthalmic branch of the trigeminal (afferent limb) and the facial nerve (efferent limb). The H-test is a maneuver used to test the extraocular muscles, which are innervated by the oculomotor, trochlear, and abducens nerves. The vision test, also known as visual acuity test, is used to assess the ability of the eye to read or identify letters from different distances.

An 82-year-old man receives a home visit from his family practitioner, who notices that the man's pupils are unequally dilated. Which test would the physician use to determine which eye had the problem and whether it was a sympathetic or oculomotor nerve lesion? A. Startle reflex B. Blink reflex C. Pupillary light reflex D. H-test E. Vision test (reading chart)

D. Aneurysms of AICA cause direct impingement on the abducens nerve as it emerges from the brainstem at the pontomedullary junction between the labyrinthine artery above and the AICA below. Impingement of this nerve leads to loss of function to the lateral rectus muscle ipsilaterally, resulting in the inability to abduct the eye. The anterior communicating artery does not relate to the abducens nerve. Aneurysm of the posterior cerebral artery will likely affect the oculomotor nerve. Aneurysm of the superior cerebellar artery will also likely affect the oculomotor nerve. Vertebral artery aneurysm will likely affect the hypoglossal nerve.

An 84-year-old man presents to the emergency department complaining of double vision. Physical examination reveals inability to abduct his right eye. In which of the locations indicated in the arteriogram (Fig. 7-20) will an aneurysm most likely be located to cause the nerve compression resulting in these symptoms/ signs? A. A B. B C. C D. D E. E

C. The facial nerve is a mixed cranial nerve, which carries both motor and (special) sensory fibers. The motor component of the facial nerve arises from the facial nerve nucleus and forms the facial nerve proper, while the sensory and parasympathetic parts of the facial nerve emerge from the brain as nervus intermedius. The motor and sensory parts of the facial nerve enter the petrous part of the temporal bone via the internal auditory meatus, which is very close to the inner ear; it then courses through the facial canal, after which it emerges from the stylomastoid foramen and passes through the parotid gland, where it divides into five major branches. The facial nerve provides special sensory to the anterior one third of the tongue, motor to the muscles of facial expression and posterior belly of the digastric.

An 87-year-old woman underwent surgery to remove a vestibular schwannoma (tumor at the internal acoustic meatus). Resection was successful but resulted in the following findings postoperatively: drooping of the right corner of the mouth, inability to close the right eye, and food collection in the right oral vestibule. Which cranial nerve was injured during the surgery? A. IX B. X C. VII D. V2 E. V3

A. The glossopharyngeal nerve emerges from the surface of the medulla and travels laterally in the pontine cistern to enter the anterior compartment of the jugular foramen. It gives off the tympanic nerve, which supplies the middle ear, a carotid branch to innervate the carotid body, the nerve to the stylopharyngeus, pharyngeal branches, a lingual branch and a tonsillar branch. The tonsillar branch provides afferent fibers for the tonsillar mucosa and the lingual branch conveys common sensation and taste from the posterior part of the tongue, as well as secretomotor fibers for lingual glands. Taste of the anterior two thirds of the tongue is innervated by the chorda tympani nerve and a branch of the facial nerve (CN VII). The vagus nerve (CN X) innervates the soft palate and the constrictor muscles of the pharynx.

An otherwise healthy 11-year-old girl with recurrent upper respiratory tract infections undergoes bilateral tonsillectomy. While performing the procedure, the surgeon accidentally damages the nerve that lies in the tonsillar fossa, deep to the palatine tonsil. Which of the following is most likely to result from this injury? A. Loss of sensation on the posterior one third of the tongue B. Loss of taste on the anterior two thirds of the tongue C. Paralysis of the constrictor muscles of the pharynx D. Paralysis of the muscles of the soft palate E. Paralysis of the muscles of the tongue

A. The superior cerebellar artery arises near the termination of the basilar artery, passes immediately below the oculomotor nerve, and eventually winds around the cerebral peduncle, close to the trochlear nerve, as it continues on toward the upper surface of the cerebellum where it will divide into branches that anastomose with the inferior cerebellar arteries. The trochlear nerve passes between the posterior cerebral artery and the superior cerebellar artery, and therefore a hematoma of the superior cerebellar artery can easily injure the trochlear nerve, which runs alongside the internal carotid artery and then enters the orbit through the superior orbital fissure. The facial and vestibulocochlear nerves both enter the skull via the internal acoustic meatus (or internal auditory meatus) in the temporal bone and do not have an intimate relationship with the superior cerebellar artery. The glossopharyngeal nerve passes through the jugular foramen, and as it exits from the skull it passes forward between the internal jugular vein and internal carotid artery.

An unconscious 64-year-old man is admitted to the hospital. A CT scan examination reveals that the patient has suffered a cerebral vascular accident (stroke), with a small hematoma produced by the superior cerebellar artery. Which of the following nerves will most likely be affected by the hematoma? A. Trochlear nerve B. Abducens nerve C. Facial nerve D. Vestibulocochlear nerve E. Glossopharyngeal nerve

E. The vagus nerve mediates both the afferent and efferent limbs of the cough reflex: afferent to the mucus membranes and efferent to the muscles of the larynx. The glossopharyngeal nerve is involved in the gag reflex as the afferent limb. The hypoglossal nerve innervates most of the tongue. The external laryngeal nerve is a branch of the vagus nerve that provides only motor innervation to the cricothyroid and cricopharyngeus muscles. The trigeminal nerve does provide sensory innervation to the face.

During a test of the cough reflex a 62-year-old woman inhales air containing different amounts of particles that will adhere to mucus primarily in the trachea. Blockade of afferent neurons in which cranial nerve will most likely suppress this woman's cough reflex? A. Glossopharyngeal B. Hypoglossal C. External laryngeal D. Trigeminal E. Vagus

E. The nerve responsible for innervating the muscles of the larynx and pharynx is the vagus nerve, which also initiates the cough reflex. The muscles of mastication and facial expression are supplied by trigeminal and facial nerves, respectively. The vestibulocochlear nerve mediates hearing and balance. The oropharyngeal mucosa is supplied by the glossopharyngeal nerve.

Following a small brainstem stroke of limited size, a 64-year-old diabetic woman has uncoordinated swallowing and difficulty with phonation. The cranial nerve responsible for the dysfunction will most likely also produce which of the following symptoms? A. Weakness/paralysis of muscles of mastication on one side B. Weakness/paralysis of muscles of facial expression on one side C. Hearing loss and balance problems D. Decreased/absent sensation in the oropharynx E. Decreased/absent cough reflex

B. The external laryngeal nerve supplies the cricothyroid muscle responsible for tensing the vocal cords. Loss of sensation would not result from injury to a motor nerve. Inability to abduct the vocal folds would result from damage to the recurrent laryngeal nerve.

Following thyroidectomy for multiple benign thyroid nodules in a 44-year-old woman, a diagnosis of surgical injury to the left external laryngeal nerve is established. This may result in which of the following symptoms and signs? A. Inability to abduct the vocal fold B. Monotone, easily fatigued voice with poor pitch control C. Decreased/absent sensation above the vocal folds D. Decreased/absent sensation below the vocal folds E. Decreased/absent cough reflex

A. Sensory innervation to the area of the larynx superior to the vocal folds is supplied by the superior laryngeal nerve. The recurrent laryngeal nerve supplies sensation to the area inferior to the vocal cords. The vagus (CN X) and glossopharyngeal (CN IX) nerves bring about the swallowing reflex. Weakness to the posterior cricoarytenoid muscles would be due to recurrent laryngeal nerve damage.

Following thyroidectomy, a 44-year-old female soprano has persistent problems with phonation. Fiberoptic laryngoscopy reveals that the vocal folds look normal and meet in the midline. A diagnosis of surgical injury of the left superior laryngeal nerve is established. Which of the following additional abnormal findings would be expected in this nerve injury? A. Decreased/absent sensation above the vocal folds B. Decreased/absent sensation below the vocal folds C. Poorly coordinated swallowing reflex D. Bilateral weakness/paralysis of the posterior cricoarytenoid muscles E. Weakness/paralysis of the left posterior cricoarytenoid muscle

C. The arterial circle (of Willis) receives its blood supply from the internal carotid and vertebral arteries. The actual circle is formed by the bifurcation of the basilar, posterior cerebral, posterior communicating, internal carotid, anterior cerebral, and anterior communicating arteries. The middle cerebral artery is the lateral continuation of the internal carotid artery and therefore not part of the arterial circle. Although it receives its blood supply from the arterial circle (of Willis), it does not actually form any part of the circle.

The arterial circle (of Willis) contributes greatly to cerebral arterial circulation when one primary artery becomes occluded by atherosclerotic disease. Which of the following vessels does not contribute to the circle? A. Anterior communicating artery B. Posterior communicating artery C. Middle cerebral artery D. Internal carotid artery E. Posterior cerebral artery

D. The otic ganglion is the location of the postganglionic parasympathetic neural cell bodies innervating the parotid gland. The ganglion lies on the mandibular division of the trigeminal nerve near the foramen ovale. The trigeminal ganglion contains cell bodies for neurons innervating sensory aspects of the face. The inferior salivatory nucleus lies within the brainstem and contains preganglionic parasympathetic neurons whose axons pass within the lesser petrosal nerve to the otic ganglion for synapse in the supply of the parotid. The superior cervical ganglion has the cell bodies of postganglionic sympathetic fibers innervating sympathetic structures to the head. The submandibular ganglia contain the cell bodies of postganglionic parasympathetic fibers innervating the sublingual and submandibular salivary glands.

Where is the location of the postganglionic parasympathetic neural cell bodies that directly innervate the parotid gland? A. Trigeminal (semilunar, Gasserian) ganglion B. Inferior salivatory nucleus C. Superior cervical ganglion D. Otic ganglion E. Submandibular ganglion

C. The auriculotemporal nerve, a branch of the mandibular division of the trigeminal nerve, passes posteriorly, deep to the ramus of the mandible and superior to the deep part of the parotid gland, emerging posterior to the temporomandibular joint to supply the skin anterior to the auricle and posterior two thirds of the temporal region. The nerve distributes to the skin of the tragus and adjacent helix of the auricle and therefore of the external acoustic meatus and skin of the superior tympanic membrane. The lesser occipital nerve, a branch of the cervical plexus, supplies the skin posterior to the auricle. The great auricular nerve, also a cervical plexus branch, supplies the skin overlying the mandible and the capsule of the parotid gland. The zygomaticotemporal nerve supplies the hairless patch of skin over the anterior part of the temporal fossa. The greater occipital nerve supplies the occipital part of the scalp.

While at a party, an intoxicated teenage girl asks her friend to pierce the tragus of her left ear. While attempting to pass a needle through the tragus, the friend slips and the needle deeply punctures the skin directly anterior to the tragus. The next morning the teenager awakens to find that she has no feeling on the left temporal side of her scalp up to the vertex of her head. Which of the following nerves is most likely damaged? A. Lesser occipital B. Greater occipital C. Auriculotemporal D. Zygomaticotemporal E. Great auricular


Conjuntos de estudio relacionados

MGMT 470 Test 3 Review (6-10 and case studies)

View Set

Finance EOC Questions: Concept Reviews

View Set

Community Nutrition Exam ch 10-14

View Set

Cultural Antrhopology Final Exam

View Set

CNT 216: Module 10 - LAN Security Concepts

View Set